SCE Nephrology Book 20106

You might also like

Download as pdf or txt
Download as pdf or txt
You are on page 1of 210

Disorders of fluid and electrolyte and acid base regulation

& hypertension

QUESTIONS
Question: 1

A 16-year-old boy was referred for investigation of renal impairment and


hypertension. These had been identified during an assessment for possible
diabetes mellitus. He gave a 12 month history of increasing polyuria and
polydipsia. He was taking no regular medications. Slit lamp examination
findings are shown in the picture (a). His blood pressure was 164/98 mmHg.

Investigations:

serum potassium 3.0 mmol/L (3.5–4.9)


serum bicarbonate 18 mmol/L (20–28)
serum creatinine 110 µmol/L (60–110)
eGFR 80 ml/min
serum phosphate 0.60 mmol/L (0.8–1.4)
serum chloride 112 mmol/L (95–107)

urinalysis trace blood, 2+ protein


1+ glucose

urinary protein:creatinine ratio 56 mg/mmol (<15)


a

What would be the most appropriate treatment?

A cysteamine
B elimination of dietary lactose
C penicillamine
D prednisolone
E pyridoxine

NOTES:

2
Question: 2

A 19-year-old woman was referred for investigation of progressive


generalised muscle weakness and lethargy. She was taking no regular
medications. Examination was unremarkable. Her blood pressure was 110/60
mmHg. Her body mass index was 19.

Investigations:

serum potassium 2.7 mmol/L (3.5–4.9)


serum bicarbonate 34 mmol/L (20–28)
serum creatinine 72 µmol/L (60–110)
eGFR >90 ml/min
serum magnesium 0.60 mmol/L (0.75–1.05)
serum chloride 83 mmol/L (95–107)

urinary chloride 60
urinary sodium 55
24-hour urinary calcium 1.5 mmol (2.5–7.5)

What is the most likely diagnosis?

A Bartter’s syndrome
B Gitelman’s syndrome
C hypokalaemic periodic paralysis
D Liddle’s syndrome
E syndrome of apparent mineralocorticoid excess

NOTES:

3
Question: 3

A 26 year old woman presented with generalised muscle weakness and


recurrent cramps in both arms. On examination her blood pressure was 95/60
mmHg.

Investigations:

serum sodium 136 mmol/L (137–144)


serum potassium 2.0 mmol/L (3.5–4.9)
serum urea 5.3 mmol/L (2.5–7.0)
serum creatinine 86 µmol/L (60–110)
serum magnesium 0.5 mmol/L (0.75–1.05)
serum bicarbonate 34 mmol/L (20–28)
urine chloride 40 mmol/24hrs (100 – 250)

What is the next most appropriate investigation?

A dexamethasone suppression test


B genetic testing for CYP11B1/CYP11B2
C renin-angiotensin ratio
D salivary sodium
E urinary calcium

NOTES:

4
Question: 4

An 18 year old man presented with a 6 month history of worsening tiredness


and muscle aches. A male cousin attends nephrology follow-up but it is not
clear from the patient why. He was taking no regular medications. On
examination his blood pressure was 126/72 mmHg.

Investigations:

serum potassium 3.0 mmol/l (3.5–4.9)


serum bicarbonate 18 mmol/l (20–28)
serum phosphate 0.55 mmol/l (0.8–1.4)
plasma glucose 5.2 mmol/l
serum creatinine kinase 500 U/L (24–195)

urinalysis +protein, +glucose

Which is the most appropriate diagnostic test?

A CT KUB
B glucose tolerance test
C plasma alpha-galactosidase-A level
D renal biopsy
E serum bicarbonate after oral loading with sodium hydrogen carbonate

NOTES:

5
Question: 5

An 18 year old man presented with accelerated hypertension. His father and
paternal uncle developed hypertension in their early 20s. After initial
intravenous antihypertensive therapy, he was treated with amlodipine 10mg
od and ramipril 10mg od. His blood pressure was 142/86 mmHg.

Investigations:

serum potassium 2.7 mmol/L (3.5–4.9)


serum bicarbonate 36 mmol/L (20–28)
plasma renin activity 2.9 pmol/mL/h (3.0–4.3)
plasma aldosterone 950 pmol/L (330–830)
spot urinary potassium 35 mmol/L

Which statement is most correct

A Cushing’s syndrome is a likely diagnosis.


B dexamethasone is the treatment of choice.
C he is a compulsive liquorice eater.
D he should be screened for 11-β hydroxysteroid deficiency.
E his blood pressure and hypokalaemia will respond well to inhibiting distal
tubular sodium reclamation.

NOTES:

6
Question: 6

A 42 year old woman was found to have hypertension and was prescribed
ramipril. She had a strong family history of hypertension. Six weeks later she
developed generalized weakness. Physical examination was unremarkable.
Her blood pressure was 136/80 mmHg.

Investigations:

serum sodium 142mmol/L (133-144)


serum potassium 6.4 mmol/L (3.3-5.3)
serum chloride 109 mmol/L (95-105)
serum bicarbonate 19 mmol/L (22-30)
serum creatinine 79 mol/L (60-120)
plasma renin activity 3.1 pmol/mL/h (3.0–4.3)
plasma aldosterone 250 pmol/L (330–830)

spot urinary sodium 75 mmol/L


spot urinary potassium 20 mmol/L
spot urinary chloride 98 mmol/L

After introduction of bendroflumethiazide the symptoms resolve and serum


potassium is 5.2 mmol/L (3.3-5.3).

Which is the most likely diagnosis?

A Gordon’s syndrome
B Liddle’s syndrome
C pseudohypoaldosteronism type 1
D primary adrenal insufficiency
E Type IV renal tubular acidosis

NOTES:

7
Question: 7

A 19 year old man with a family history of hypertension presented with a 3


week history of palpitations. Bendroflumethazide 2.5mg od had been
commenced 5 weeks earlier. On examination blood pressure was 170/106
mmHg and he had grade 2 hypertensive retinopathy.

Investigations:

serum sodium 140mmol/L (133-144)


serum potassium 2.2 mmol/L (3.3-5.3)
serum chloride 90 mmol/L (95-105)
serum bicarbonate 34 mmol/L (22-30)
serum creatinine 82 mol/L (60-120)
plasma renin activity 0.5 pmol/mL/h (3.0–4.3)
plasma aldosterone 750 pmol/L (330–830)
plasma 18-hydroxycortisol markedly above normal

ECG frequent premature ventricular


beats

What is the most appropriate treatment?

A amiloride
B dexamethasone
C dutasteride
D metyrapone
E spironolactone

NOTES:

8
Question: 8

A 65 year old man is admitted with non-specific abdominal pain, nausea, and
shortness of breath. He had type II diabetes mellitus and epilepsy. He had
been taking linezolid for 6 weeks for an infected knee prosthesis. He was also
taking topiramate 100mg bd, metformin 1000mg bd, and rosiglitazone 8mg
od. On examination his blood pressure was 136/82, and abdominal
examination was unremarkable.

Investigations:

serum sodium 138 mmol/L (133-144)


serum potassium 4.8 mmol/L (3.3-5.3)
serum chloride 101 mmol/L (95-105)
serum creatinine 80 mol/L (60-120)
plasma glucose 8.2 mmol/L (3.3-6.0)

Arterial blood gas


pH 7.35 (7.35-7.45)
pO2 14.1 kPa (9.3-13.3)
pCO2 3.5 kPa (4.7-6.0)
HCO3 15 mmol/L (22-26)
Base Excess -9.4 (-3 to +3)

What is the most likely cause of the acidaemia?

A diabetic ketoacidosis
B linezolid-induced lactic acidosis
C metformin-induced lactic acidosis
D ischaemic bowel
E topiramate-induced type III renal tubular acidosis

NOTES:

9
Question: 9

A 69-year-old man with refractory IgA lambda multiple myeloma started


treatment with lenalidomide 15 mg/day. He was also taking zoledronic acid
4mg IV every 4 weeks. He had stable chronic kidney disease (serum
creatinine 150 mol/L). Three weeks later, he presented with weakness,
fatigue, thirst, polyuria, and orthostatic dizziness.

On examination his blood pressure was 110/79 mmHg supine and 83/45
mmHg standing.

Investigations:

serum sodium 147mmol/L (133-144)


serum potassium 2.7 mmol/L (3.3-5.3)
serum chloride 120 mmol/L (95–107)
serum bicarbonate 15 mmol/L (22-30)
serum phosphate 0.25 mmol/L (0.8-1.4)
serum creatinine 239 mol/L (60-120)
serum urate 59 mol/L (260-500)
plasma glucose 4.9 mmol/L (3.3-6.0)
arterial blood pH 7.29 (7.35-7.45)

urinalysis pH 7.0
2+ glucose

Urine lambda light chains 5.76 mg/dL


(28.5 mg/dL pre-lenalidomide)

What is the most likely cause of these electrolyte abnormalites?

A AL amyloidosis
B bisphosphonate therapy
C intermediate cystinosis
D lenalidomide therapy
E light chain disease

NOTES:

10
Question: 10

A 68 year old afro-caribbean man presented with a 6 week history of mild


confusion and a single complex partial seizure 48 hrs earlier. He had a past
medical history of hypertension and diet-treated type II diabetes mellitus. He
was usually taking bendroflumethiazide 2.5mg od and atenolol 100mg od,
although had taken none for 5 days.
On examination blood pressure 146/92mmHg with no postural drop. He was
confused in time, place, and person.

Investigations:

serum sodium 115mmol/L (133-144)


serum potassium 3.8 mmol/L (3.3-5.3)
serum bicarbonate 26 mmol/L (22-30)
serum creatinine 56 mol/L (60-120)
serum urate 320 mol/L (260-500)
plasma glucose 25.5 mmol/L (3.3-6.0)
serum cholesterol 4.1 mmol/L (<5.2)
plasma TSH 4.9 mU/L (0.4–5.0)
random plasma cortisol 461 nmol/L (200–700)

spot urine sodium 61 mmol/L


spot urine osmolality 537 mosmol/kg

What is the most likely cause of the hyponatraemia?

A adrenal insufficiency
B bendroflumethiazide
C hypothyroidism
D syndrome of inappropriate ADH
E translocational hyponatraemia

NOTES:

11
Question: 11

A 67 year old man was admitted for a total knee replacement. He had
undergone renal transplantation 2 years earlier. His long-term medication
included tacrolimus 2mg bd, mycophenolate mofetil 500mg bd, prednisolone
5mg od, atenolol 50mg od, doxazosin 4mg od, and losartan 50mg od.
Six months earlier his serum sodium was 122 mmol/L (133-144) and serum
creatinine 212 mol/L (60-120).
The operation was uneventful but he was slow to mobilize.
On examination he was euvolaemic, blood pressure 139/70mmHg supine
and blood pressure 95/45mmHg standing.

Investigations:

haemoglobin 8.8 g/dL (13.0-18.0)


white cell count 8.2 x10^9/L (4.0-11.0)
serum sodium 124 mmol/L (133-144)
serum potassium 4.9 mmol/L (3.3-5.3)
serum chloride 94 mmol/L (95-105)
serum creatinine 205 mol/L (60-120)
plasma glucose 25.6 mmol/L (3.3-6.0)
plasma TSH 7.20 miu/L (0.30-5.00)
plasma free thyroxine 12 pmol/L (9.0-25.0)
(8am) plasma cortisol 657 nmol/L (138-690)
plasma prolactin 240 miu/L (50-400)
plasma osmolality 279 mosmol/kg (275 to 295)

spot urine sodium 47 mmol/L


spot urine osmolality 460 mosmol/kg

What is the most likely cause of the hyponatraemia?

A adrenal insufficiency
B volume depletion
C hypothyroidism
D syndrome of inappropriate ADH
E translocational hyponatraemia

NOTES:

12
Question: 12

A 75 year old man was admitted with a 5 day history of fever, productive
cough, and reduced oral intake.
On examination his temperature was 39.0°C, blood pressure 110/70 mmHg
supine and 84/60 mmHg standing. Crackles and reduced air entry were noted
over the right lung base.

Investigations:

serum sodium 140 mmol/L (133-144)


serum potassium 4.3 mmol/L (3.3-5.3)
serum chloride 102 mmol/L (95-105)
serum bicarbonate 22 mmol/L (22-30)
serum urea 10.2 mmol/L (2.5 to 6.5)
serum creatinine 106 mol/L (60-120)

Community acquired pneumonia was diagnosed and he was commenced on


intravenous amoxicillin 1g tds.

Investigations 3 days later:

serum sodium 138 mmol/L (133-144)


serum potassium 2.6 mmol/L (3.3-5.3)
serum chloride 90 mmol/L (95-105)
serum bicarbonate 34 mmol/L (22-30)

spot urine sodium 35 mmol/L


spot urine potassium 40 mmol/L
spot urine chloride <10 mmol/L
spot urine pH 5.5

What is the most likely cause of this acid-base disorder?

A administration of a loop diuretic


B Bartter’s syndrome
C effects of a nonreabsorbable anion
D post-hypercapnic metabolic alkalosis
E vomiting

NOTES:

13
Question: 13

A 55 year old black woman with a history of type II diabetes mellitus, and
morbid obesity, was admitted with an infected right knee prosthesis. The
prosthesis was removed but she required multiple surgical wound
debridements and prolonged parenteral antibiotics (levofloxacin and linezolid).
She was given co-proxamol for pain relief. Five weeks into the admission she
became acutely unwell and was transferred to the intensive care unit.

Investigations:

white cell count 30.6 x10^9/L (4.0-11.0)


serum sodium 145 mmol/L (133-144)
serum potassium 4.4 mmol/L (3.3-5.3)
serum chloride 102 mmol/L (95-105)
serum bicarbonate 8 mmol/L (22-30)
serum urea 5.4 mmol/L (2.5 to 6.5)
serum creatinine 194 mol/L (60-120)
plasma glucose 8.8 mmol/L (3.3-6.0)
plasma lactate 4.3 mmol/L (0.5-2.2)
plasma ketones negative

Arterial blood gas (60% oxygen)


pH 7.44 (7.35-7.45)
pO2 18.1 kPa (11.3–12.6)
pCO2 2.6 kPa (4.7–6.0)

She was commenced on tazocin, intravenous sodium bicarbonate was


administered, and her lactate fell to the normal range; however, a high anion
gap metabolic acidosis persisted.

What is the most likely cause of her persistent acid-base disturbance?

A acetaminophen toxicity
B linezolid toxicity
C Pseudomonas aeruginosa infection
D renal tubular acidosis
E rhabdomyolysis

NOTES:

14
Question: 14

A 55-year-old woman with distal renal tubular acidosis and type II diabetes
mellitus complained of polydipsia and polyuria. Her urine output was > 6 L per
day. She was taking metformin 1g twice a day.

On examination, her blood pressure was 128 / 68 mmHg lying and 126 / 72
standing.

Investigations:

serum sodium 135 mmol/L (137–144)


serum potassium 4.7 mmol/L (3.5–4.9)
serum creatinine 118 µmol/L (60–110)
estimated GFR (MDRD) 48 mL/min (>60)
fasting plasma glucose 8.2 mmol/L (3–6)
haemoglobin A1c 9.2 % (3.8–6.4)
serum corrected calcium 2.75 mmol/L (2.2–2.6)
plasma osmolality 282 mosmol/kg (278–300)
urinary osmolality 92 mosmol/kg (350–1000)

abdominal X-ray nephrocalcinosis.

A water deprivation test was performed. After 8 hours the following results
were obtained:
urine volume 980 ml
urine osmolality 760 mosmol/kg
plasma osmolality 288 mosmol/kg.

What is the most likely cause of the polyuria?

A central diabetes insipidus


B hypercalcaemia
C hyperglycaemia
D nephrogenic diabetes insipidus
E primary polydipsia

NOTES:

15
Question: 15

A 65-year-old man with long-standing stage 3 CKD due to diabetic


nephropathy developed significant hyperkalaemia. He also had a background
of chronic lymphocytic leukaemia and was due to start treatment with low-
dose chlorambucil. He was taking gliclazide 80mg bd, furosemide 80mg od,
lisinopril 2.5mg od, alfacalcidol 0.25μg od, adcal 2 tablets tds before food He
had also been taking calcium resonium 15g bd for 1 week..

On examination, his blood pressure was 138 / 68 mmHg and he was


euvolaemic.

Investigations:

serum sodium 135 mmol/L (137–144)


serum potassium 6.8 mmol/L (3.5–4.9)
serum creatinine 205 µmol/L (60–110)
estimated GFR (MDRD) 30 mL/min (>60)
fasting plasma glucose 8.2 mmol/L (3–6)
haemoglobin A1c 9.2 % (3.8–6.4)
haemoglobin 10.5 g/dL (13–18)
white cell count 154 109/L (4–11)
platelet count 125  109/L (150–400

ECG normal sinus rhythm, normal


morphology of QRS-
complexes and T-waves

What is the most appropriate next investigation?

A red cell membrane permeability studies


B renin –aldosterone ratio
C serum potassium on a promptly centrifuged sample
D short synacthen test
E simultaneous serum and plasma potassium concentration

NOTES:

16
APCKD, hereditary nephritis

& less common renal conditions

QUESTIONS

17
Question: 1

A 19-year-old man was referred for evaluation of renal impairment identified


during a recent admission for investigation of palpitations. Examination
demonstrated skin changes (Figures a & b) which had become more
extensive over the past 2 years. Blood pressure was 145/85 mmHg.

Investigations:

serum creatinine 131 µmol/L (60–110)


eGFR 65 ml/min
protein/creatinine ratio (untimed specimen) 137 mg/mmol (<3.5)

ultrasound normal sized kidneys


no obstruction

Which investigation is most likely to confirm the diagnosis?

A glucagon test
B peripheral leukocyte cystine level
C plasma alpha-galactosidase-A level
D serum galactose 1-phosphate level
E serum ceruloplasmin level

NOTES:

18
Question: 2

An 18-year-old man was referred for investigation of persistent invisible


haematuria His paternal uncle also had haematuria and deafness. His father
and mother were both apparently healthy. Examination was unremarkable.
His blood pressure was 121/66 mmHg.

Investigations:

serum creatinine 61 µmol/L (60–110)


eGFR >90 ml/min

urinalysis blood 2+

protein/creatinine ratio (untimed specimen) 7 mg/mmol (<3.5)

ultrasound normal sized kidneys


no obstruction
Renal biopsy:

Glomeruli appear normal by light microscopy. The glomerular basement


membrane uniformly measures <200 nM by electron microscopy.
Immunofluorescence reveals negative staining for Ig and C3 and there is
positive staining for the α3 and α5 collagen chains in the glomerular
basement membrane and tubular basement membrane.

What is the most likely diagnosis?

A autosomal recessive Alport syndrome


B brachio-oto-renal syndrome
C nail-patella syndrome
D thin membrane disease
E X-linked Alport syndrome

NOTES:

19
Question: 3

An 18-year-old man was referred for investigation of persistent invisible


haematuria. Two of his paternal uncles also had haematuria. His father and
mother were both apparently healthy. Examination was unremarkable. His
blood pressure was 121/70 mmHg.

Investigations:

serum creatinine 72 µmol/L (60–110)


eGFR >90 ml/min

urinalysis blood++

protein/creatinine ratio (untimed specimen) 5 mg/mmol (<3.5)

ultrasound normal sized kidneys


no obstruction

audiogram normal

Renal biopsy:

Glomeruli appear normal by light microscopy. The glomerular basement


membrane is irregularly thinned (<200 nM) by electron microscopy.
Immunofluorescence reveals negative staining for Ig and C3 and absent
staining for the α3 and α5 chains in the glomerular basement membrane
and tubular basement membrane

Which statement best describes the prognosis for this patient?

A he is at risk for later development of proteinuria and renal failure


B in the absence of a hearing abnormality, renal failure is uncommon
C it is unlikely that he will ever develop progressive renal failure
D the absence of hypertension indicates a favourable prognosis
E the absence of proteinuria indicates a favourable prognosis

NOTES:

20
Question: 4

A 28-year-old man was referred for investigation of hypertension and renal


impairment. He had a 15 year history of well controlled epilepsy. He was
taking sodium valproate and levitiracetam. Examination revealed skin
changes (a) and nail changes (b). His blood pressure was 155/105 mmHg.

Investigations:

serum creatinine 141 µmol/L (60–110)


eGFR 55 ml/min

urinalysis no blood or protein

protein/creatinine ratio (untimed specimen) 7 mg/mmol (<3.5)

ultrasound hyperechoic kidneys with


several small cysts
a b

Which form of surveillance should be commenced?

A echocardiography
B head MRI
C renal ultrasound scan
D urinary acetyl phenylcitrate
E urinary catecholamines

NOTES:

21
Question: 5

A 38-year-old man was referred for evaluation of renal impairment. He had


suffered with gout for the past 10 years. His maternal grandfather had died of
kidney failure and his mother started dialysis when she was 56 years old.
Examination was unremarkable. His blood pressure was 143/82 mmHg.

Investigations:

serum potassium 2.7 mmol/L (3.5–4.9)


serum creatinine 140 µmol/L (60–110)
eGFR 52 ml/min
serum urate 0.60 mmol/L (0.23–0.46)

urinalysis trace blood 2+ protein

protein/creatinine ratio (untimed specimen) 46 mg/mmol (<3.5)

ultrasound normal sized kidneys with


increased echogenicity and a
single 2cm cyst in the right kidney

MRI abdomen multiple small cysts ranging from 3


mm to 2 cm in diameter in the
corticomedullary junction

What is the most likely diagnosis?

A autosomal dominant polycystic kidney disease


B autosomal recessive polycystic kidney disease
C medullary cystic kidney disease
D nephronophthisis
E tuberose sclerosis

NOTES:

22
Question: 6

A 36-year-old male haemodialysis patient was admitted with generalised


weakness, worsening dyspnoea and peripheral paraesthesiae which had
failed to respond to alterations in his dialysis prescription. He had a history of
nephrolithiasis and recurrent urinary tract infections since childhood. On
examination, his blood pressure was 155/85 mmHg. Auscultation of the chest
revealed bilateral pleural effusions, heart sounds were normal and there was
pitting oedema in both ankles. There was widespread muscle wasting with
hyporeflexia and he had an ataxic gait.

Investigations:

echocardiogram left ventricular ejection fraction of 30%


multiple areas of increased echogenicity

electrophysiological studies atypical demyelinating neuropathy

What is the most likely diagnosis?

A cystinosis
B Dent’s disease
C Fabry disease
D familial amyloidosis
E primary hyperoxaluria

NOTES:

23
Question: 7

A 21 year old man was found to have a murmur on routine medical


examination. He had no significant family history. On clinical examination his
blood pressure was 140/90 mmHg. No family history was available as he was
adopted. An ultrasound scan revealed multiple cysts in both kidneys.

What is the most likely cause of his heart murmur?

A atrial septal defect


B aortic stenosis
C hypertrophic obstructive cardiomyopathy
D mitral valve prolapse
E ventricular septal defect

NOTES:

24
Question: 8

A 25 year old man with Alport’s was seen in the renal outpatient clinic. He had
mild sensorineural hearing loss, lenticonus and well controlled hypertension
on ACE inhibitor. One year ago, as a part of research study, he underwent,
genetic testing and was found to have a large deletion mutation of the COL4
A5 gene on the X chromosome. He asked about prognosis and future
treatment options.

Investigations:

serum creatinine 126 µmol/L (60 – 110)

urinary protein excretion 1 g/day (<0.2)

What is the most likely clinical course that he will follow?

A renal failure likely to progress and renal transplantation is the best way
forward.
B renal failure likely to progress and renal transplantation may be
considered, but risk of post transplant glomerulonephritis is moderate at
about 10-25%
C renal failure likely to progress and transplantation is associated with >50%
recurrence of post transplant GN
D renal failure likely to progress and transplantation is contraindicated
because of risk of pulmonary haemorrhage.
E renal failure unlikely to progress and renal transplantation therefore not
required

NOTES:

25
Question: 9

A 25 year old man presented with right sided abdominal pain and visible
haematuria. On examination blood pressure was 167/91 mmHg and bilateral
renal masses were palpable.

Investigations:

serum creatinine 326 µmol/L (60 – 110)

renal ultrasound scan bilateral enlarged kidneys containing


multiple cysts

Which of the following best describes the patho-physiology of this condition?

A by the time end stage renal failure is reached the majority of nephrons will
be affected by cystic change
B cysts develop in all regions of the nephron
C cystic fluid contains a high chloride concentration as a result of cGMP
driven activity of the cystic fibrosis transmembrane regulator (CFTR)
D Pkd2 mutation is more common than Pkd1
E polycystin-2 is a large membrane associated protein that is involved in
membrane calcium conductance

NOTES:

26
Question: 10

A 42 year old man with recently diagnosed adult polycystic kidney disease
attended for routine outpatient review. On examination his blood pressure
was 156/89 mmHg and he was clinically euvolaemic.

Investigations:

eGFR >90 ml/min

protein/creatinine ratio (untimed specimen) 56 mg/mmol (<3.5)

Which of the following is the most appropriate next step in his management?

A early use of V-2 receptor antagonists to delay progression of CKD


B high fluid intake to suppress cyst formation through reducing ADH
C maximise renin-angiotensin system blockade to reduce proteinuria
D start HMG Co-A reductase inhibitors as they improve renal blood flow
E titrate antihypertensives to a target blood pressure of 130/80 mm/Hg

NOTES:

27
Question: 11

A 35 year old man presented with a 6 month history of intermittent abdominal


pain. He had a five year history of gout. His maternal aunt had been on
dialysis for 15 years before dying of a myocardial infarction. His mother had
died in a road traffic accident when he was 3 years old. No other members of
his family were known to have renal disease. On examination his blood
pressure was 159/93 mmHg and he was clinically euvolaemic.

Investigations:

eGFR 53ml/min

renal ultrasound scan bilateral medullary cysts

Which one of the following is most likely to be found?

A a low spot urinary sodium


B a mutation in the Pkd2 gene
C an increased fractional urate excretion
D defective uromodulin secretion by the loop of Henle
E excessive Tamm Horsfal containing protein casts on urine microscopy

NOTES:

28
Question: 12

A 27 year old man presented with a 6 month history of polyuria, polydipsia


and worsening shortness of breath on exertion for the past 3 weeks. He had
also experienced pain in both feet but could not remember injurying them
recently. On examination blood pressure was 165/83 mmHg, there were
bibasal crepitations on auscultation of the chest and a number of abdominal
telangectasia.

Investigations:

serum creatinine 270 μmol/l (60 – 110)

protein/creatinine ratio (untimed specimen) 320 mg/mmol (<3.5)

A renal biopsy was performed.

Which of the following is most likely to be found on electron microscopy?

A mesangial electron dense deposits


B multilamellar myelin bodies in podocytes
C subepithelial electron dense deposits
D tubular reticular inclusion bodies
E widespread foot process effacement

NOTES:

29
Question: 13

A 24 year old woman presented with haematemesis. She had no past medical
history of note and consumed no alcohol. On examination her blood pressure
was 116/78 mmHg, heart rate 101 beats/minute and abdominal examination
revealed masses in both loins and right hypochodrium.

Investigations:

serum creatinine 170 μmol/l (60 – 110)

abdominal ultrasound scan enlarged liver with a dilated


biliary tree, reversal of normal
portal blood flow, bilaterally
enlarged echogenic kidneys
and splenomegaly

upper GI endoscopy oesophageal varices

What is the most likely diagnosis?

A autosomal dominant polycystic kidney disease


B autosomal recessive polycystic kidney disease
C glomerulocystic disease
D nephronopthisis
E orofacialdigital syndrome, type 1

NOTES:

30
Question: 14

An 22-year-old man was referred for investigation of persistent invisible


haematuria. Two of his uncles also had haematuria. His father and mother
were both apparently healthy. Examination was unremarkable. His blood
pressure was 121/70 mmHg.

Investigations:

serum creatinine 62 µmol/L (60–110)


eGFR >90 ml/min

urinalysis blood++

protein/creatinine ratio (untimed specimen) 5 mg/mmol (<3.5)

You suspect he has Alport’s syndrome.

Which of the following is most likely to support this diagnosis?

A absence of collagen type V staining on renal biopsy


B conductive hearing loss
C corneal crystals on slit lamp examination
D episodic visible haematuria following mucosal infection
E hypoplastic patellae

NOTES:

31
Renal vasculitis

SLE, diabetic nephropathy

interstitial nephritis & chronic GN

QUESTIONS

32
Question: 1

A 36 year old woman with systemic lupus erythematosus underwent


treatment for Class V lupus nephritis using oral prednisone and pulsed
intravenous cyclophosphamide. On starting treatment her 24 hour urinary
protein was 10.4g/24 hours (<0.2).

Three months into the course of therapy her 24 hour urinary protein was
5.2g/24 hours.

She attended the renal outpatient department five months into treatment
complaining of abdominal pain and lethargy.

Investigations:

haemoglobin 10.7g/dL (11.5–16.5)


MCV 95fL (80–96)
serum albumin 18g/L (37–49)
24 hour urinary total protein 9.4g (<0.2)
serum C-reactive protein 7mg/L (<10)

What is the most appropriate initial course of action?

A extend the course of cyclophosphamide for three further monthly pulses


B convert from cyclophosphamide to ciclosporin
C convert from cyclophosphamide to rituximab
D radiologically assess renal vasculature
E repeat renal biopsy

NOTES:

33
Question: 2

A 37 year old African woman presented to her local accident and emergency
department with a five week history of marked ankle swelling.

24 hour urinary total protein 6g/24 hours (<0.2)


serum albumin 26g/L (37–49)
serum creatinine 286 µmol/L (60–110)
antinuclear antibodies 1:80 (negative at 1:20)
anti-double-stranded DNA (ELISA) Negative
anti-neutrophil cytoplasmic antibodies Negative
cryolglobulins Negative
Hepatitis B antibody Negative
Hepatitis C antibody Positive
HIV test Positive
CD4 count 390 106/L (430–1690)

Renal biopsy showed mesangial hypercellularity with areas of focal and


segmetal glomerulosclerosis. There were extensive immune deposits of IgG,
IgM, C1q and C3. There were widely scattered electron dense deposits in the
subendothelial space.

What is the most likely diagnosis?

A diffuse lupus nephritis not related to HIV


B HIV associated nephropathy
C immune-complex HIV-associated glomerlonephritis
D membranoproliferative disease related to hepatitis C
E minimal change disease

NOTES:

34
Question: 3

A 58 year old lady with acute renal failure was transferred from a district
general hospital to her regional renal department for renal biopsy and further
management. Her investigation results were as follows:

antinuclear antibodies Negative


anti-double-stranded DNA (ELISA) Negative
anti-neutrophil cytoplasmic antibodies Positive

The renal biopsy showed active necrotising glomerulonephritis with crescents


but no significant chronic change.

What would offer the strongest argument for plasma exchange in addition to
standard therapy?

A age under 60 years


B concurrent positive anti-GBM antibody and dialysis dependent at
presentation
C concurrent positive anti-GBM antibody and dialysis independent at
presentation
D dialysis dependent at presentation irrespective of immunology results
E serum creatinine under 600 at presentation

NOTES:

35
Question: 4

A 64 year old woman saw her GP with fresh rectal bleeding and constitutional
symptoms of malaise, lethargy, myalgia and weight loss of 10 kilograms. 9
years ago she had a right hemicolectomy to remove an adenocarcinoma and
4 years ago suffered an intra-cerebral haemorrhage.

She was referred for urgent colonoscopy but before this could happen she
was admitted to hospital with worsening symptoms and confusion.

On examination she had normal heart sounds, bibasal crepitaitons, her


jugular venous pressure was raised 3cm, blood pressure was 195/114 and
temperature was 37.7ºC. There was a nodular rash overlying her shins.
Other than a laparotomy scar, abdominal examination was normal.

Investigations:

haemoglobin 9.7 g/dL (11.5–16.5)


platelet count 598 109/L (150–400)
serum urea 49 mmol/L (2.5–7.0)
serum creatinine 970 µmol/L (60–110)
serum C-reactive protein 132 mg/L (<10)
antinuclear antibodies Negative
anti-glomerular basement membrane Negative
antibody
Urine dipstick Blood ++ Protein ++

Which test is most likely to yield the diagnosis?

A angiography
B anti-neutrophil cytoplasmic antibodies
C colonoscopy
D renal biopsy
E skin biopsy

NOTES:

36
Question: 5

A 26 year old lady was admitted to hospital feeling generally unwell with
headache, blurred vision and a reduction in the amount of urine passed. She
was normally fit and well, running over 10 miles each week. She has not
been in contact with secondary care previously, having failed to attend a
dermatology appointment several months before.

On examination, she had thickened skin from her hands to above her elbows
with two peri-ungual ulcers on her right hand. Her blood pressure was 153/96
mmHg.

Investigations:

haemoglobin 9.2 g/dL (11.5–16.5)


platelet count 74 109/L (150–400)
serum urea 28 mmol/L (2.5–7.0)
serum creatinine 328 µmol/L (60–110)
serum total bilirubin 56 µmol/L (1–22)
Urine dipstick Blood - protein +

What is the most beneficial treatment?

A angiotensin converting enzyme inhibitor


B glucocorticoids
C haemodialysis
D plasma exchange
E supportive care

NOTES:

37
Question: 6

A 32 year old African man presented to the general medical on call with
weight loss, cough, dyspnoea and erythema nodosum on his lower limbs.
Chest imaging was compatible with sarcoidosis and a lymph node biopsy
provided confirmation.

Investigations:

haemoglobin 12.3 g/dL (11.5–16.5)


serum corrected calcium 2.9mmol/L (2.2–2.6)
serum urea 14.1 mmol/L (2.5–7.0)
serum creatinine 140 µmol/L (60–110)
serum albumin 47g/L (37–49)
Urine dipstick protein negative; blood negative

Renal tract ultrasound showed normal sized kidneys (left 11.2cm, right
11.3cm) with normal cortices and no evidence of obstruction.

What is the most likely cause of his renal impairment?

A cellular crescents
B glomerulosclerosis
C granulomatous involvement
D nephrocalcinosis
E nephrolithiasis

NOTES:

38
Question: 7

In a generalised ANCA positive vasculitis, which if the following statements


regarding pulsed vs. oral cyclophosphamide is not true?

A. both treatments should be given concurrently with glucocorticoids


B. oral treatment has a higher incidence of leucopoenia
C. pulsed iv allows a lower cumulative cyclophosphamide dose
D. pulsed iv treatment is less effective at inducing remission
E. remission is achieved in the same time frame with both treatments

NOTES:

39
Question: 8

In ANCA positive vasculitis, in which of the following situations is plasma


exchange not felt to confer benefit?

A. acutely dialysis dependent at presentation


B. concurrent anti-GBM positivity
C. remaining dialysis dependent despite greater than two weeks treatment
with cyclophosphamide
D. serum creatinine 500 at presentation
E. serum creatinine 600 at presentation

NOTES:

40
Question: 9

A 54 year old man presented with a palpable purpuric rash predominantly


over his legs and marked dependent oedema. Further examination revealed
2cm hepatomegly.

Investigations:

serum creatinine 255 μmol/L (60–110)


urine protein/creatinine ratio 421 mg/mmol (<15)
serum albumin 21 g/dL g/L (37–49)
HbA1C 5.8%

ANCA negative
Anti-GBM negative
dsDNA negative
C3 Normal
C4 low
Pre-dialysis screen HIV ab neg, Hep B ab neg, Hep C ab positive

What is the best management?

A. conservative management
B. chlorambucil and glucocorticoids
C. pegylated interferon 
D. pegylated interferon  and ribavirin
E. rituximab

NOTES:

41
Question: 10

A 64 year old man (smoker) treated with ANCA positive vasculitis was treated
on two occasions with cyclophosphamide. The first treatment was 5 years
ago, the second 3 years ago. He had a 40 pack year smoking history. He was
seen in clinic following an episode of gross haematuria. His medication at the
time was prednisone, azathioprine, ramipril, aspirin and atorvastatin.

Investigations:

Today One Three Six Nine Twelve


month months months months months
previously previously previously previously previously
Hb 11.9 11.7 12.1 12.2 11.9 12.0
Ur 11.1 10.2 11.1 10.5 11.8 10.2
Cr 169 163 172 170 168 171
INR 1.0 - - - - -
ANCA - - Neg - - Neg
Urine ++ + + Neg Neg Neg
blood
Urine + Neg Neg Neg Neg Neg
protein

MSSU <10 WCC. No growth.

What is the most important investigation?

A. ANCA including MPO/PR3 titre


B. cystoscopy
C. renal biopsy
D. repeat urine culture
E. urine cytology

NOTES:

42
Question: 11

A 36 year old lady with biopsy proven idiopathic membranous nephropathy


was reviewed in clinic. Her blood pressure was 122/74mmHg, and her
medication was aspirin 75mg and ramipril 10mg per day each.

Investigations:

Today Two Four Six months Eight


months months previously months
previously previously previously
(diagnosis)
Urea 6.1 6.3 6.2 7.1 6.3
Creatinine 76 82 78 81 79
eGFR >60 >60 >60 >60 >60
24 hour 5.8g 5.4g 6.0g 6.4g 7.5g
urinary
protein
Creatinine 88 89 87 89 86
clearane
Albumin 30 29 28 28 25
Potassium 5.8 5.7 5.8 5.2 4.5

What is the most appropriate treatment?

A. addition of irbsartan
B. addition of warfarin
C. alternating monthly oral steroids and chlorambucil
D. alternating monthly oral steroids and cyclophosphamide
E. continuation of current therapy

NOTES:

43
Question: 12

A 53 year old man with Wegener’s Granulomatosis treated with pulsed


intravenous cyclophosphamide two years ago was seen in clinic. Following
treatment he became ANCA negative and had been maintained initially on
steroid and azathioprine and subsequently on azathrioprine 75mg alone. His
last clinic letter noted that although he has no symptoms, he had become
ANCA positive again. No titre was available.

What is the most appropriate course of action?

A. increase azathioprine to 150mg


B. mycofenolate mofetil
C. no changes to medication, more frequent clinic follow up
D. repeat ANCA test with titre, MPO, PR3 levels prior to treatment change
E. recommence oral steroids

NOTES:

44
Question: 13

A 32 year old woman presented with gross peripheral oedema. She


complained of general malaise and arthralgia for which she had been taking
ibuprofen 400mg BD for one week.

Investigations:

serum creatinine 107 μmol/L (60-110)


serum cholesterol 9.3 mmol/L (<5.2)
serum albumin 21 g/dL g/L (37–49)
ANCA negative
ANA positive
dsDNA negative
C3 / C4 normal
HbA1C 5.9%

urinalysis protein ++++, blood +


urine protein/creatinine ratio 620 mg/mmol (<2.5)

A renal biopsy was reported as showing “Fourteen glomeruli one globally


sclerosed. Diffuse thickening of the glomerular basement membrane on light
microscopy present in all thirteen remaining glomeruli. Immunoflourescnace
positive for IgG, C3, C1q. Subepithelial deposits seen on electron microscopy
and also tubuloreticular structures seen within endothelial cells.”

What is the most likely diagnosis?

A. class V lupus nephritis


B. focal and segmental glomerulosclerosis
C. HIV related immune complex nephritis
D. idiopathic membranous nephropathy
E. minimal change disease

NOTES:

45
Question: 14

A 28 year old African lady with systemic lupus erythematosus for several
years was referred by the rheumatology team as she was found to have
proteinuria on dipstick testing and a blood pressure of 154/88mmHg.

Investigations:

24 hour urinary total protein 5.9g (<0.2)

A subsequent renal biopsy was consistent with class III A/C lupus nephritis
with most glomeruli on the sample involved. Her medication at the time were
ramipril and sertraline.

What is the best treatment for her?

A corticosteroids
B corticosteroids and irbesartan
C corticosteroids and mycophenolate mofetil
D corticosteroids and pulsed iv cyclophosphamide
E irbesartan

NOTES:

46
Haematuria, proteinuria,

the nephrotic syndrome

& renal disease in pregnancy

QUESTIONS

47
Question: 1

A 38 yr old male was referred to the renal outpatient clinic by his general
practitioner having been found to have haematuria during routine tests for life
insurance. He did not smoke and had no history of recent respiratory tract
infections or gross haematuria. He had a family history of hypertension and
physical examination was normal and blood pressure was 139/86 mmHg. His
only medication was ramipril 10mg once daily.

Investigations:

serum sodium 144 mmol/L (137–144)


serum potassium 4.2 mmol/L (3.5–4.9)
serum urea 5.4 mmol/L (2.5–7.0)
serum creatinine 102 µmol/L (60–110)

white cell count 6.8 109/L (4–11)


haemoglobin 11.2 g/dL (13–18)
platelet count 302  109/L (150–400)

24-h urinary total protein 1.7g (<0.2)

A comprehensive immunology screen was negative and an ultrasound scan


of his renal tract did not reveal any abnormalities. The patient refused to have
a renal biopsy.

Which of the following is the best approach to management?

A addition of ARB
B aspirin and omacor therapy
C observe
D pulsed methylprednisolone
E tonsillectomy

NOTES:

48
Question: 2

A 22 year old man was found to have microscopic haematuria on urinalysis


during a routine medical before joining the armed forces. His physical
examination, including blood pressure, was normal. He was referred to the
renal outpatient department for further investigation. He had an active lifestyle
and exercised every day. There was no family history of kidney problems or
deafness. He denied any urinary tract symptoms, loin pain, fever, or episodes
of frank haematuria.

Investigations:

serum sodium 137 mmol/L (137–144)


serum potassium 4.6 mmol/L (3.5–4.9)
serum urea 5.1 mmol/L (2.5–7.0)
serum creatinine 94 µmol/L (60–110)
24-h urinary total protein 0.2 g (<0.2)

Which investigation would you undertake next?

A abdominal ultrasound
B audiogram
C autoantibody screen
D renal biopsy
E repeat Repeat urinalysis and urine microscopy

NOTES:

49
Question: 3

A 26 year old woman was referred to the outpatient department with a recent
discovery of persistent microscopic haematuria. She was otherwise in good
health and took no prescription or over-the-counter medications. She had not
noticed any episodes of macroscopic haematuria, dysuria, frequency of
micturition, back pain or fever. Her physical examination was normal with a
blood pressure of 110/60 mmHg. Her younger sister was evaluated several
years ago for a similar finding but no cause was found and she did not have a
renal biopsy.

Investigations:

urine microscopy 15 - 20 dysmorphic red cells


no casts
<10 white blood cells

serum sodium 139 mmol/L (137–144)


serum potassium 4.4 mmol/L (3.5–4.9)
serum urea 5.0 mmol/L (2.5–7.0)
serum creatinine 88 µmol/L (60–110)
24-h urinary total protein 0.1 g (<0.2)
serum albumin 39 g/L (37–49)
serum C3 109 mg/dL (65–190)

What is the most likely pathogenesis of her haematuria?

A an inherited disorder involving the α1/α2 chains of Type IV collagen


B an inherited disorder involving the α3/α4 chains of Type IV collagen
C an inherited disorder involving the α5/α6 chains of Type IV collagen
D formation of immune complexes containing under-galactosylated IgA1
E in situ formation of immune complexes on the sub-epithelial side of the
glomerular basement membrane

NOTES:

50
Question: 4

A 37 yr old woman presented to the renal new patient clinic with a decline in
renal function. She was well with no symptoms or medical history. She took
occasional ibuprofen for menstrual cramps and an over the counter herbal
therapy for weight loss. 6 months ago, during a routine examination, she had
a serum creatinine of 89 µmol/L (60–110). On physical examination she had a
body mass index of 32 and blood pressure of 144/82 mmHg.

Investigations:

urine microscopy 8 - 10 red blood cells


4 - 6 white blood cells
no casts

serum sodium 143 mmol/L (137–144)


serum potassium 4.9 mmol/L (3.5–4.9)
serum urea 14.0 mmol/L (2.5–7.0)
serum creatinine 289 µmol/L (60–110)
serum albumin 41 g/L (37–49)

24-h urinary total protein 0.3 g (<0.2)

Her serum immunoglobulin profile was normal and an ultrasound scan of the
renal tract showed normal sized kidneys with increased echogenicity. She
subsequently underwent renal biopsy.

What is the biopsy most likely to show?

A hypocellular interstitial fibrosis and tubular atrophy


B interstitial non-caseating granulomas
C lymphocytic interstitial inflammation and tubular atrophy
D secondary FSGS
E small vessel vasculitis

NOTES:

51
Question: 5

A 30 year old woman developed weight loss, low grade fever, eye pain with
visual blurring.

Investigations:

serum creatinine 132 µmol/L (60–110)


24-h urinary total protein 0.3 g (<0.2)

anti-nuclear antibody 1:160


serum C3 109 mg/dL (65–190)

haemoglobin 10.1 g/dL (11.5–16.5)


MCV 86 fL (80–96)

Urine dipstick leucocytes 2+,pr++

What is the most likely diagnosis?

A lupus nephritis
B microscopic polyangiitis
C mixed connective tissue disease
D sarcoidosis
E tubulointerstitial nephritis with uveitis (TINU) syndrome

NOTES:

52
Question: 6

A 21 year old man presented with dipstick proteinuria, identified during part of
a routine insurance screen. He was asymptomatic.

Which of the following is not a recognized cause of proteinuria?

A cardiac failure
B exercise
C high protein diet
D obesity
E orthostasis

NOTES:

53
Question: 7

A 56 year old woman presented with nephrotic syndrome. Renal biopsy


findings including electron microscopy were consistent with minimal change
nephrotic syndrome.

Which of the following is least likely to be associated with minimal change


nephrotic syndrome?

A gold therapy
B graft versus host disease
C heavy metal ingestion
D Hodgkin’s disease
E non steroidal anti-inflammatory (NSAIDs) use

NOTES:

54
Question: 8

A 52 year old man presented to the renal clinic with a 4 month history of
tiredness, leg swelling and weight gain. Dipstick urinalysis revealed 4+ protein
and 2+ blood. His blood pressure was 170/92 mmHg.

Investigations:

haemoglobin 12.1g/L (13 - 18)


serum sodium 132 mmol/L (137 - 144)
serum potassium 4.1 mmol/L (3.5 – 4.9)
serum urea 8.1 mmol/L (2.5 - 7)
serum creatinine 113 mmol/L (60 - 110)
serum albumin 24g/L (37 - 49)
serum cholesterol 8.0 mmol/L (5.2)
urine protein:creatinine ratio 680mg/mmol (<2.5)

He underwent an urgent kidney biopsy. The light microscopy findings were as


follows (see figures):

He was treated for a six month period with a combination of maximum


recommended doses ACEi and Angiotensin II blocker. Repeat tests after that
period showed that neither his proteinuria nor his renal function had improved.

What is the most appropriate management strategy for this patient?

A continue treatment with ACEi +/- angiotension II blocker


B steroids and azathioprine
C steroids and cyclophosphamide / chlorambucil combination therapy
D steroids and MMF combination therapy
E steroid monotherapy

NOTES:

55
Question: 9

A 45 year old Asian woman underwent a renal biopsy for nephrotic range
proteinuria. She had a past medical history of Hepatitis C infection. She had
no evidence of cirrhosis. She had previously been an intravenous recreational
drug user more than 5 years ago.

What will be the most likely finding on renal biopsy?

A IgA Nephropathy
B focal and segmental glomerulosclerosis
C minimal change disease
D membranous nephropathy
E membranoproliferative GN

NOTES:

56
Question: 10

A 65 year old woman with a history of multiple myeloma was admitted with
hypercalcemia.

Investigations:

serum calcium 3.4 mmol/L (2.2–2.6)


serum creatinine 110 µmol/L (60–110)
urine protein excretion 0.5g/day (<0.2)

She was treated with intravenous fluids and intravenous pamidronate.


Because of recurrent hypercalcemia she was then advised to have monthly
pamidronate. She was reviewed 3 months later in outpatients.

Investigations:

serum calcium 3.0 mmol/L (2.2–2.6)


serum creatinine 143 µmol/L (60–110)
urine protein excretion 1.3 g/day (<0.2)

In view of persistent hypercalcemia the dose of monthly pamidronate was


increased to 120mg. Her results a year after this treatment were as follows:

serum calcium 2.69 mmol/L (2.2–2.6)


serum creatinine 256 µmol/L (60–110)
urine protein excretion 6.0 g/day (<0.2)

What would be the most likely finding on renal biopsy?

A acute interstitial nephritis


B cast nephropathy
C focal and segmental glomerulosclerosis
D IgA nephropathy
E membranous GN

NOTES:

57
Question: 11

A 56 year old man with type 2 diabetes mellitus was referred to the renal
department with chronic kidney disease. His blood pressure was 170/98
mmHg. He had background retinopathy and was taking ramipril 10mg and
amlodipine 10mg daily.

Investigations:

serum creatinine 170µmol/L (60-110)


urine protein excretion 3 g/day (<0.2)

A diagnosis of diabetic nephropathy was made. Irbesartan 75mg and


furosemide 40mg were added to his regimen and the dose of irbesartan was
gradually increased to 300mg per day.

Three months later, he returned to clinic with a blood pressure of 116/72


mmHg. He has no oedema on clinical examination.

Investigations:

haemoglobin A1c 6.3% (3.8–6.4)


serum creatinine 65 µmol/L (60 – 110)
serum potassium 5.8 µmol/L (3.5-4.9)
urine protein excretion 2 g/day (<0.2)

What would be the most appropriate next step in his management?

A add doxazosin
B add spironolactone
C change amlodipine to diltiazem
D maintain current regimen
E recommend low protein diet

NOTES:

58
Question: 12

A 26 year old primagravida was found to be hypertensive at a routine check


by her midwife at 36 weeks gestation. Previous blood pressure and urine
results had been normal. The following morning she became acutely unwell
with visual obscurations and abdominal pain, and CTG suggested fetal
distress. Her blood pressure was 163/101.

urinalysis showed protein ++, blood -, leucocyte esterase -, nitrites -.


urinary protein:creatinine ratio 421mg/mmol (<15)

An emergency caesarean section was performed and a 2.8kg male infant was
delivered in good health. Estimated blood loss was 300mL. Urine output was
10mL/hour. Physical examination revealed lower limb oedema, sacral
oedema and the jugular venous pressure was raised 2 cm.

Investigations post-operatively revealed:

serum sodium 131 mmol/L (137–144)


serum potassium 4.7 mmol/L (3.5–4.9)
serum creatinine 255 μmol/L (60–110)
serum urate 490 mmol/L (0.19–0.36)
serum alanine aminotransferase 45 U/L U/L (5–35)
serum lactate dehydrogenase 150 U/L (10–250)
haemoglobin 97 g/L (115–165)
platelet count 109 x109/dL (150–400)
blood film microcytosis

What is the most appropriate management?

A fluid challenge with 1000ml 0.9% saline stat


B intravenous frusemide 40mg
C magnesium sulphate infusion
D plasma exchange
E transfuse 2 units of packed red cells

NOTES:

59
Question: 13

A 30 year old woman was found to have asymptomatic bacteriuria 18 weeks


into her second pregnancy, growing coliform bacilli sensitive to trimethoprim,
ciprofloxacin and cefalexin in a midstream sample of urine. She had no
history of renal disease.

Investigations:

serum creatinine 42 μmol/l (60–110)

Which of the following is not applicable?

A a first episode of uncomplicated asymptomatic bacteruria in pregnancy


should be treated with antibiotics
B antibiotic prophylaxis is indicated during pregnancy if two or more
episodes of bacteriuria occur
C asymptomatic bacteriuria is associated with preterm delivery
D asymptomatic bacteriuria should be monitored with follow up urine culture
and treatment instituted if symptoms develop
E bacteriuria is more common during pregnancy

NOTES:

60
Question: 14

A 19 year old woman with reflux nephropathy presented at 16 weeks


gestation. Vesicoureteric reflux disease had been diagnosed when she was 8
years old and imaging had shown unilateral upper pole scarring. She had no
history of hypertension and was on no medication.

Investigations:

serum creatinine 75μmol/L (60-110)

Which of the following statements is false?

A her child is at risk of inheriting reflux disease and should be offered


screening in infancy
B her risk of developing hypertension during pregnancy or pre-eclampsia is
increased
C monthly urine culture to screen for asymptomatic bacteruria is advisable
D pregnancy is likely to be associated with significant (>20%) loss of renal
function post-partum
E the likelihood of a successful pregnancy is similar to that of the general
population

NOTES:

61
Question: 15

A 37 year old woman presented 3 years ago with nephrotic syndrome and
was diagnosed with primary focal and segmental glomerulosclerosis.
Following 2 years of prednisolone and 18 months of ciclosporin therapy, the
nephrotic state was in partial remission. 12 months later she was off
immunosuppressive therapy, being on treatment with ramipril 10mg, losartan
50mg, atorvastatin 20mg and bendroflumethiazide 2.5mg, each once daily.
Her blood pressure was 144/82 mmHg.

Investigations:

serum creatinine 102 μmol/l (60-110)


urine protein/creatinine ratio 174 mg/mmol (<15)

She wished to start a family. Which of the following management approaches


is most appropriate?

A advise to avoid pregnancy


B discontinue current treatment and optimise control of blood pressure with
methyldopa and nifedipine before conception
C elective placement of a tunnelled dialysis catheter given the risk of
requiring dialysis during pregnancy
D recommence prednisolone at 10mg od to prevent nephrotic relapse during
pregnancy
E stop all medication as soon as conception is confirmed

NOTES:

62
Question: 16

A 36 year old woman at 13 weeks’ gestation complained to her midwife of


swollen legs. Her blood pressure was 142/91 mmHg. Examination revealed
pitting oedema to her knees.

Investigations:

serum sodium 132 μmol/L (137–144)


serum creatinine 48 μmol/L (60-110)
serum albumin 27 g/dL g/L (37–49)
serum alkaline phosphatase (over 14 yr) 95 U/L (45–105)
serum cholesterol 9.3 mmol/L (<5.2)

dipstick urinalysis protein +++, blood -, leu -, nit–

urine protein/creatinine ratio 936 mg/mmol (<2.5)

What is the most likely diagnosis?

A class 5 lupus nephritis


B haemolytic uraemic syndrome
C inferior vena cava thrombosis
D minimal change nephropathy
E pre-eclampsia

NOTES:

63
Question: 17

Which of the following is least likely to be present in normal pregnancy?

A 50% increase in GFR


B decreased serum osmolality
C doubling of urine protein excretion
D metabolic alkalosis
E unilateral pelvicalyceal dilatation

NOTES:

64
Question: 18

A 35 year old woman with IgA nephropathy was seen at her first transplant
annual review after receiving a pre-emptive 1-0-0 mismatch kidney from her
sister. She had delayed graft function but no episodes of rejection and no
urinary tract infection.

serum creatinine 110 μmol/l (60-110)

Renal biopsy (following standard protocol) at 10 months showed no evidence


of recurrent disease or chronic allograft nephropathy. Her blood pressure was
137/83 mmHg whilst taking lisinopril 10mg and amlodipine 5mg per day. Her
other medication was tacrolimus 4mg twice a day (with satisfactory trough
serum drug concentration), mycophenolate mofetil 1g twice a day and
prednisolone 5mg once a day.
During the consultation she stated that she may be pregnant. A test
performed in clinic was positive and an ultrasound scan confirmed a viable 6
week fetus.

What changes should be made to her regimen?

A continue amlodipine, mycophenolate and prednisolone. Stop tacrolimus


and lisinopril. Start azathioprine.
B continue amlodipine, tacrolimus and mycophenolate. Stop lisinopril and
prednisolone. Start bendroflumethiazide.
C continue amlodipine, tacrolimus and prednisolone. Stop lisinopril and
mycophenolate. Start azathioprine and aspirin.
D continue all current treatment and arrange emergency obstetrics
appointment.
E increase prednisolone to 10mg od. Stop lisinopril, amlodipine, tacrolimus
and mycophenolate. Start aspirin and sirolimus.

NOTES:

65
Question: 19

A 24 year old woman with congenital renal hypoplasia on maintenance


haemodialysis for two years becomes amenorrhoeic. Pregnancy is confirmed
at 12 weeks gestation.

What is the most appropriate management?

A arrange termination of pregnancy


B change dialysis sessions to 2 hours 5 times a week to minimise
cardiovascular instability
C change to peritoneal dialysis to facilitate a continuous therapy.
D increase dialysis frequency to 6 or 7 times a week, aiming for >20 hours of
dialysis per week
E stop erythropoietin and offer transfusion to maintain haemoglobin > 11g/dl

NOTES:

66
Question: 20

A 27 year old woman presented with increased pedal and periorbital swelling
for 3 weeks. Prior to this she had lethargy, generalised arthralgia and noted a
pale rash over her shins. She had taken occasional ibuprofen and
paracetamol for the last month.

Investigations:

creatinine 135μmol/l (60-110)


eGFR 44ml/min (>60)
albumin 27g/l (37-49)
serum C3 55mg/dl (65 -190 mg/dl)
serum C4 9mg/dl (15-50 mg/dl)
antinuclear antibody negative
anti-double-stranded DNA antibody negative
extractable nuclear antigen antibody negative

urine dipstick: blood +, protein +++

urine protein:creatinine ratio 760 mg/mmol creat (<30)

What is the most likely diagnosis?

A acute interstitial nephritis


B focal segmental glomerulosclerosis
C lupus nephritis class IV
D membranous nephropathy
E mesangiocapillary glomerulonephritis type I

NOTES:

67
Question: 21

A 66 year old man with type 2 diabetes, proteinuria, ischaemic heart disease
and hypertension was referred to the renal clinic due to poor blood pressure
control and declining renal function. Medication included aspirin, gliclazide,
ramipril and bendroflumethiazide. Background diabetic retinopathy had been
diagnosed 6 months earlier and glycaemic control remained poor. On
examination, blood pressure was 176/88 mmHg, he had pedal oedema and
was obese.

Investigations:

creatinine 173μmol/l (60 – 110)


cholesterol 6.2mmol/l (<5.2)
glycated haemoglobin 9.2% (3.8-6.4)

urine dipstick: blood trace, protein ++

renal ultrasound: left kidney 9.0cm


right kidney 7.9cm
no hydronephrosis

He was established on insulin, simvastatin and amlodipine. Renal function


was stable over the following 12 months before presenting to the emergency
department with an 8 hour history of left flank pain, dyspnoea, reduced urine
output and visible haematuria. Blood pressure was 212/99.

Investigations:

creatinine 654μmol/l (60 – 110)

urine dipstick: blood ++++, protein +++

KUB X-ray: no radio-opaque calculi seen

Which investigation is least appropriate?

A CT renal colic
B electrocardiogram
C immunology screen
D renal arteriogram
E urine culture

NOTES:

68
Question: 22

An 18 year old female student was found to have proteinuria on routine


urinalysis when starting university. Repeat testing one week later again
showed isolated proteinuria. Blood pressure was 118/72 mmHg and there
was no peripheral oedema.

Investigations:

creatinine 101μmol/l (60 – 110)


eGFR 88ml/min (>60)

First pass urine samples (x2):


urine protein:creatinine ratio 6 mg/mmol creatinine
urine protein:creatinine ratio 9 mg/mmol creatinine

What is the next appropriate step?

A 24 hour urine protein measurement


B no action required
C pregnancy test
D renal ultrasound and biopsy
E urine for Bence Jones protein

NOTES:

69
Question: 23

A 27 year old primigravida presented at 36 weeks’ gestation with mild


headache and increased pedal oedema. Past medical history and early
pregnancy progress were unremarkable with normal blood pressure and
urinalysis at booking. Blood pressure was 162/92mmHg.

Investigations:

haemoglobin 9.9g/dl (11.5 – 16.5)


platelets 130 x109/l (150 – 400)
creatinine 55μmol/l (40 - 90)
urate 450μmol/l (190 – 360)
ALT 35 IU/l (5 – 35)

urinalysis protein +++

urine protein:creatinine ratio 101 mg/mmol creat (<30)

Over the next 48 hours, blood pressure failed to improve despite


antihypertensives. Ultrasonography revealed intrauterine growth retardation.
An emergency caesarean section was performed.

Eighteen hours after delivery she remained hypertensive with oliguria. On


examination she was pale and tachypnoeic. Blood pressure was
166/94mmHg and she had pitting oedema to mid shin. Chest examination
revealed a few basal crepitations. Urine output was 10 to 15ml/hour.

Investigations:

haemoglobin 7.9g/dl (11.5 – 16.5)


platelets 80 x109/l (150 – 400)
creatinine 144μmol/l (40 - 90)
urate 500μmol/l (190 – 360)
ALT 105 IU/l (5 – 35)
LDH 80 U/l (10 – 250)
bilirubin 18μmol/l (1 – 22)
albumin 29 g/l (37 – 54)

What is the next appropriate step in her management?

A central venous pressure monitoring


B intravenous fluid challenge
C intravenous furosemide
D plasma exchange
E platelets

NOTES:

70
Question: 24

Which of the following statements about nephrotic syndrome and pregnancy


is true?

A most cases of nephrotic syndrome in pregnancy are secondary to pre-


eclampsia
B renal biopsy is contraindicated in pregnancy
C corticosteroids should not be used to treat nephrotic syndrome in
pregnancy
D relapses of steroid-responsive minimal change nephropathy in pregnancy
do not affect fetal outcomes
E thromboprophylaxis in pregnancy patients with nephrotic syndrome should
be commenced in the puerperium

NOTES:

71
QUESTION: 25

A 37 yr old Caucasian woman was diagnosed with nephrotic syndrome. A


subsequent renal biopsy revealed FSGS. Her estimated GFR using the
MDRD formula was 68ml/min/1.73m2.

Which one of the following best predicts her risk for progression to end-stage
renal disease?

A ACE genotyping for polymorphisms


B a family history of hypertension
C the extent of tubulointerstitial disease on biopsy
D the level of plasma renin activity
E the percentage of glomeruli with focal changes on biopsy

NOTES:

72
Urinary tract infection

stones

& obstruction

QUESTIONS

73
Question: 1

A 24-year old man was referred for management of recurrent renal stone
disease. He was taking no regular medications. Examination was normal. His
blood pressure was 123/65 mmHg.

Investigations:

serum creatinine 115 µmol/L (60–110)


eGFR 88 ml/min
serum corrected calcium 2.2 mmol/L (2.2–2.6)
serum phosphate 1.0 mmol/L (0.8–1.4)

24-h urinary calcium 5.0 mmol (2.5–7.5)


24-h urinary urate 2.0 mmol (<3.6)
24-h urinary oxalate 1.3 mmol (0.15-0.45)
24-h cystine 10 µmol (<100)

Urine microscopy of fresh urine:

What treatment would be most appropriate to prevent further stone formation?

A allopurinol
B furosemide
C indapamide
D penicillamine
E pyroxidine

NOTES:

74
Question: 2

A 47-year-old man was referred with a 12 month history of recurrent renal


colic and passage of multiple renal calculi. Stone analysis had identified
calcium oxalate. 24 months previously he had undergone a jejunoileal bypass
for management of obesity. He had been placed on an oxalate restricted diet
3 months previously.

Investigations:

serum creatinine 135 µmol/L (60–110)


eGFR 80ml/min
serum corrected calcium 2.2 mmol/L (2.2–2.6)
serum phosphate 1.0 mmol/L (0.8–1.4)

24-h urinary calcium 5.0 mmol (2.5–7.5)


24-h urinary oxalate 2.1 mmol (0.15-0.45)

Which of the following is most likely to reduce the risk of stone formation?

A hydrochlorothiazide
B increase in dietary calcium intake
C penicillamine
D pyridoxine
E surgical reversal of the jejunoileal bypass

NOTES:

75
Question: 3

A 25-year old man presented with macroscopic haematuria and renal colic.
CT KUB confirmed a stone in the distal right ureter. The stone was passed
spontaneously with complete resolution of symptoms.

Investigations:

serum creatinine 90 µmol/L (60–110)


eGFR 88ml/min
serum corrected calcium 2.2 mmol/L (2.2–2.6)
serum phosphate 1.0 mmol/L (0.8–1.4)

24-h urinary calcium 3.0 mmol (2.5–7.5)


24-h urinary urate 2.0 mmol (<3.6)
24-h urinary oxalate 0.20 mmol (0.15-0.45)
24-h urinary cystine 200 µmol (<100)

Which treatment is most likely to prevent further stone formation?

A allopurinol
B cranberry juice
C furosemide
D indapamide
E penicillamine

NOTES:

76
Question: 4

A 50-year-old man was referred with recurrent renal colic and passage of
renal calculi. He was taking no regular medication. Examination was
unremarkable. Blood pressure was 143/82 mmHg.

Investigations:

serum creatinine 90 µmol/L (60–110)


eGFR 88ml/min
serum phosphate 1.0 mmol/L (0.8–1.4)

24-h urinary calcium 10.5 mmol (2.5–7.5)


24-h urinary urate 2.0 mmol (<3.6)
24-h urinary oxalate 0.20 mmol (0.15-0.45)

Which treatment would be most appropriate to prevent further stone


formation?

A allopurinol
B furosemide
C indapamide
D penicillamine
E sodium bicarbonate

NOTES:

77
Question: 5

A 35-year old man with CKD4 secondary to vesicoureteric reflux attended


clinic seeking advice on the likelihood of any of his children inheriting
vesicoureteric reflux. He had 2 brothers and 2 sisters. Both of his brothers
had vesicoureteric reflux.

How likely are his children to inherit vesicoureteric reflux?

A all boys will be affected


B all girls will be carriers
C 50% of his children will be affected
D 25% of his children will be affected
E there is an increased (but not quantifiable) risk of his children being
affected

NOTES:

78
Question: 6

A 50-year old man presented with a 2 year history of non-specific bilateral loin
pain. He described intermittent nocturia and hesitancy. He was taking no
regular medications. Examination was unremarkable, there was no palpable
bladder and rectal examination was normal. Blood pressure was 149/87
mmHg.

Investigations:

serum creatinine 250 µmol/L (60–110)


eGFR 32 ml/min

urinalysis Blood 2+ Protein 2+


Leucocyte 1+
CT scan:

What would be the most appropriate next investigation?

A DMSA scan
B isotope bone scan
C KUB X-ray
D MAG-3 renogram
E renal ultrasound

NOTES:

79
Question: 7

A 54 year man with hypertension and type 2 diabetes mellitus presented with
right loin pain radiating to his groin. His medications included metformin,
gliclazide, ramipril and atrovastatin. On examination his blood pressure was
158/84 mmHg.

Investigations:

creatinine 85 umol/l (60–110)


corrected calcium 2.45 mmol/l (2.2–2.6)
uric acid 500 μmol/l (190 – 360)

urine dipstick: blood +++, urine pH 4.5

24-h urinary calcium 7mmol (<7.5 mmol in men)


24-h urinary oxalate 0.3 mmol (<0.36 mmol)
24-h urinary uric acid 7 mmol (< 4.7 mmol in men)
24-h urinary sodium 140mmol (<130 mmol)
24-h urinary creatinine 12000 μmol (>132 μmol/kg in men)

X-ray KUB unremarkable

CT renal colic 4 mm stone in right ureter

A diagnosis of a uric acid stone was made and he was commenced on


allopurinol and sodium bicarbonate.

Two years later he presented with further symptoms of left loin pain. Clinically
he appeared well hydrated and urine pH was 6.8.

What is the most likely composition of this stone?

A calcium oxalate
B calcium phosphate
C struvite
D uric acid
E xanthine

NOTES:

80
Question: 8

A 28 year old man was referred with recurrent bilateral urinary stones
identified on plain abdominal X- ray. He had no past medical of note. On
examination, blood pressure was 138/70 mmHg and there was tenderness in
the left loin.

Investigations:

creatinine 109 μmol/l (60–110)


corrected calcium 2.34 mmol/l (2.2–2.6)

urine dipstick: blood ++, protein trace

24-h urinary calcium 11mmol (<7.5 mmol in men)


24-h urinary oxalate 0.5 mmol (<0.36 mmol)
24-h urinary uric acid 5mmol (< 4.7 mmol in men)
24-h urinary citrate 12 mmol (>17 mmol)
24-h urinary sodium 140mmol (<130 mmol)
24-h urinary creatinine 8000 μmol (>132 μmol/kg men)

renal USS right kidney 11.2 cm


left kidney 12.0 cm
no scarring

He is advised to drink plenty of fluid, restrict dietary sodium intake and started
on indapamide. Despite this he represents with recurrent right sided loin pain.
On this occasion he managed to catch a stone and analysis identified calcium
oxalate.

Assuming he has been compliant with the above management plan, what
would be the next appropriate change in his management?

A bisphosphonate
B change indapamide to bendrofluazide
C encourage intake of animal protein
D potasium citrate
E sodium bicarbonate

NOTES:

81
Question: 9

A 54 year old man was referred with progressive chronic kidney disease, his
eGFR had fallen from 44 ml/min to 32 ml/min over the previous 12 months.
He had a 10 year history of Crohn’s disease and had required an ileal
resection 5 years previously after which he had developed diarrhoea due to
fat malabsorption. He was taking cholestyramine, mesalazine, loperamide,
adcal D3, folic acid and multivitamins. His diarrhoea was reasonably
controlled on these medications. On examination he was euvolaemic, blood
pressure was 112/54mmHg. Abdominal examination was unremarkable apart
from the laparotomy scar

Investigations:

urinalysis unremarkable

24-h urinary calcium 8mmol (<7.5 mmol in men)


24-h urinary oxalate 1.2 mmol (<0.36 mmol)
24-h urinary uric acid 5mmol (< 4.7 mmol in men)

X-ray KUB bilateral renal calculi

A diagnosis of enteric oxalate nephropathy was made.

Which statement regarding enteric oxalate nephropathy is incorrect?

A calcium supplements will reduce oxalate absorption


B enteric colonisation of Oxalobacter formigenes is uncommon
C potassium alkali should be given to increase urine pH
D pyridoxine reduces oxalate stone formation
E the risk of stone disease increases with increasing length of ileal resection

NOTES:

82
Question: 10

A 54 year old woman attended for outpatient review. She had CKD stage 3a
secondary to diabetes mellitus and recurrent urinary tract infections. She had
been free of infection for over 2 years and was not on prophylactic antibiotics.
On review she was well and denied any urinary symptoms. Blood pressure
was 134/80 mmHg.

Investigations:

glycated haemoglobin 6.5%

urine dipstick +2 blood, + 1 protein, + 2 leucocyte


esterase, +ve nitrites

What is the most appropriate next step?

A empirically treat with antibiotics


B send a urine culture, and treat if positive
C only treat if there are 2 positive urine cultures
D no further investigation necessary, continue routine follow-up
E start prophylactic antibiotics

NOTES:

83
Question:11

A 17 year old man attended the adult-paediatric transition clinic. He had been
diagnosed with grade III vesicoureteric reflux aged 12 months following
recurrent urinary tract infections. He was taking prophylactic antibiotics. He
had suffered infrequent urinary tract infections throught his childhood. Blood
pressure was 110/70 mmHg.

Investigations:

creatinine 60μmol/L (60–110)

renal USS scarred left kidney 8.4 cm


right kidney 10.4 cm with
normal outline

Which statement regarding vesicoureteric reflux is incorrect?

A 5 % of affected children will develop end stage renal disease


B prophylactic antibiotics in adults reduce the risk of progression of CKD
C there is no advantage of surgical over medical treatment in terms of future
risk of CKD, hypertension or renal scarring
D there is a reduced risk of further renal scarring in adults
E vesicoureteric reflux often spontaneously resolves as children enter
adulthood

NOTES:

84
Question: 12

A 54 year old South Asian woman presented with an 18 month history of


recurrent urgency, nocturia and dysuria despite repeated courses of
antibiotics. Repeated mid stream urine specimens had grown no organism
despite persistent pyuria. Her medication included prophylactic trimethoprim
and mebeverine for irritable bowel syndrome.

Investigations:

creatinine 90μmol/L (60–110)

early morning urine MSSU (x3) no mycobacteria grown

cystoscopy diffuse glomerulations and a


reduced bladder capacity of
150ml

Which of the following is not true of this condition?

A a Hunner ulcer is a recognised feature of this condition


B hydrodistension is a useful therapeutic option
C it is a form of reflex sympathetic dystrophy affecting the bladder
D symptoms can be relieved by anticholinergics and antispasmodics
E urinary diversion is required in many patients

NOTES:

85
Question: 13

A 56 year old man was admitted generally unwell with anorexia and vomiting.
He had undergone a left nephroureterocystectomy and ileal conduit for locally
invasive bladder carcinoma 2 years previously. Renal function 6 months
previously was stable with a creatinine of 130μmol/l (60–110).

Investigations:

serum creatinine 450μmol/L (60–110)

urinalysis +2 haematuria

renal USS single right kidney


no hydronephrosis

loop conduitogram unremarkable

His renal function continued to deteriorate despite intravenous fluids. He


maintained a good urine output (3 litre/24hour). A right-sided nephrostomy
was inserted which resulted in an immediate diuresis followed by an
improvement in renal function.

Which of the following statement concerning acute kidney injury due to urinary
tract obstruction is incorrect?

A an urgent nephrostomy is required if the patient is pyrexial


B diuretic renography/ MAG-3 is useful in establishing a diagnosis of partial
obstruction
C little or no recovery is expected after 2 weeks of continuous and complete
obstruction
D normal renal function can be restored if obstruction is relieved within 1
week
E polyuria and an absence of hydronephrosis is a feature of partial or
incomplete renal obstruction

NOTES:

86
Question: 14

A 35 year old woman under investigation for a presumed multisystem disorder


was referred by the rheumatologists for investigation of an abnormal urinary
sediment.

Investigations:

serum creatinine 85μmol/L (60–110)


corrected calcium 2.34 mmol/l (2.2–2.6)

urinalysis +2 haematuria, pH 8.0

X-ray KUB nephrocalcinosis

24-h urinary calcium 8mmol (<7.5 mmol in men)

Which of the following is true

A anti ro and anti la antibodies are associated with this presentation


B primary hyperparathyroidism is likely despite the normal serum calcium
C renal biopsy would show plugging by calcium oxalate deposits
D treatment is with urinary acidification to dissolve the calcium deposits
E type 2 (proximal) renal tubular acidosis is the likely diagnosis

NOTES:

87
Question: 15

Which of the following drugs can precipitate and form renal stones?

A acetozalamide
B indanivir
C probenecid
D theophylline
E vitamin C

NOTES:

88
Question: 16

A 16 year old girl was referred for investigation of intermittent visible


haematuria. On examination she appeared small for her age but there were
no other abnormal physical findings.

Investigations:

serum creatinine 90 µmol/L (60–110)


eGFR 88ml/min
serum corrected calcium 2.2 mmol/L (2.2–2.6)
serum phosphate 1.0 mmol/L (0.8–1.4)

X-ray KUB nephrocalcinosis

DMSA/MAG3 renogram split function R 51%, L 43%


normal handling of isotope

24-h urinary calcium 2.62 mmol (2.5–7.5)


24-h urinary oxalate 1.44 mmol (0.15-0.45)
24-h urinary phosphate 12.3 µmol (15-50)
24-h urinary L-glyceric acid elevated

Which of the following is true?

A liver-kidney transplantation is the treatment of choice


B progression to end stage renal disease is likely
C systemic oxalosis is likely to occur
D the metabolic defect results in conversion of glyoxalate to oxalate
E the enzyme defect is alanine:glyoxalate aminotransferase (AGT)

NOTES:

89
Dialysis

(general principles, clinical management & complications)

QUESTIONS

90
Question: 1

A 49 year old woman with polycystic kidney disease attended the outpatient
follow up clinic with a history of fatigue and ankle swelling. She had started on
CAPD 18 months before, but changed to APD four weeks later for lifestyle
reasons. Her APD prescription was 6 overnight dwells of 1.5litre using 1.36%
dextrose solution, and 700ml icodextran fluid during the day. Her only other
history was of hypertension which was well controlled with ramipril and
nifedipine. On examination she had pitting oedema to her knees, fine bibasal
crepitations and a small left-sided pleural effusion. Her abdomen was not
tender and she had brought a sample from her last overnight dwell which was
clear. She was 6kg over her target weight.

Results of an adequacy test taken 3 weeks before were as follows:

urine output 113 mL/24hr


ultrafiltration 746 mL/24hr
Kt/V 1.96
4 hour dialysate/plasma creatinine 0.90

What is the most likely cause for her becoming fluid overloaded?

A APD is not suitable for her transporter type


B loss of residual renal function
C membrane failure due to early peritoneal sclerosis
D reabsorbtion of fluid caused by use of dextrose-based dialysis fluid
E use of dihydropyridine calcium channel blocker

NOTES:

91
Question: 2

A 49 year old peritoneal dialysis patient had been on renal replacement


therapy since the age of nine. From then until twelve years ago he had
functioning renal grafts but had been on peritoneal dialysis since. However,
for the past eighteen months he had developed progressive cramps,
headaches and resistant, widespread oedema that had necessitated leave
from work. His only other history is of diet-controlled type 2 diabetes mellitus
for three years. The patient refuses haemodialysis. He is currently on CAPD
with 4 x 2 litre exchanges of 2.27% glucose fluid each day. His last PET /
adequacy test was as follows:

urine output 113 mL/24hr


ultrafiltration 446 mL/24hr
Kt/V 1.48
4 hour dialysate/plasma creatinine 0.90

What should be the first change to his management?

A add in high dose diuretics


B change to APD
C change to an amino acid based solution for all exchanges
D increase to 2.5 litre volume exchanges
E refer to a specialist renal counsellor

NOTES:

92
Question: 3

A 66 year old man on haemodialysis complained of progressive abdominal


pain and bloating for three months. He was otherwise well. His primary renal
illness was crescentic IgA nephropathy, diagnosed 8 years before. He was
started on peritoneal dialysis immediately and was only changed to
haemodialysis nine months ago following an episode of staph aureus
peritonitis. On examination he was afebrile with a tight, distended abdomen.

What finding is most likely to be seen on investigation?

A CT evidence of visceral peritoneal calcification


B ECG evidence of bradycardia
C evidence of small bowel obstruction on abdominal radiograph
D gram negative bacilli on effluent culture
E transudative peritoneal effluent

NOTES:

93
Question: 4

A 35 year old woman on CAPD attended the accident and emergency


department with generalised abdominal pain and cloudy dialysis fluid. She
was on day 5 of a course of oral flucloxacillin and rifampicin given for a
Staphylococcus aureus exit site infection. Her primary disease was unknown
and she had been on CAPD for the 3 months since her transplant failed due
to chronic allograph nephropathy. She was still on small doses of
prednisolone and mycophenolic acid to preserve residual renal function. On
examination, she was diffusely tender in the abdomen, with erythema tracking
from the tenckhoff catheter exit site towards the umbilicus. Her temperature
was 38.1°C, and blood pressure 105/69 mmHg. Treatment was started with
intraperitoneal vancomycin and gentamicin pending results of the dialysis fluid
culture. The following day the on call microbiologist telephoned the ward to
confirms the presence of Staphylococcus aureus in the culture.

What is the most important step in this patient’s management?

A add mupirocin ointment to clear the exit site infection


B laxative therapy to ensure full drainage of her fluid
C request surgical consult to remove the catheter
D stop all immunosuppressive therapy
E stop the intraperitoneal gentamicin

NOTES:

94
Question: 5

A 74 year old man presented was seen on the haemodialysis unit. He had
presented acutely with anti-GBM disease 4 weeks ago and was immediately
started on plasma exchange and haemodialysis via a tunnelled line. He had
made no clinical improvement and was told that morning that he was likely to
need dialysis in the long term. He wished to enquire about his long term
dialysis options as another patient told him that he could have dialysis at
home. His medical records indicated that he was anuric, being treated with
pulsed intravenous cyclophosphamide and oral steroids, had a body mass
index of 33 and an ileal conduit formed following resection of a bladder
malignancy. He lived alone in sheltered housing following the death of his wife
2 years before.

What clinical factor means this man cannot have peritoneal dialysis?

A anuria
B his age and social circumstances
C his weight
D the presence of an ileal conduit
E the use of long term immunosuppression

NOTES:

95
Question: 6

A 71 year old man presented to the accident and emergency department


feeling generally very unwell. He had hypertension and stage 4 chronic kidney
disease but had not been to clinic for 12 months. On examination he was
twitchy, his blood pressure was 166/88 mmHg, respiratory rate 40 per minute,
O2 saturations 100% on air, temperature 36.3°C and there were no localising
neurological signs or papilloedema. His serum biochemistry results were as
follows:

serum sodium 141 mmol/L (137–144)


serum potassium 5.6 mmol/L (3.5–4.9)
serum urea 71.2 mmol/L (2.5–7.0)
serum creatinine 733 µmol/L (60–110)
serum bicarbonate 9 mmol/L (20–28)

He is sent for urgent haemodialysis, 2.5 hours into which he rapidly became
unresponsive. Again, there were no localising neurological signs, though he
had developed papilloedema. His blood pressure was unchanged.

What should be the first intervention?

A arrange for a CT of the head to be undertaken after dialysis


B perform an ECG
C start an intravenous labetolol infusion
D stop the dialysis heparin
E stop the dialysis immediately

NOTES:

96
Question: 7

Which of the following components of a haemodialysis water system is most


important in ensuring that haemodialysis patients are not exposed to
microbiological or chemical contamination?

A good quality water from the water provider


B carbon filters
C ion exchange filters
D reverse osmosis
E effective disinfection of the haemodialysis monitor

NOTES:

97
Question: 8

Malnutrition occurs frequently in chronic haemodialysis patients and is


associated with adverse outcomes. The nutritional status of patients can be
monitored in a variety of ways. The protein catabolic rate is one measure
which has been used. This can be generated from formal urea kinetic
modelling.

Which one of the following input data is necessary for accurate measurement
of protein catabolic rate?

A assessment of dietary protein intake


B measurement of body composition by bioimpedance
C urea excretion by residual renal function
D composition of dialysate
E measurement of basal metabolic rate

NOTES:

98
Question: 9

A 66 year old woman has had haemodialysis at home for 10years and comes
in for centre dialysis during her partner’s illness. Her dialysis nurse measures
the flow in her longstanding forearm arteriovenous fistula using a Transonics
US monitor and informs you that the access flow is measured at 4500ml/min.
There are no previous measures with which to compare. The patient reports
no problems with the fistula and needling is not problematic. She normally
dialyses for 4hrs using a Fresenius FX100 dialyser, blood flow of 400ml/min
and the most recent URR is 78%.

What is the most appropriate first response to this information?

A ask surgeon to ligate the AVF urgently


B insert a tunnelled dialysis catheter to rest AVF
C examine AVF clinically and check position of needling
D organise urgent echocardiograph
E reduce pump speed on dialysis from 400 to 300ml/min

NOTES:

99
Question: 10

A 36 year old woman with reflux nephropathy on home haemodialysis for 19


years moved house. She had previously been using high flux haemodialysis
due to high β2 microglobulin levels. Her new hospital dialysis training unit
used haemodiafiltration machines.

As proven by meta-analysis of controlled clinical trials which of the following


has been proven to be better with HDF compared to high flux HD?

A better anaemia control


B less intra-dialysis hypotension
C lower all cause mortality
D lower β2M levels
E none of the above

NOTES:

100
Question: 11

A 66 year old woman attended the home haemodialysis outpatient clinic


suffering three months of progressive discomfort in her thumbs, index and
ring fingers. The discomfort occurred most often at night. She also
complained of shoulder pains that she blamed on arthritis. She had been a
home therapy patient since developing IgA nephropathy aged 45, and did not
wish to be considered for transplantation. Clinical examination revealed
normal capillary refill and warm digits on both hands. There was some
weakness of palmar abduction of the thumb and early distal joint
osteoarthritis. There were no other findings of note.

What measure would be of most benefit in this setting?

A change to a high flux membrane


B arrange ligation of her fistula and insertion of tunnelled dialysis catheter
C change to haemodiafiltration
D persuade her to accept son’s offer to donate kidney
E switch to daily nocturnal haemodialysis

NOTES:

101
Question: 12

A 19 year old woman was seen on a haemodialysis unit ward round because
her urea reduction ratio was 62%. She had chronic pyelonephritis and was
dialysing vain a forearm arteriovenous loop graft. Clinical examination
revealed a healthy-looking graft with well spaced needling areas. She was not
overloaded and her blood pressure was well controlled. Her last dialysis
prescription commented that the venous pressure was high and had been so
for 2 weeks. The sheet also gives the following information:

HD membrane cellulosic; surface area 1.4m2


prescription 4 hours, 3 times per week
arterial pressure within normal range
blood flow rate 290 ml/min
needle size 16 gauge

What measure is most important in her management?

A change to a membrane with greater surface area


B change to a more biocompatible dialysis membrane
C increase her dialysis hours
D refer her for venography +/- angioplasty
E use smaller bore needles

NOTES:

102
Question: 13

A 39 year old man on CAPD attended a drop in clinic complaining of low


grade abdominal pain for three days. He was systemically well but had a
temperature of 38.1°C. His dialysis fluid was drained and the effluent was
cloudy. A sample was sent for microscopy and culture and the patient was
started on gentamicin and vancomycin IP. The results showed a high
leucocyte count with a predominance of neutrophils. Gram stain and aerobic /
anaerobic cultures were negative. The patient’s brother had been treated for
tuberculosis two years before and so a smear for acid-fast bacilli was taken.
This was also negative. Two days later the patient was reviewed and was
clinically unchanged.

What is the next step in his management?

A culture of the effluent for tuberculosis


B immediate catheter removal
C plain abdominal radiograph
D start anti-tuberculous therapy
E start intravenous gentamicin and vancomycin

NOTES:

103
Question: 14

A 66 year old woman presented to hospital with anuria and fatigue. She was
diagnosed with cANCA associated vasculitis and was immediately started on
plasma exchange and haemodialysis via a tunnelled internal jugular catheter.
She had a history of DVT 6 years ago and suffered an MI with successful
primary PCI six months before.

Following discussions with her renal team, the patient opted for long term
haemodialysis and asked for a PTFE graft rather than an arterio-venous
fistula.

What would be the greatest benefit of this type of vascular access?

A a significantly shorter maturation period


B greater rate of success in cases of percutaneous “de-clotting”
C less likelihood of steal
D lower rate of thrombosis
E lower long-term risk of blood-borne infection

NOTES:

104
Question: 15

A 49 year old peritoneal dialysis patient was failed to attend a day case
appointment for formation of an arterio-venous fistula. He had underlying
reflux nephropathy and had been on renal replacement therapy since the age
of nine. From then until twelve years ago he had functioning renal grafts but
had been on peritoneal dialysis since. He had persistently refused
haemodialysis for lifestyle issues. However, for the past eighteen months he
had developed progressive cramps, headaches and resistant, widespread
oedema that had necessitated leave from work. His only other history is of
diet-controlled type 2 diabetes mellitus for three years. The patient cannot be
contacted and he is subsequently found dead at home by the police.

What is the most likely cause of death?

A acute myocardial infarction


B epilepsy
C hypertensive CVA
D hypoglycaemia
E sudden death due to arrhythmia or pump failure

NOTES:

105
Question: 16

Cardiac deaths account for 41% of deaths for dialysis patients in the USA
(USRDS data). Of these more than half are coded as ‘sudden cardiac death
(SCD)’ presumed due to arrhythmia or sudden pump failure.

For HD patients, sudden cardiac death occurs more commonly at certain


times in the dialysis schedule. Compared to the mid week scheduled day off
dialysis, SCD is more common:-

A during dialysis
B within a few hours of dialysis
C in the 24hours prior to the next dialysis session
D immediately before dialysis on the longest inter-dialytic interval
E all of these times

NOTES:

106
AKI, acute renal replacement therapy

& plasmapharesis

QUESTIONS

107
Question: 1

A 70 year old man with a history of hypertension, type 2 diabetes mellitus and
congestive cardiac failure underwent coronary artery bypass grafting. Post-
operatively, his pulse was 85 beats per minute regular, blood pressure -
125/75 mmHg, central venous pressure 10 cm H2O and urine output 20
mL/h. His pre-operative serum creatinine had been 106 μmol/L.

Investigations (post-operatively):

serum sodium 136 mmol/L (137–144)


serum potassium 4.2 mmol/L (3.5–4.9)
serum creatinine 264 μmol/L (60–110)

He was started on a dopamine infusion (2µg/kg/min [low-dose]).

Which statement about dopamine is correct?

A decreases the duration of dialysis dependence


B increases oxygen tension in the inner medulla of the kidney
C increases renovascular resistance
D reduces in-hospital mortality
E reduces peak serum creatinine

NOTES:

108
Question: 2

A 65 year old man with chronic kidney disease, coronary artery disease, and
peripheral vascular disease underwent a surgical abdominal aortic aneurysm
repair. His pre-operative serum creatinine was 156 μmol/L (60-110). On the
first post-operative day his pulse was 96 beats/min, blood pressure 120/80
mmHg, central venous pressure 10 cm H2O and urine output 15 mL/h.

Investigations:

serum sodium 134 mmol/L (137–144)


serum potassium 4.0 mmol/L (3.5–4.9)
serum creatinine 355 μmol/L (60–110)

The surgical team started him on furosemide 40 mg/h to maintain his urine
output at 40 mL/h.

Which statement about this management is correct?

A furosemide aids renal recovery by increasing urine output


B furosemide decreases the duration of renal replacement therapy should
the patient require it
C furosemide improves medullary oxygenation and decreases severity of
acute kidney injury
D furosemide increases the risk for ototoxicity
E furosemide reduces in-hospital mortality

NOTES:

109
Question: 3

A 28 year woman with HIV and normal renal function started HAART
treatment comprising efavirenz, tenofovir, and lamivudine. She was also
started on co-trimoxazole for prophylaxis against pneumocystis. She was
reviewed 2 months later.

Investigations:

serum sodium 135 mmol/L (137–144)


serum potassium 5.2 mmol/L (3.5–4.9)
serum creatinine 230 μmol/L (60–110)
urinalysis trace protein, trace blood
MSU WCC <10, RCC <10, hyaline
casts and no growth

What is the most likely cause of her acute kidney injury?

A acute interstitial nephritis due to co-trimoxazole


B efavirenz nephrotoxicity
C HIV associated nephropathy
D lamivudine nephrotoxicity
E tenofovir nephrotoxicity

NOTES:

110
Question: 4

A 55 year old woman with type 2 diabetes mellitus, coronary artery disease
and chronic kidney disease was electively admitted for coronary angiography.

Investigations:

serum sodium 138 mmol/L (137–144)


serum potassium 4.5 mmol/L (3.5–4.9)
serum creatinine 165 μmol/L (60–110)
serum corrected calcium 2.42 mmol/L (2.2–2.6)
serum bicarbonate 24 mmol/L (20–28)

Which treatment will provide the greatest reduction in her risk for contrast-
induced nephropathy?

A 600 mg bd oral N-acetyl cysteine from 1 day pre- to 1 day post-procedure


B 5% dextrose in water at 1mL/kg/hr for 12 hours pre and post procedure
C 0.9% saline at 3mL/kg/h for 1 hour pre-procedure and 1mL/kg/h for 3
hours post-procedure
D isotonic sodium bicarbonate at 3 mL/kg/h for 1 hour pre-procedure and
1mL/kg/h for 6 hours post-procedure
E Pre-hydration with isotonic crystalloid; no clear benefit of bicarbonate over
saline.

NOTES:

111
Question: 5

A 48 year old man with end-stage liver disease secondary to hepatitis C


developed hepatorenal syndrome whilst on the liver transplant waiting list.

Investigations:

serum sodium 125 mmol/L (137–144)


serum potassium 3.5 mmol/L (3.5–4.9)
serum urea 9.7 mmol/L (2.5–7.0)
serum creatinine 268 μmol/L (60–110)
serum albumin 25 g/L (37–49)
urinary sodium concentration <10 mmol/L

What is the most appropriate management?

A administration of 75g of hyperoncotic (25%) albumin


B a vasopressin analogue (e.g. terlipressin) should be started
C he should be considered for a combined liver and kidney transplant
D he should be removed from the liver transplant waiting list
E renal replacement therapy should not be considered

NOTES:

112
Question: 6

A 25 year old fire-fighter was taken to hospital after spending 5 hours trapped
under a collapsed building. His blood pressure was 110/75 mmHg, heart rate
84 beats per minute and urine output 30 mL/h.

Investigations:

serum sodium 140 mmol/L (137–144)


serum potassium 4.3 mmol/L (3.5–4.9)
serum creatinine 121 μmol/L (60–110)
creatine kinase 16500 U/L (24–195)

Which treatment will provide the greatest benefit in preventing acute kidney
injury?

A intravenous dopamine infusion at a rate of 1.5μg/kg/min


B intravenous half-isotonic sodium bicarbonate plus 10% mannitol at a rate
of 125 mL/h
C intravenous isotonic saline initially at a rate of 1 L/h
D oral N-acetylcysteine plus half-isotonic saline at a rate of 1mL/kg/h
E prophylactic continuous venovenous haemofiltration

NOTES:

113
Question: 7

A 52 year old man presented with a community-acquired pneumonia. He had


been previously fit and healthy with no past medical history. He was admitted
to the intensive care unit and given inotropic and ventilatory support. He was
oliguric with a central venous pressure of 12 cm H2O.

Investigations:

serum sodium 135 mmol/L (137–144)


serum potassium 5.6 mmol/L (3.5–4.9)
serum creatinine 368 µmol/L (60–110)
serum bicarbonate 14 mmol/L (20-28)

He was started on continuous venovenous haemofiltration.

What is his prognosis?

A if he survives for 6 months after hospital discharge, the likelihood of his


health-status being good is approximately 75%
B if he survives to hospital discharge, his likelihood of requiring long-term
dialysis is approximately 40%
C if he survives to hospital discharge, his likelihood of residual renal
impairment is approximately 70%
D if he survives to hospital discharge, his mortality at 6 months is
approximately 50%
E likelihood of surviving to hospital discharge is approximately 15%

NOTES:

114
Question: 8

A 65 year old woman was admitted with a community-acquired pneumonia.


Her pulse was 120 beats per minute, blood pressure 80/60 mmHg, respiratory
rate 36 per minute, and oxygen saturation (by pulse oximetry) 90% on FiO2
0.85. She was taken to the intensive care unit where she was mechanically
ventilated for respiratory failure and started on a noradrenaline infusion.

Investigations:

serum sodium 130 mmol/L (137–144)


serum potassium 6.2 mmol/L (3.5–4.9)
serum creatinine 398 μmol/L (60-110)

Her urine output was 10mL/min.

Which renal replacement therapy modality is associated with the highest


probability of 28 day survival?

A acute peritoneal dialysis


B continuous venovenous haemofiltration
C intermittent haemodialysis
D no single modality is associated with an increased probability of survival at
28 days, compared with other modalities of renal replacement.
E sustained low-efficiency dialysis

NOTES:

115
Question: 9

A 64 year old woman was referred to the nephrologists after she was noted to
be oliguric following an elective coronary angiography. She had a past
medical history of hypertension, for which she was taking ramipril. On
examination she was euvolaemic and had mottled skin changes on her legs
and back.

serum sodium 134 mmol/L (137–144)


serum potassium 5.2 mmol/L (3.5–4.9)
serum urea 33.3 mmol/L (2.5–7.0)
serum creatinine 356 µmol/L (60–110)
creatine kinase 560 U/L (24–170)
haemoglobin 11.2 (11.5–16.5)
white cell count 13.6  109/L (4–11)
neutrophil count 9.1 109/L (1.5–7.0)
lymphocyte count 2.5 109/L (1.5–4.0)
eosinophil count 1.2 109/L (0.04–0.40)

What is the most likely diagnosis?

A acute interstitial nephritis


B cholesterol emoboli
C contrast nephropathy
D rhabdomyolysis
E systemic vasculitis

NOTES:

116
Question: 10

A 50 year old woman with high grade non-Hodgkin lymphoma developed


acute kidney injury following her first course of chemotherapy. On
examination she had generalized lymphadenopathy, hepatomegaly and
splenomegaly.

Investigations:

serum sodium 135 mmol/L (137–144)


serum potassium 5.8 mmol/L (3.5–4.9)
serum urea 32.3 mmol/L (2.5–7.0)
serum creatinine 450 µmol/L (60–110)

What investigation is most likely to give a diagnosis?

A creatinine kinase
B KUB X ray
C lactate dehydrogenase
D renal angiography
E uric acid

NOTES:

117
Question: 11

A 65-year-old man was admitted following a traumatic hip fracture and


required total hip replacement. He was known to have CKD 3 secondary to
hypertension (on treatment) but was otherwise fit and well. During surgery he
had a significant amount of bleeding and post-operative blood pressure on the
ward remained <95 mmHg systolic for 4 hours. On examination his blood
pressure was 85/45 mmHg, heart rate 110 beats/min and there were bibasal
crepitations on auscultation of the chest.

Investigations:

haemoglobin 10.4 g/dL (11.5-16.5)


serum sodium 135 mmol/L (133–144)
serum potassium 6.4 mmol/L (3.3–5.3)
serum urea 27.3 mmol/L (2.5-6.5)
serum creatinine 700 μmol/L (60–120)

Regarding haemodialysis in this patient, which of the following statements is


correct?

A haemodialysis performed daily is superior to less intensive regimens


B intensive haemodialysis regimes reduce the length of time haemodialysis
is required
C mortality at 60 days is approximately 50%
D prior kidney disease does not influence the probability of recovery of renal
function
E thrice weekly haemodialysis is inadequate therapy

NOTES:

118
Question: 12

A 55-year-old man presented with cardiac chest pain. He had stable CKD 3
secondary to diabetic nephropathy. His regular medications comprised
metformin, gliclazide, aspirin, ramipril and simvastatin. On examination blood
pressure was 130/68 mmHg, heart rate 88 beats/min and he was clinically
euvolaemic.

Investigations:

troponin-I 0.2 μg/L (<0.04)

ECG infero-lateral ischaemia

A coronary angiogram was scheduled for evaluation of his chest pain.

Which of the following interventions is most likely to reduce the risk of contrast
induced nephropathy?

A four hours of HD using a high flux membrane immediately post procedure


B use of high osmolar contrast media
C use of prophylactic oral N-acetylcysteine
D volume expansion using intravenous normal saline solution
E volume expansion using intravenous sodium bicarbonate solution

NOTES:

119
Question: 13

A 67-year-old woman presented following a mechanical fall at home. She had


sustained extensive soft tissue injuries and had been lying on the floor for
several hours before she was found. She had no history of renal disease. On
examination her blood pressure was 130/79 mmHg, heart rate 98 beats/min
and chest was clear to auscultation.

Investigations:

serum urea 14.5 mmol/L (2.5-6.5)


serum creatinine 264 μmol/L (60–120)
serum creatinine kinase 23,000 U/L (25-170)

Which one of the following treatments is most likely to reduce the risk of
further acute kidney injury?

A intravenous furosemide
B intravenous mannitol
C intravenous normal saline
D intravenous sodium bicarbonate
E prophylactic haemofiltration

NOTES:

120
Question: 14

A 59-year-old man underwent emergency surgery for a ruptured abdominal


aortic aneurysm. During the operation the aorta was cross-clamped above the
renal arteries for 20 minutes. Two days later, he became oligo-anuric.

Investigations:

Pre-op
serum creatinine 79 μmol/L (60–120)

48 hours post-op
serum creatinine 570 μmol/L (60–120)
serum bicarbonate 17 mmol/L (22-26)

He was commenced on renal replacement therapy.

Which of the following is associated with the best outcome with regard to
mortality and renal function?

A alternate day intermittent haemodialysis


B continuous veno-venous haemofiltration (CVVH)
C daily intermittent haemodialysis
D rates of survival and renal recovery are similar with all modalities of RRT
E sustained low efficiency daily dialysis (SLEDD)

NOTES:

121
Question: 15

A 32-year-old man presented feeling non-specifically unwell with fatigue,


nausea and anorexia. One week previously he had injured his knee and been
prescribed regular paracetamol and diclofenac. He was taking no other
medications. On examination his blood pressure was 125/75 mmHg.

Investigations:

serum eosinophil level 5.3 x109/L (0.04-0.4)


serum sodium 138 mmol/L (133–144)
serum potassium 5.5 mmol/L (3.3–5.3)
serum creatinine 464 μmol/L (60–120)

urine dipstick 2+ protein

urine protein:creatinine 396 (<30)

What would be the most likely light microscopy findings on renal biopsy?

A extensive interstitial lymphocytic infiltrate and oedema with normal


glomeruli
B flattening of the tubular epithelium and loss of the brush border with mild
interstitial oedema
C intimal fibrosis, medial hypertrophy and hyaline deposits of the arteriole
D mesangial cell proliferation with mesangial matrix expansion
E nodular sclerosis with increased mesangial matrix and cellularity

NOTES:

122
Question: 16

A 40-year-old man presented with acute confusion and slurred speech after
consuming a bottle of antifreeze. He had a long history of depression and had
attempted to take his life in the past. On examination his blood pressure was
110/78 mmHg, heart rate 115 beats/min, GCS 12/15. Chest was clear to
auscultation and heart sounds normal.

Investigations:

serum creatinine 175 μmol/L (60–120)

arterial blood gases on 40% oxygen


pH 7.15 (7.35-7.45)
pO2 16.2 kPa (9.3-13.3)
pCO2 2.1 kPa (4.7-6)
HCO3 19.5 mmol/L (22-26)

Which of the following is the most appropriate intial treatment?

A haemodialysis
B haemofiltration
C intravenous fomepizole
D intravenous methanol
E oral activated charcoal

NOTES:

123
Question: 17

A 56-year-old woman presented with a severe community acquired


pneumonia requiring mechanical ventilation. She also required a continuous
infusion of noradrenaline (0.12 μg/kg/min) to maintain a mean arterial blood
pressure of 65 mmHg. 48 hours after admission to the ITU she became
oliguric and developed acute kidney injury requiring commencement of renal
replacement therapy.

Which of the following is the most appropriate renal replacement therapy


regimen?

A CVVH with an effluent flow rate of at least 20 mLs/min/hour


B CVVH with an effluent flow rate of at least 35 mLs/min/hour
C haemodialysis six times a week aiming for a Kt/V 0.9-1.0 per treatment
D haemodialysis six times a week aiming for a Kt/V >1.2 per treatment
E no clear evidence for any one of the above, over the others

NOTES:

124
Question: 18

A 24-year-old intravenous drug user presented with a severe community


acquired pneumonia. On examination core body temperature was 32oC, blood
pressure was 86/52 mmHg and capillary refill >5 seconds. Despite aggressive
fluid resuscitation through a single intravenous cannula that had been very
difficult to site, his blood pressure remained <90 mmHg systolic.

Investigations:

serum sodium 140 mmol/L (133–144)


serum potassium 6.8 mmol/L (3.3–5.3)
serum urea 34.6 mmol/L (2.5-6.5)
serum creatinine 670 μmol/L (60–120)

INR 2.3 (1-1.2)


APTT ratio 1.2 (1-1.3)

hepatitis B surface antigen positive


hepatitis C antibody positive

ECG broadened QRS complex

Which of the following is most likely to prevent you from considering acute
peritoneal dialysis in this patient?

A coagulopathy
B haemodynamic instability
C hyperkalaemia
D hypothermia
E poor intravenous access

NOTES:

125
Transplant Medicine

(pre-transplant, acute and chronic)

QUESTIONS

126
Question: 1

A 68 year old man with chronic glomerulonephritis electively started home


haemodialysis five nights per week via a mature brachiocephalic fistula.
He had hypertension and suffered a myocardial infarction three years before.
His was taking aspirin, ramipril, bisoprolol, atorvastatin and omacor and his
blood pressure readings in the primary care setting were on average 124/76
mmHg.

Which statement is true when discussing the risks and benefits of


transplantation in this patient?

A a living donor will give the best results for graft and patient survival
B if he received an extended criteria donor kidney his risk of death at two
years would be reduced by 25% compared to remaining on haemodialysis
C current data on the relative benefits of transplantation in the elderly are
based on standard haemodialysis prescription
D psychomotor abilities and memory improve significantly after
transplantation
E those undergoing successful transplantation have a greater mortality in the
first year post operatively than those who continue home haemodialysis

NOTES:

127
Question: 2

A 45 year old woman with end stage renal failure secondary to IgA
nephropathy was referred for consideration of a live donor renal transplant as
her eGFR had declined from 15 to 10 mL/min/1.73m2 (>90) over the
preceding twelve months. Tissue typing results from two potential donors
were as follows:

HLA Typing:
Patient: A 02,02 B 15,3502 DRB1 11, 12 DQB1 02, 0302 DPB1 0201
Donor1: A 02,11 B 35,44 DRB1 04, 15 DQB1 03, 06 DPB1 0401
Donor2: A 01,02 B 08,39 DRB1 03, 04 DQB1 02, 0302 DPB1 0401

Antibody Testing:
B cell PRA: positive, 23%, numbers tested: 30, specificity IgM A3
Luminex class I PRA: positive, 18%,
specificity A25, A26, A34, A66 (IgG), A3 (IgM)
Luminex class II single antigen ID: positive,
specificity DQ5, DQ6, DP1, DP3, DP5, DP9, DP10, DP11,
DP13, DP14, DP17, DP19

Which statement regarding this report is true?

A all IgM antibodies are benign and can be ignored


B donor 1 is compatible with the patient
C donor 2 is compatible with the patient
D the combination of donor1 and the patient can be described as a one A,
two B, and two DR mismatch
E the patient has a persistent HLA-A3 IgM antibody, but this would not
preclude him from receiving a kidney from a HLA-A3 positive donor

NOTES:

128
Question: 3

A 62 year old Afro-Caribbean man with underlying familial focal segmental


glomerular sclerosis received a living donor kidney transplant. The transplant
operation was complicated by unexpected vascular anatomical variation
leading to a prolonged total ischaemic time of 9 hours. The kidney began to
produce urine within minutes of revascularization. His immunosuppression
therapy was ciclosporin, mycophenolate mofetil and prednisolone.

Blood results show:

serum creatinine pre op 950 μmol/L (2.5–7.0)


serum creatinine 24 hours 732 μmol/L (2.5–7.0)
serum creatinine 48 hours 550 μmol/L (2.5–7.0)
serum creatinine 72 hours 549 μmol/L (2.5–7.0)
serum creatinine 94 hours 551 μmol/L (2.5–7.0)

Blood pressure and fluid status were optimally managed in this time.

Which statement is true?

A acute tubular necrosis is a likely diagnosis and if the doppler ultrasound is


unremarkable a conservative approach can be undertaken for the next 24
to 48 hours
B a full blood count, blood film and LDH should be requested urgently to
exclude thrombotic microangiopathy and immunosuppression empirically
changed to tacrolimus
C if doppler ultrasonography shows normal flows and the resistive index is
not elevated the patient should be commenced on plasmapharesis
D in the light of continuing good urine output graft vascular thrombosis is an
unlikely diagnosis
E same day graft biopsy including C4d staining and repeat cross matching
are indicated

NOTES:

129
Question: 4

A 45 year old man with end stage renal failure of unknown aetiology received
a kidney from a live unrelated donor. The kidney was a four HLA antigen
mismatch. Laporoscopic graft retrieval and the implantation were uneventful.

Which statement regarding potential immunosuppression is true?

A for those at high risk of acute rejection, induction strategies using anti-
thymocyte globulin will significantly lower the incidence of this occurring
B mycophenolate sodium (Myfortic) has an overall more favourable side
effect profile compared to mycophenolate mofetil
C the increasingly widespread use of induction therapy with IL-2 receptor
antagonists (Basiliximab, Daclizumab) is based on clinical trial evidence
showing improved graft survival
D the use of the combination mycophenolate/sirolimus compared to
mycophenolate/tacrolimus results in superior long-term graft survival and
renal function
E use of tacrolimus instead of ciclosporin is associated with less CNI toxicity
and lower rates of interstitial fibrosis and tubular atrophy

NOTES:

130
Question: 5

A 55 year old woman with a functioning renal transplant attended her first
annual review clinic. She was on ciclosporin, mycophenolate and
prednisolone for immunosupression. She had a strong family history of
osteoporosis and asked for advice regarding best management to minimize
her risk.

What is the best advice to give?

A bone disease in renal transplant recipients is heterogeneous and dual-


energy x-ray absorptiometry (DEXA) should be supplemented by bone
histology in order to guide therapy
B bone mineral density at the lumbar spine and femoral neck is equally
favourably affected by bisphosphonates and vitamin D analogues
C long-term use of pamidronate and alendronate are equally safe
D patients on tacrolimus develop significantly less osteopenia compared to
those on ciclosporin
E steroid withdrawal would significantly improve her BMD and only minimally
increase the risk of late rejection

NOTES:

131
Question: 6

A 38 year old man with hepatitis C received a renal transplant.


Immunosuppresive therapy of tacrolimus, mycophenolate and prednisolone
was used.

Investigations before transplant showed:

serum alpha-fetoprotein <10kU/L (<10)


serum total bilirubin 14µmol/L (1–22)
serum alanine aminotransferase 27U/L (5–35)
serum aspartate aminotransferase 13U/L (1–31)
abdominal ultrasound normal liver echotexture
liver biopsy no cirrhosis

Fifteen months post transplant, investigations showed:

serum total bilirubin 19µmol/L (1–22)


serum alanine aminotransferase 257U/L (5–35)
serum aspartate aminotransferase 331U/L (1–31)
hepatits c viral load high

What investigations would you undertake next?

A check hepatitis B and HIV serology


B liver biopsy to ascertain the degree of liver fibrosis and activity of hepatitis
C no investigation needed. Commence ultra-low-dose pegylated IFN-alpha
plus standard dose ribavirin
D repeat ultrasound to assess echotexture and structure
E serial monitoring of viral load whilst withdrawing prednisolone

NOTES:

132
Question: 7

A 32 year old woman attended the renal transplant follow up clinic. She had
end stage renal failure secondary to reflux nephropathy and had received a
kidney from her father six months earlier. There had been no complications
since.

In the clinic she sought advice about contraception and pregnancy. Her
eGFR was 64 ml/min.

Which statement is correct?

A barrier contraception is the safest modality


B CNI dose reductions may be required during pregnancy
C conception should be delayed until at least 24 months after transplantation
and contraception practiced until then
D long-acting, subcutaneously placed contraceptive hormone preparations
are highly effective and have been formally tried in the transplant situation
E low-dose estrogen-progesterone oral contraceptive preparations may
precipitate thromboembolism but have no effect on blood pressure

NOTES:

133
Question: 8

An asymptomatic 44 year old renal transplant recipient with type two diabetes
mellitus was seen at 10 weeks post transplant. He was taking ciclosporin and
mycophenolate for immunosuppression and co-trimoxazole prophylaxis. A
ureteric stent had been removed at 8 weeks and he had had an indwelling
urinary catheter for seven days post-transplantation.

Investigations:

serum creatinine baseline 130 μmol/L (2.5–7.0)


serum creatinine at week 10 157 μmol/L (2.5–7.0)
MSU - >100/µL white cells; E.Coli sensitive to co-amoxiclav

Which statement is correct?

A a prolonged course (7 days) of oral co-amoxiclav would be a reasonable


treatment choice
B a single dose of oral co-amoxiclav is likely to eradicate the UTI and
prevent pyelonephritis
C the MSU result indicates over-immunosuppression and as well as from
oral antibiotics the ciclosporin dose should be reduced
D the patient does not need treatment as they are asymptomatic and the
MSU should be repeated in 1 week
E the patient should be admitted for intravenous antibiotics and urgent
allograft biopsy to exclude concurrent rejection

NOTES:

134
Question: 9

A 43 year old woman with end stage renal failure secondary to type 2
diabetes mellitus had been on haemodialysis for two years and wa
undergoing transplantation work up. She had no major co-morbidities, but her
body mass index was 37 kg/m2. Her resting electrocardiogram was normal and she
did not describe any problems with walking briskly for 30 minutes.

Which statement is correct?

A coronary angiography at 3 yearly intervals has been shown to be the


optimal screening test to identify covert coronary artery disease
B exercise electrocardiography is more sensitive than dobutamine stress
echocardiography or myocardial perfusion studies
C screening asymptomatic patients prevents cardiovascular events after
transplantation
D the benefits of regular physical activity on cardiovascular mortality while
waiting for a transplant have been well established
E the risk of significant coronary stenosis (70% or greater in one or more
coronary arteries) is approximately 50%

NOTES:

135
Question: 10

A 64 year old renal transplant recipient was reviewed in clinic nine months
post transplantation.

Investigations:

serum creatinine baseline 115μmol/L (2.5–7.0)


serum creatinine in clinic 145μmol/L (2.5–7.0)

Clinically has was volume depleted and was therefore admitted for
intravenous fluids.

The next morning, investigations showed:

serum creatinine 160μmol/L (2.5–7.0)

A graft ultrasound that day was normal. Transplant biopsy showed mild-to-
moderate arteritis in two blood vessels and moderate tubulitis in
approximately 15% of the sample.

Which statement is correct?

A anti-thymocyte globulin (ATG) is more effective than OKT3 in treating the


worsening renal function
B C4d staining on frozen sections is more sensitive than on formalin fixed
and paraffin embedded tissue
C positive C4d immunohistochemistry is sufficient to initiate therapy with
plasmapheresis and intravenous Ig (IVIg)
D pulse methylprednisolone therapy (0.5 to 1 g/day) as well as changing
ciclosporin to tacrolimus is likely to reverse the pathology
E single dose rituxamab is now standard of care in acute rejection if anti-
HLA antibodies are found

NOTES:

136
Question: 11

An 18 year old woman with ESRD on peritoneal dialysis attended the


transplant assessment clinic. She was accompanied by her sister who was
interested in being a live kidney donor. She wanted to know the long term
risks of kidney donation.

Which statement is correct?

A there is a increased risk of ESRD when compared to normal population


B there is an average 5mm Hg increase in systolic and diastolic BPs than
would be anticipated with aging over a 5 - 10 year period
C there is an increased long term mortality in live kidney donors
D there is no increased risk of hypertension when compared with age
matched controls
E there is no increased risk of urinary protein excretion in donors when
compared to age-matches controls

NOTES:

137
Question: 12

A 45 year old man was being assessed for kidney transplantation. He had
CKD stage 5. He was due to start hospital based haemodialysis via an AV
fistula. His BMI was 32 kg/m2. He had no co-morbidities that precluded
transplantation.

Which statement is correct?

A he should be referred for gastric bypass to help with weight loss prior to
transplantation
B his life expectancy would be better with transplantation compared to being
on dialysis.
C weight loss would reduce the risk of graft loss.
D his BMI precludes renal transplantation.
E his BMI does not increase the risk of postoperative wound-related
complications.

NOTES:

138
Question: 13

A 50 year old Asian man with ESRD secondary to glomerulonephritis was


seen in the transplant assessment clinic. He had been on haemodialysis for 2
years. He had a strong family history of type 2 diabetes. He had read on the
internet about the risk of developing diabetes after kidney transplantation and
was worried about having steroid therapy.

Which statement is not true regarding development of new onset diabetes


after transplantation (NODAT)?

A despite the risk of developing NODAT his survival would be better with
transplantation
B early steroid withdrawal reduces the risk of NODAT compared to low dose
dose steroids
C NODAT has been associated with increased cardiovascular risk and
atherosclerosis and higher graft failures
D randomized trials show trends for higher rates of acute rejection in patients
who are withdrawn from steroids when compared with patients who are
maintained on steroid therapy
E tacrolimus is more likely to cause NODAT compared to ciclosporin

NOTES:

139
Question: 14

A 45 year old man received his third cadaveric renal transplant. His induction
regime included ATG. He was discharged on tacrolimus, MMF and low dose
steroids. He was noted to have worsening graft function on his follow up visit
6 months post transplantation. His serum creatinine had risen to 200μmol/L (baseline
110). He was concordant with his immunosuppression and was clinically well. He
underwent a graft biopsy which showed the following:

Which is the most appropriate first step in his management?

A convert tacrolimus to ciclosporin


B convert MMF to leflunamide
C reduce dose of MMF
D start cidofovir
E start ciprofloxacin

NOTES:

140
Question: 15

A 40 year old woman with ESRD secondary to primary FSGS recieved a


deceased donor renal transplant. A year after transplantation she was found
to have 4+ proteinuria on dipstick. Further evaluation revealed the following:

Serum creatinine 150 µmol/L (60–110)


Serum albumin 30 g/L (37–49)
urinary protein:creatinine ratio 670 mg/mmol (<15)

A renal biopsy showed features of FSGS.

Which statement regarding FSGS in transplantation is true?

A collapsing variant is more likely to cause recurrent GN as opposed to de


novo disease
B familial forms of FSGS are more likley to recur than non-familial ones
C FSGS in transplant does not lead to significant graft loss
D plasma exchange can lead to remission of proteinuria in recurrent disease
E recurrent or de novo FSGS is the commonest cause of proteinuria in
transplant recipients

NOTES:

141
Question: 16

A 65 year old man had received a cadaveric renal transplant 9 years earlier.
He presented with worsening exertional breathlessness. Three weeks earlier
he was started on sirolimus following a diagnosis of CAN. His other
immunosuppression included MMF and prednisolone. On examination his
temperature was 37.8° C, blood pressure 134/90 mmHg, pulse 100 beats per
minute, oxygen saturation 89% (room air), and respiration rate 25 breaths per
minute. He had no peripheral oedema and jugular venous pressure was not
elevated. Fine bibasal crepitations were present in both lung field.

Investigations:

haemoglobin 10 g/dL (13–18)


white cell count 7.8 x 109/L (4–11)
serum sodium 139 mmol/L (137–144)
serum potassium 4 mmol/L (3.5–4.9)
serum urea 9.4 mmol/L (2.5–7.0)
serum creatinine 128 μmol/L (60–110)
(baseline creatinine between 110 - 130 μmol/L)
CXR revealed bilateral pulmonary infiltrates.

What is the most appropriate initial step in his management?

A broad spectrum antibiotics


B discontinue sirolimus
C high dose septrin
D intravenous diuretics
E intravenous ganciclovir

NOTES:

142
Question: 17

A 35 year old man was found to have acute worsening of allograft function.
He received a cadaveric renal transplant 7 years earlier and his
immunosuppression was MMF and ciclosporin. His serum creatinine had
risen from a baseline of 140 μmol/L to 324μmol/L. He underwent an allograft
biopsy which showed Banff grade 2b rejection. Staining for C4d was negative.
His ciclosporin level was subtherapeutic for 1 week prior to the biopsy.

What is the most appropriate treatment?

A change ciclosporin to tacrolimus


B increase ciclosporin dose
C intravenous methyprednisolone
D intravenous OKT3
E intravenous rabbit anti-thymocyte globulin (ATG)

NOTES:

143
Question: 18

A 56 year old recipient of a live unrelated kidney transplant 8 years earlier


underwent allograft biopsy for worsening allograft function. The biopsy
revealed features of chronic allograft nephropathy. He was hypertensive and
had proteinuria (urinary protein:creatinine ratio 150 mg/mmol (<15)). His
immunosuppression was tacrolimus and prednisolone.

What is an appropriate change in his management?

A convert tacrolimus to ciclosporin, continue prednisolone


B convert tacrolimus to sirolimus, continue prednisolone
C increase tacrolimus dose, add MMF, continue steroids
D no changes to immunosuppression, add ACE inhibitor
E reduce tacrolimus dose, add MMF and continue prednisolone

NOTES:

144
Question: 19

A 29 year old woman recieved a live-related transplant from her mother 1


year earlier. She had stable graft function with a serum creatinine of
90μmol/L. Her immunosuppression included tacrolimus, MMF and low dose
prednisolone. She mentioned that she would like to consider having children.

Which treatment regimen would be the most appropriate?

A add low dose aspirin


B consider changes to immunosuppression post-conception
C convert MMF to azathioprine
D convert MMF to sirolimus
E stop MMF

NOTES:

145
Question: 20

A 46 year old man presented with the following lesions on his left shin.

He had received a cadaveric renal transplant 10 years earlier. His


maintenance immunosuppression included tacrolimus, azathioprine and
prednisolone.

What would be the most appropriate next step in his management?

A convert tacrolimus to cicloporin; stop azathiprine


B convert tacrolimus to sirolimus; reduce azathioprine
C reduce tacrolimus, convert azathioprine to MMF
D stop tacrolimus and azathioprine
E stop tacrolimus; reduce azathioprine

NOTES:

146
Chronic Kidney Disease

(anaemia, bone disease, drug prescribing,

reno & cardiovascular disease,

hyperlipidaemias & nutrition)

QUESTIONS

147
Question: 1

A 68 year old man with known chronic kidney disease had a blood pressure of
165/75 mmHg despite taking ramipril, bisoprolol, nifedipine and frusemide.
Renal angiography showed significant bilateral renal artery stenosis.

Investigations:

serum creatinine 196 mol/L (60-110)

What is the most likely outcome following renal revascularisation?

A improvement in glomerular filtration rate


B improvement in patient survival
C improvement in blood pressure control
D none of the above
E reduction in resistive index of both kidneys

NOTES:

148
Question: 2

A 76 year old man with long-standing hypertension was admitted with


increasing shortness of breath. His blood pressure was 170/70 mmHg, heart
rate 92 beats per minute, respiratory rate 28 per min, and pulse oximetry 96%
on FiO2 0.4.

Investigations:

serum creatinine 135 mol/L (60-110)

chest X-ray pulmonary oedema

echocardiogram left ventricular ejection fraction


of 60%

Along with diuretic therapy, he was started on an ACE inhibitor. During his in-
patient stay his serum creatinine progressively increased to a serum
creatinine of 195 μmol/L (60-110) over a period of 5 days. His blood pressure
at this stage was 145/80.

What is the most appropriate next step?

A add a beta-blocker
B add calcium channel blocker
C increase the ACE inhibitor dose
D no change in therapy
E stop the ACE inhibitor

NOTES:

149
Question: 3

A 64 year old woman with membranous nephropathy attended the general


nephrology clinic complaining of fatigue. She was taking warfarin for recurrent
deep vein thrombosis and was free of any other symptoms. Clinical
examination was unremarkable. On examination her blood pressure was
131/77 and there was a trace pf pedal oedema.

Investigations:

haemoglobin 9.2 g/dL (13–18)


white cell count 4.7 109/L (4–11)
platelet count 202  109/L (150–400)
serum sodium 139 mmol/L (137–144)
serum potassium 4.6 mmol/L (3.5–4.9)
serum urea 11.4 mmol/L (2.5–7.0)
serum creatinine 192 µmol/L (60–110)
serum ferritin 30 µg/L (15–300)

faecal occult blood positive

What would be the most appropriate initial therapy for the anaemia?

A blood transfusion
B intravenous iron sucrose
C oral ferrous sulphate
D proton pump inhibitor pending endoscopy
E start ertythropoeitin

NOTES:

150
Question: 4

A 37 year old man with stage 3 chronic kidney disease of uncertain aetiology
was seen in the renal outpatient clinic. He had no symptoms of renal disease
and was otherwise well. On examination, his body mass index was 29, blood
pressure 130/70 mmHg and there were no stigmata of hyperlipidaemia or
cardiovascular disease.

Investigations:

serum urea 12.7 mmol/L (2.5–7.0)


serum creatinine 166 µmol/L (60–110)
serum cholesterol 6.6 mmol/L (<5.2)
serum LDL cholesterol 4.1 mmol/L (<3.36)

What is true of lipid management in chronic kidney disease?

A cholestyramine therapy reduces risk of progression of CKD


B hyperlipidaemia improves with a low phosphate diet
C statin therapy reduces mortality in CKD stage 5
D statins can improve C reactive protein as well as cholesterol
E statins reduce the antihypertensive burden in CKD patients

NOTES:

151
Question: 5

A 46 year old man with hypertensive nephrosclerosis and stage 3 chronic


kidney disease was referred to the renal outpatient clinic with difficult to
control blood pressure. He had undergone a primary angioplasty for an ST
elevation myocardial infarction six months previously. He was taking ramipril,
valsartan, bisoprolol, and doxazosin but was unable to tolerate high dose
titrations due to side effects.

Investigations:

24 hour blood pressure mean daytime recording of


156/69 mmHg with a >10%
overnight drop

What is the prognostic significance of poor blood pressure control in this


patient?

A a nocturnal dip of >10% is associated with more rapid decline in GFR


B he is more likely to progress to end stage renal disease because of dual
ACE inhibitor and ARB therapy than his hypertension
C hypertension carries a better prognosis than hypotension in CKD
D isolated systolic hypertension need not be addressed in CKD
E reducing his blood pressure will not reduce his risk of CVA

NOTES:

152
Question: 6

A 59 year old woman with type 1 diabetes mellitus on peritoneal dialysis was
seen in the diabetic foot clinic with a painful, dusky left great toe and an
associated ulcer over the distal phalanx. Her other toes were warm with a
capillary refill time of less than 2 seconds. She had bilateral weak pedal
pulses.

Investigations:

plain X-ray left foot osteomyelitis in the distal


phalanx of the left great toe
and diffuse vascular
calcification

What is true of the finding of vascular calcification?

A it indicates that she takes a calcium-based phosphate binder


B it is due to diabetes mellitus and is unrelated to renal disease
C it is likely to be progressive regardless of management
D it is likely to regress with better control of her PTH
E it is present in all dialysis patients

NOTES:

153
Question: 7

A 50 year old man presented with two large areas of skin necrosis on his left
thigh. He had been haemodialysed through a left femoral loop graft for the
past 5 years for chronic kidney disease secondary to Henoch Schonlein
purpura. He had undergone coronary angiography 7 days previously as part
of work up of exertional chest pain. On examination, his body mass index was
31 and he had intact pulses throughout his lower extremities. The necrotic
areas of skin were superficial, measuring 4 to 5 cm each.

Investigations:

serum calcium 2.42 mmol/L (2.2–2.6)


serum phosphate 2.1 mmol/L (0.8–1.4)
plasma parathyroid hormone 85 pg/ml (0.9–5.4)
serum C-reactive protein 37mg/L (<10)

What is the likely underlying cause of this clinical condition?

A atheroembolic disease from the graft


B cholesterol emboli
C calciphylaxis
D steal syndrome
E recurrent Henoch Sconlein purpura

NOTES:

154
Question: 8

A 32 year old man was reviewed in the renal clinic for longstanding chronic
kidney disease due to IgA nephropathy. Hyperphosphataemia had been
noticed 3 months previously and he had been reviewed by the dieticians at
that time. His medications were ramipril, losartan and amlodipine. On
examination blood pressure was 114/75.

Investigations:

serum urea 18.3 mmol/L (2.5–7.0)


serum creatinine 200 µmol/L (60–110)
plasma parathyroid hormone 45 pmol/L (0.9–5.4)
serum corrected calcium 2.20 mmol/L (2.2–2.6)
serum phosphate 1.8 mmol/L (0.8–1.4)

What is the most appropriate first line of management?

A alphacalcidol
B calcium acetate
C cinacalcet
D paricalcitol
E sevelamer hydrochloride

NOTES:

155
QUESTION: 9

A 47 yr old Afro-carribean male was referred by his general practitioner with


peripheral oedema. On examination his blood pressure was 150/100 mmHg
and there was pitting oedema to mid calf.

Investigations:

serum sodium 142 mmol/L (137–144)


serum potassium 4.7 mmol/L (3.5–4.9)
serum creatinine 255 μmol/L (60–110)

urinalysis 4+ protein, no blood

24 hr urine protein 6g/24hours (0.2)

renal biopsy membranous nephropathy

He was commenced on furosemide and lisinopril. He was reviewed 2 weeks


later. On examination his blood pressure was 125/75 mmHg and there was
oedema to mid calf.

Investigations:

serum sodium 142 mmol/L (137–144)


serum potassium 45.4 mmol/L (3.5–4.9)
serum creatinine 286 μmol/L (60–110)

24 hr urine protein 3g/24hours (0.2)

Which one of the following is the best course of action?

A continue current treatment and recheck serum creatinine in 1 week


B start steroids and cyclophosphamide
C stop furosemide
D stop lisinopril and start losartan
E stop lisinopril and start bisoprolol

NOTES:

156
QUESTION: 10

A 72-yr-old Caucasian man with renovascular disease was admitted with


unstable angina. His serum creatinine one month before admission was
220µmol/L (60–110). A coronary angiogram demonstrated a 90% lesion in his
left anterior descending coronary artery. Angioplasty with stenting was
proposed by the cardiologists.

Which one of the following is true regarding his therapy?

A coronary angiography and angioplasty should not be done because of the


risk of acute renal failure
B he is at higher risk for restenosis compared with a patient with a normal
serum creatinine
C long-term survival is better with coronary artery bypass grafting than with
angioplasty
D medical therapy is more effective in improving long-term survival than is
percutaneous coronary intervention
E the use of a platelet glycoprotein IIb/IIIa inhibitor should be avoided
because of the increased risk for bleeding

NOTES:

157
QUESTION: 11

A 45-yr-old Asian woman with chronic glomerulonephritis and advanced renal


insufficiency attended for routine clinic review. Her regular medications were
furosemide, amlodipine, and enalapril. On examination her blood pressure
was150/90 mmHg.

Investigations:

haemoglobin 9.5 g/dl (13–18)


serum creatinine 395µmol/l (60–110)
serum corrected calcium 2.20 mmol/L (2.2–2.6)
serum phosphate 1.8 mmol/L (0.8–1.4)

24 hr urine protein 3g/24hours (0.2)

echocardiogram left ventricular hypertrophy

Which of the following is most likely to occur following commencement of


erythropoietin?

A an acceleration of renal disease progression


B an improvement in quality of life measures
C a reduction in cardiovascular morbidity and mortality
D a reduction in the development of coronary heart disease
E a reduction in urinary albumin excretion rate

NOTES:

158
QUESTION: 12

A 50 year old man attended for routine review. He had been on haemodialysis
through an AV fistula for 7 years. There had been continued problems with
medication compliance and he refused to follow a phosphate restricted diet.

Investigations:

serum corrected calcium 2.55 mmol/L (2.2–2.6)


serum phosphate 2.03 mmol/L (0.8–1.4)
plasma parathyroid hormone 154 pmol/L (0.9–5.4)

He was commenced on intravenous calcitriol and dialysate calcium level


maintained at 1.25mmol/L. He was reasssed 2 weeks later.

Investigations:

serum corrected calcium 2.65 mmol/L (2.2–2.6)


serum phosphate 2.12 mmol/L (0.8–1.4)
plasma parathyroid hormone 120 pmol/L (0.9–5.4)

What is the most appropriate management?

A addition of calcium acetate


B addition of cinacalcet
C addition of lanthanum carbonate
D increase calcitriol
E parathyroidectomy

NOTES:

159
QUESTION: 13

A 68-yr-old Caucasian woman attended for routine review. She had been on
haemodialysis for 3 years for ESRD secondary to diabetes and 6 months
previously had a myocardial infarction requiring placement of a stent in the
circumflex coronary artery. Review of her laboratory values revealed a history
of poor phosphate control with many values ≥2.4mmol/L (0.8–1.4) despite
trials of calcium carbonate, calcium acetate, and sevelamer. Serum calcium
values had always been between 2.50mmol/L and 2.6mmol/L (2.2–2.6). Intact
parathyroid had been consistently ≤21 pmol/L (0.9–5.4).

Investigations:

echocardiogram heavily calcified aortic valve

Which of the following best describes her future management?

A as the parathyroid hormone level is not significantly elevated, phosphate


binders are not necessary
B calcium-based phosphate binders may increase the risk of progression of
valvular and medial vascular calcification
C cinacalcet is an effective phosphate binder and should be started
immediately
D lanthanum carbonate decreases bone turnover and would worsen her
adynamic bone disease
E sevelamer hydrochloride has been associated with an increased
incidence of adynamic bone disease and would not be suitable in this
case

NOTES:

160
QUESTION: 14

On reviewing a recently completed 24-h ambulatory BP recording on a 52-yr-


old Caucasian man with hypertension and stage 2 CKD (eGFR 76 ml/min per
1.73 m2), it was noted that the mean night time systolic BP (SBP) was 5%
less than the mean daytime SBP.

Which one of the following is true regarding the relationship between diurnal
BP variation assessed with ambulatory BP monitoring and CKD?

A a nocturnal fall in mean SBP≤10% occurs only in patients with elevated


24-h SBP levels
B non-dipping does not appear in patients with CKD until the GFR has fallen
below approximately 30 ml/min per 1.73 m2
C non-dipping has been shown to be associated with the presence of CKD in
patients with type 1 but not type 2 diabetes
D patients with nocturnal fall in SBP≤10% have a more rapid decline in GFR
compared with those who are “dippers”
E prospective trials have shown that non-dipping is not associated with
increased risk for cardiovascular mortality

NOTES:

161
QUESTION: 15

A 66 year old man with hypertension and stage 3 chronic kidney disease
(eGFR 34 ml/min per 1.73 m2) attended for routine review.

Investigations:

serum cholesterol 6.6 mmol/L (<5.2)


serum LDL cholesterol 4.1 mmol/L (<3.36)

Which of the following is most likely to be true in patients with CKD?

A hyperlipidaemia is uncommon in late stages of CKD


B lipid lowering with cholestyramine has been demonstrated to reduce the
rate of renal disease progression
C statin therapy has been demonstrated to be effective at reducing risk for
death in ESRD
D treatment with simvastatin reduces lipid levels as well as C-reactive
protein in CKD
E treatment with atorvastatin reduces the need for ACEI or angiotensin II
receptor blockers in 40% of patients with stage 4 CKD

NOTES:

162
Disorders of fluid and electrolyte and acid base regulation

& hypertension

ANSWERS

Answer: 1
A – Cystinosis: autosomal recessive defect in cystinosin in lysosomes leading
to lysosomal cystine accumulation; most cases present in childhood;
multisystem disorder
 eyes (corneal crystals)
 renal (Fanconi syndrome of pct dysfunction)
 endocrine (hypothyroidism, hypogonadism, IGT & diabetes)
 neuro (dysphagia, myopathy, psychomteric defects)
Diagnosis: leucocyte cystine concentration, molecular
Treatment: cysteamine (mercaptamine)

Answer: 2
B - hypokalaemic metabolic alkalosis with normal BP- consider
Bartter’s syndrome: mode of presentation & severity varies; often present in
childhood with polyuria (nocturia), FTT, muscle weakness; can be associated
with nephrocalcinosis; can cause ESRD.
Gitelman’s: often diagnosed later, freq diagnosed on routine testing but
patient may complain of fatigue, muscle cramps, tetany. Distinguished from
Bartter’s by hypomagnesaemia and hypocalciuria. Associated with
chondrocalcinosis.

Answer: 3
E – the combination of hypokalaemia, probable metabolic alkalosis,
hypomagnesaemia, high urine chloride and normal blood pressure is
compatible with a diagnosis of Gitelman’s Syndrome. This would be
associated with hypocalciuria and therefore measurement of urine calcium
excretion is an appropriate investigation. The dexamethasone suppression
test would be used for Cushing’s, CYP11B1/B2 for glucocorticoid remediable
aldosteronism, salivary sodium for pseudohypoaldosteronism type 1, and
renin/aldo ratio for defectsof the rennin aldosterone axis. All of these would be
expected to be associated with hypertension.

Answer: 4
D - He has a proximal (type 2) RTA and there is phosphate and glucose
wasting and has predominantly LMW proteinuria. Sodium delivery to the distal
nephron will increase risk of stones. Oral loading with sodium bicarbonate to
some extent differentiates type 2 RTA from type 1. RTA is not a feature of
Fabry-Anderson disease. The raised CK is consistent with a mitochondrial

163
cytopathy, increasingly recognised as a cause of proximal tubular dysfunction.
The utility of a biopsy lies in the EM where mitochondrial number will be
abnormal.

Answer: 5
B - A, C, and D refer to activity of cortisol at the mineralocorticoid receptor,
an effect that is normally inhibited because of metabolism of cortisol by 11-β
hydroxysteroid dehydrogenase – which can be inhibited by liquorice.
Aldosterone would be low in all three conditions. E is true to some extent but
the genetic defect results in a chimerism where aldosterone production is
under the control of the ACTH promoter. Therefore, inhibiting ACTH
production with cortisol/dexamethasone reduces aldosterone production and
is therefore the most correct answer.

Answer: 6
A - The underlying condition is pseudohypoaldosteronism type 2 (Gordon’s
syndrome), exacerbated by the use of an ACEi. The defect is transmitted as
an autosomal dominant trait and affects WNK kinase 1 or 4. This defect
causes increased activity of the thiazide sensitive Na-Cl co-transporter. The
best treatment option is therefore a thiazide diuretic. Liddle’s syndrome
causes hypertension with hypokalaemia, PHA type 1 causes hyperkalaemia
with normotension, primary adrenal insufficiency causes hyperkalaemia with
normo/hypotension, and type IV RTA would be unusualwith this level of renal
function.

Answer: 7
B - This patient has glucocorticoid-remediable aldosteronism (GRA) which is
inherited as an autosomal dominant trait and typically presents with
hypertension at young age. Administration of a thiazide diuretic frequently
leads to the development of marked hypokalaemia.
Aldosterone release in GRA is primarily under the influence of ACTH. This is
released in a circadian rhythm. Accordingly Aldosterone is above normal for
only part of the day.
Dexamethasone is the most appropriate treatment, however amiloride and
spironolactone will both improve the manifestations although they don’t
correct the underlying defect. Metyrapone is a steroid 11β-hydroxylase
inhibitor which will reduce aldosterone levels but less efficiently that
dexamethasone, Dutasteride is a 5α-reductase inhibitor involved in testerone
metabolism and therefore has no role in this condition.

Answer: 8
B - This patient presents with a high anion gap metabolic acidosis. Linezolid
has been described to cause lactic acidosis, typically developing several
weeks into therapy. Topiramate is a cause of normal anion gap metabolic
acidosis due to inhibition of carbonic anhydrase.
Lactic acidosis due to ischaemic bowel is unlikely given the lack of acute
illness in the patient. Metformin can cause lactic acidosis but mostly in the
setting of renal failure or liver disease.

164
Answer: 9
D - This patient exhibits features of Fanconi syndrome (FS) with severe
proximal tubular dysfunction as well as normal-anion gap metabolic acidosis
and a concentrating defect. FS is a well-established complication of multiple
myeloma, but in 96% of cases occurs in the setting of kappa light chain
disease. The temporal association between initiation of therapy and the
development of FS make this medication the likely cause.

Answer: 10
D - The combination of hyponatraemia, high urine osmolality, high urine
sodium, and normal adrenal and thyroid tests makes SIADH most likely.
Although he was normally taking bendroflumethiazide, if this were the cause
of the hyponatraemia the urine sodium should be lower (due to volume
depletion) having stopped the drug 5 days earlier. Translocational
hyponatraemia cannot be the cause – to get the sodium to 115 the plasma
glucose would have to be about 70mmol/L.

Answer: 11
A - Inspite of the apparently normal morning cortisol (affected by prednisolone
use), adrenal insufficiency is the most likely cause. Volume depletion is
unlikely given the high urine sodium, TSH is often elevated in patients with
adrenal insufficiency and is therefore not implicated, SIADH can only be
diagnosed once other diagnoses are excluded, and translocational
hyponatraemia does not happen with this level of hyperglycaemia (see
answer 10).

Answer: 12
C - This patient has developed a metabolic alkalosis with a high urine chloride
and a normal BP. The presence of a non-reabsorbed anion (penicillin) leads
to an increase in distal Na delivery with increased cation (K or H) secretion.
This can be easily corrected by IV 0.9% Saline.

Answer: 13
A - This lady has developed pyroglutamic acidosis secondary to ‘therapeutic’
ingestion of acetaminophen. She has high plasma 5-oxoproline levels and
stopping the co-proxamol will slowly resolve the metabolic acidosis. In some
studies intravenous N-acetylcysteine has been used with some effectiveness.

Answer: 14
E – She has true polyuria, with a low urine osmolality (which argues against
significant glycosuria). In response to water deprivation, there is an
appropriate fall in urine volumes and rise in urine osmolalitywhich indicates an
intact vasopressin-renal axis compatible with primary polydipsia.This
response would not be seen with diabetes insipidus or hypercalcaemia.

Answer: 15
E – White cell counts above 70000x109/L can cause pseudohyperkalaemia
caused by potassium release from the white cells during clot formation. This
can be identified by measuring the plasma and serum potassium

165
concentration simultaneously. Renin-aldosterone ratios are unhelpful in this
setting but a short synacthen test may be appropriate. Prompt spinning down
of the sample is usually required for assessment of hypokalaemia – white
cells in the blood can take up potassium if a blood sample is left standing in
the warm. Red cell membrane permeability studies are used to identify rare
familial forms of hyperkalaemia due to abnormal potassium permeability.

166
APCKD, hereditary nephritis

& less common renal conditions

ANSWERS

Answer: 1
C – This is Fabry disease which is a lysosomal storage disease caused by
deficiency of -galactosidase A which leads to accumulation of
globotriaosylceramide & ceramide trihexose. X-linked recessive (>100
mutations), can affect females but usually milder & start later; multisystem
(Classic) & cardiac forms.
 skin (angiokeratomas)
 eyes (corneal opacities)
 kidneys (progressive CKD with MHT & proteinuria, ESRD in 40-50s)
 heart (IHD, conduction, valvular lesions, LVH)
 neuro (autonomic dysfunction, acroparaesthesia, CVA)
Diagnosis: exam and slit lamp; urine microscopy, -Gal A activity, molecular,
renal biopsy
Treatment with recomb human -Gal A reduces neuropathic pain & stabilises
CKD- must continue treatment post-transplant.

Answer: 2
D – Thin membrane disease: autosomal dominant, large number of mutations
identified in collagen genes (COL4A4 & COL4A3); 5-9% of pop have TMN,
20-25% of those with isolated MHT; lifelong MHT; macroHt may occur; HTN
may be more common, proteinuria uncommon, prog CKD rare; deafness
ABSENT. GBM diffusely thin but otherwise normal (< 250nm, norm 350-
450nm)- normal collagen chain distribution on renal biopsy.

Answer: 3
A – Alport’s disease: defect in synthesis of type IV collagen; type IV collagen
composed of 2 collagen trimers (1-6 chains).
X-linked (5, 80%), auto rec (3,4,5 15%) & auto dom (3,4 5%)
multisystem disease:
 renal (micro & macroHt, proteinuria, HTN, CKD & ESRD)
 sensorineural deafness
 eyes (anterior lenticonus)
Renal biopsy: IF: absence of 3(IV), 4(IV) and 5(IV); EM: lamellation of
GBM, variability in thickness

Answer: 4
C – Tuberose sclerosis: autosomal dominant (TSC1 and 2- tumour
suppressors); multisystem disease associated with hamartomas in multiple
organs:

167
 CNS (epilepsy, learning difficulties)
 skin (shagreen patch, hypomelanotic macule, ungual fibroma …)
 cardiac (rhabdomyoma)
 kidney (angiomyolipoma, cysts, RCC)
Renal angiomyolipomas: benign; can cause haematuria; usually grow in size
& become more vascular as grow; unlikely to become RCC
CKD most usual in those with multiple cysts

Answer: 5
C – see answer for Ans 11

Answer: 6
E - The history of recurrent nephrolithiasis leading to a nephrectomy raises
the possibility of primary hyperoxaluria as the cause of ESRD. The poor
clinical course and multiple complications after starting haemodialysis are
consistent with deposition of oxalate in multiple tissues and organs. The
increased echogenicity of the myocardium noted on the echocardiogram was
due to deposition of oxalate crystals. The clinical history does not suggest
other causes of infiltrative cardiomyopathy such as amyloidosis or Fabry
disease. The age and clinical history are not consistent with a diagnosis of
cystinosis. Except for nephrolithiasis, the clinical history and findings do not
support a diagnosis of Dent disease.

Answer: 7
D–

Answer: 8
C – see Jais JP, JASN 11: 649-657, 2000

Answer: 9
B - By the time ESRD is reached the minority of nephrons are cystic though
anatomy is markedly changed. Chloride dependent shunting is cAMP
dependent. Pkd1 mutation is more common by some way. E is true of
polycystin-1.

Answer: 10
E - No RCTs for the use of Vaptans in this context have been reported. Whilst
circulating levels of renin are high in ADPCKD which supports theoretical
benefit of ACEi/ARB this has not been shown to be of benefit in RCTs. Statins
improve blood flow but again no trial evidence to support their use and a high
fluid intake may well suppress ADH secretion but as with other interstitial
renal disease the ability to secrete a free water load is impaired and
hyponatraemia a real risk.

Answer: 11
D - Medullary cystic kidney disease is thought to arise due to polymorphisms
in uromodullin and an inability of the manufactured protein to be secreted
from the thick ascending limb of the loop of Henle. Sodium reabsorption is
reduced. Nephronopthisis is rarely diagnosed after adolescence. FE urate is
reduced as proximal tubular urate resorption is increased because of volume

168
depletion secondary to salt and water loss. Tamm Horsfall (aka uromodullin)
is increased in abundance.

Answer: 12
B - He has Fabry’s disease. See Ans 1

Answer: 13
B - Although a rare presentation the renal and hepatic changes makes this
the most likely diagnosis.

Answer: 14
D – see Ans 3: note majority of affected individuals are the result of X-linked
inheritance; deafness is sensori-neural; mutations arise in collagen type IV.

169
Renal vasculitis

SLE, diabetic nephropathy

interstitial nephritis & chronic GN

ANSWERS

Answer: 1
D – Assess renal vasculature to rule out renal vein thrombosis as a cause of
increased proteinuria prior to further decisions. They are at high risk for
thrombotic events.
Repeating the histology will not change management as there is already a
clear diagnosis; if cyclophosphamide is not working then there is little to gain
from extending the course.

Answer: 2
C – Immune-complex HIV-associated glomerulonephritis. Serum immunology
is not consistent with SLE (subendothelial deposits in this case are due to
untreated HIV); histology not consistent with 3 or 4; HIV associated
nephropathy would not have full house immunology.

Answer: 3
C – GBM positivity has clearest evidence for plasma exchange. If dialysis
dependent then chance of renal recovery minimal so as a risk / benefit
equation, being dialysis independent at presentation is the best argument for
plasma exchange.

Answer: 4
A – She has polyarteritis nodosa. Biopsy may be dangerous / fail to show
affected areas.

Answer: 5
A – This is scleroderma renal crisis (thick skin; hypertension; ulcers). There is
no urgent need to dialyse.

Answer: 6
D – Excess calcium is absorbed from the gut in sarcoidosis, which is excreted
in the urine. Nephrolitiasis is common in sarcoid, affecting up to 14% and can
be the presenting feature in 4%. It is however less common than
nephrocalcinosis which is the most common cause of renal failure in
sarcoidosis. Sarcoid does not directly affect the glomerulus. Granulomatous
interstitial nephritis is common in sarcoid but does not normally lead to renal
insufficiency.

170
Answer: 7
D - There is little prospective data. The best published trial is de Groot et al,
Ann Intern Med. 2009 May 19;150(10):670-80.
It is well recognised that pulsed iv cyclophosphamide allows a lower
cumulative dose to be given (15.9g vs. 8.2g de Groot et al following Cyclops
style regime).
Several studies show a reduced incidence of leucopoenia with pulsed
treatment: Ann Intern Med. 2009 May 19;150(10):670-80; Arthritis Rheum
1998 Oct;41(10):1835-44. In some studies there is also an increased
incidence of severe infection, but this is not a consistent finding.
Several studies (the best being de Groot) demonstrate no difference in the
time frame for induction of remission or in the proportion of patients achieving
remission. The main limitation is the size of trials (the largest being 149
patients). Meta analysis is limited due to marked heterogeneity between
published data, but de Groot et al NDT (2001) 16:2018-2027 combined three
prospective unblended RCTs and found that pulsed iv cyclophosphamide was
at least as effective as oral in inducing remission.

Answer: 8
C - Again, the available data is limited. The largest RCT is by Jayne et al
JASN 18:2180-2188, 2007. This compared the use of plasma exchange
against the use of pulsed methylprednisalone whilst using oral
cyclophosphamide. One of the inclusion criteria was a serum creatinine of
>500 as this is the EUVAS definition of “severe renal disease”. In this setting,
they found benefit in plasma exchange on renal outcomes but not in long term
patient survival or in the rate of adverse events. Previous, smaller trials have
failed to find benefit in less severe disease Pusey et al, KI 40(1991) 7570763
or have been very small and designed to answer different questions
Nakamura et al NDT (2004) 19:1935-1936.
Pusey et al demonstrated improved renal survival in the context of acute
dialysis dependence if treated with PEX vs. drugs alone (10 / 11 patients vs. 3
/ 8 patients, p=0.041). Jayne et al did not publish figures at to the number of
patients dialysis dependent t presentation / throughout the course of
treatment, but specifically excluded those who were dialysis dependent for
greater than two weeks as they were felt unlikely to recover function.
Benefit of PEX in the context of anti-GBM antibodies has been much longer
established; Lockwood et al Lancet 1:711-715, 1976.

Answer: 9
D - This man has nephrotic range proteinuria with moderate renal impairment.
The rash and reduced complement levels make membranous nephropathy
less likely as a diagnosis. The presence of Hepatitis C antibody in this setting
should prompt consideration of Type II cryoglobulinaemic vasculitis.
Johnson et al KI 46(1994) 1700-1704 demonstrated a reduction in proteinuria
with the treatment of HCV using interferon alpha without significant
improvement in renal function (milder dysfunction, however, sCr average
1.8mg/dl). A later study Misiani et al NEJM 1994 Mar 17;330(11)751-756
confirmed this reduction in proteinuria but also demonstrated a post treatment
improvement in sCr 1.65mg/dl vs1.15mg/dl (p=0.006). In all studies, there are
large numbers of relapses when treatment is withdrawn. Interferon alfa has

171
been shown to be safe in advanced (dialysis dependent) renal dysfunction
Kokoglu et al J Gastroenterol Hepatol 2006 Mar;21(3):575-80.
Concurrent treatment with ribavirin has been shown to increase the chances
of sustained viral clearance but there have been many concerns about its use
in renal dysfunction due to severe haemolytic anaemia anaemia as a side
effect when levels accumulate. However there have been two trials
suggesting that with dose reduction it can be a safe treatment Rendina et al J
Hepatol 2007 May;46(5):768-74 showed that use of ribavirin was safe in
dialysis dependent patients. Bruchfeld et al NDT (2003) 18: 1573-1580
demonstrated that with close monitoring of Hb and appropriate iron and epo
supplementation use of ribavirin was safe down to an eGFR of 10ml/min at
entry. This is supported by Alric et al Am J Kid Dis 2004 Apr;43(4):617-23.

Answer: 10
B - This man has had gross haematuria suggesting a non-glomerular origin.
This is on a background of having gone from being dipstick negative to
dipstick positive for blood. He has also received two courses of
cyclophosphamide so it is reasonable to assume that his cumulative total
dose is high. The most important diagnosis to prove / exclude is of bladder
cancer which means referral for urgent cystoscopy.
The main paper on this issue is from Talar-Williams et al Ann Intern Med
1996;124:477-484. This analysis was based on 147 patients who were
treated with high dose oral cyclophosphamide. They demonstrated that
smokers were likely to develop non glomerular haematuria (NGH) earlier (34
vs. 56 months) but the proportion of smokers vs. non-smokers who eventually
developed NGH did not differ to statistic significance (though they applied
very strict criteria for significance!). They also noted that urine cytology
results did not correlate with cystoscopic results with results ranging from
normal to suspicious dysplasia.

Answer: 11
E - Treatment of idiopathic membranous nephropathy is challenging. The
natural history of the disease is for spontaneous remission in up to 30%,
making data analysis difficult. A study of 184 patients in the Toronto
glomerulonephritis registry Pei et al KI, 42(1992), 960-966 assessed variables
to determine which were predictive of chronic renal insufficiency – of those
assessed, initial creatinine clearance and rate of change of creatinine
clearance were the best. Persistence of proteinuria also factored into the
model. This was validated by Cattran et al KI 51 (1997) 901-907and can be
used to assess risk of progression to a GFR of <60ml/min. Despite the
relatively high level of proteinuria, the normal creatinine and creatinine
clearance and the stability mean that she is at low risk of progression to
chronic renal insufficiency. As such, she does not warrant treatment with
cytotoxic agents.
Angiotensin inhibition is a mainstay of managing renal disease. The main
studies do not present separate data for membranous nephropathy and it has
been presumed that the benefits of proteinuria reduction are not disease
specific. Specific studies assessing angiotensin inhibition in membranous
have been small and not blinded or randomised. The benefit seen is less
than in overall figures and shows a reduction of less than 30%. The most

172
beneficial data is seen in Gansevoort et al NDT 1992;27 Suppl 1:91-96 where
14 patients treated with and ACEi vs 14 in a control group were compared.
They showed respective falls in proteinuria from 9.8 to 3.9g in the ACEi group
compared to 6.9 to 5.5g/day in the control group. This level of benefit is not
seen in other (small) trials, again perhaps reflecting the effect of the natural
history of the disease. Given this, the limitations and criticisms of the
COOPERATE and ON-TARGET trials, and her high potassium, the evidence
doesn’t seem to support addition of an ARB.
Venous and arterial thromboembolism are recognised as having increased
incidence in the nephritic states with membranous nephropathy seeming to
present the highest risk of these Llach et al Am J Med 1980 Dec;69(6):819-
827. VTE is most common in the first six months following diagnosis Llach et
al and when serum albumin level is <20. For these reasons, she does not
merit anti-coagulation with warfarin.

Answer: 12
C - The most important statement is that he has no symptoms of disease
relapse and as such there is nothing specific to treat. A recent meta-analysis
Tomasson et al – ACR annual scientific meeting October 2009 found that
rising ANCA values during remission correlated with predictability of future
relapse but notes that treatment decisions based solely on ANCA results run
the risk of exposing patients to potentially harmful unnecessary therapies.
Though there was a large amount of heterogeneity in studies, they suggest
that a rise in ANCA gives a likelihood ratio of 3.39 for a future relapse.

Answer: 13
A - Although the immunology does not appear consistent with lupus, patients
with class V (or membransous lupus nephritis) often have normal complement
levels and are not ds-DNA positive. A recent randomised control trial of
treatments in membranous LN A Howard et al JASN April 20:901-911 (2009)
noted that out of 42 patients with class V LN diagnosed on renal biopsy, only
7% had low C3, 7% low C4 and 21% positive ds-DNA antibodies.
Thickened basement membrane in isolation would be consistent with MCD
but the other findings rule this out. Typically spikes are reported in idiopathic
membranous nephropathy, though general thickening can be noted. IgG and
C3 trapping, along with sub-epithelial deposits are seen in IMN, but the C1q
and tubuloreticular structures are not.

173
HIV immune complex nephritis is reported and appears much like LN with full
house immunology in the context of negative lupus type immunology (ANA
can be weakly positive) and the absence of lupoid type symptoms Haas et al
KI Vol 67 (2005), pp 1381-1390. This is a potential differential diagnosis, but
is much less common than lupus nephritis and therefore not the most likely
diagnosis.
The biopsy does not describe FSGS at all.

Answer: 14
C - Class III lupus nephritis (along with class IV) is a proliferative form and
immunosuppression is widely used. Patients with Class III LN and diffuse
glomerular involvement have a prognosis similar to class IVA which carries
the worst prognosis – treatment with immunosuppression is therefore
warranted. Najafi et al KI 2000;59(2156-2163). This precludes the addition of
irbesartan alone as the correct answer.

The evidence base for cyclophosphamide has been established well over the
years with excellent remission rates achieved. It has been shown that when
used as an additive treatment it is superior to steroids alone Gourley et al Ann
Intern Med 1996;125 549-557. Further immunosuppressive therapy than
steroids are therefore appropriate. Due to the side effect profile of
cyclophosphamide much research has looked at alternative agents.
As one may expect, MMF vs oral cyclophosphamide had an improved side
effect profile (ferwe infections, lower mortality) with no difference in the
numbers of remissions. Chan et al NEJM 2000;343:1156-1162. A further
study showed that MMF vs pulsed cyclophosphamide improved histological,
immunuological and urinary parameters with a lower side effect profile but did
not assess renal outcomes Hu W et al Chin Med J 2002;115:705-709. A
further 24 week study Ginzler et al NEJM 2005;353:2219-2228 showed MMF
to be more likely to induce complete but not partial remission.
The most recent and largest trial, the ALMS trial Appel et al JASN 20:1103-
1112 2009 failed to show any difference in efficacy or side effect profile
between MMF and pulsed iv cyclophosphamide overall. It did, however, in
subgroup analysis suggest a racial difference in responses to treatment with
non whites and non Asians responding more favourably to MMF.

174
Given the above and her young age, MMF seems the most appropriate first
line therapy.

175
Haematuria, proteinuria,

the nephrotic syndrome

& renal disease in pregnancy

ANSWERS

Answer: 1
A
This patient has preserved renal function with microscopic haematuria,
moderate proteinuria and a normal renal ultrasound consistent with a chronic
GN. The likely diagnosis is IgA nephropathy and the patient has 2 adverse
prognostic factors – hypertension and proteinuria > 1g. Management centres
on optimisation of hypertension control, with the use of ACE inhibitors
possibly in combination with ARB, and proteinuria reduction b. While
combination ACE inhibitor-ARB thereapy appears to have a more marked
effect on proteinuria reduction, there is no outcome data on renal progression
and there appears to an increased incidence of advers events.

Answer: 2
E
Transient microscopic haematuria is a common finding in young adults and in
the absence of symptoms or other evidence of kidney disease does not
warrant investigation in patients under 40. Therefore, in the absence of
persistent haematuria, further investigation is not necessary.

Answer: 3
B
Thin membrane nephropathy is the most likely diagnosis in this patient with
isolated microhamaturia and a family history of microscopic haematuria but
not ESRD.

Answer: 4
A - Likely complication of the use of herbal medicine containing aristocholic
acid. Recognised to produce severe and progressive form of IN. Ibuprofen is
possible but less likely due to infrequent use and chronicity of findings.

Answer: 5
E - This patient has clinical features suggestive of uveitis and a systemic
disease (fever, weight loss and an elevated sedimentation rate). The renal
findings suggest a tubulo-interstitial disease (mild proteinuria, leukocyturia,
impaired renal function) rather than a glomerular disease. The most likely
renal disease is the “Tubulo-Interstitial Nephritis with Uveitis Syndrome
(TINU). This disorder not uncommonly is accompanied by anaemia and low-

176
grade auto-immunity (positive antinuclear antibody) but renal biopsies do not
show immunoglobulin deposits. Evidence of systemic vasculitis is absent.

Answer: 6
C – In patients with no pre-existing kidney disease or risk factors high protein
diet by itself does not lead to proteinuria. The other conditions are associated
with proteinuria.

Answer: 7
A – Gold therapy is associated with membranous GN and not MCNS

Answer: 8
C – in cases of persistent proteinuria option A is not appropriate.
Most evidence in MN is for combination therapies. This patient would fall in
the moderate risk category and has failed to improve despite 6 six months of
RAAS inhibition. Recommended treatment are alternating monthly IV
methlypred and alternate day prednisolone followed by cyclophosphamide /
chlorambucil . Steroids alone are not recommended. Steroids and MMF is not
currently the recommended first line therapy. Steroids and Azathioprine is not
recommended for induction therapy.

Answer: 9
E - MPGN is STRONGLY associated with Hep C and cryoglobulinaemia.
FSGS is other option in heroin users but is not the correct option in v/o no
heroin use for > 5y. IgAN can be associated with liver disease but would be
less likely in the absence of cirrhosis.

Answer: 10
C - FSGS (collapsing variant) is associated with pamidronate therapy. There
is AIN is not typically associated with severe proteinuria. IgAN and MN would
be unlikely in view of the history and temporal relation to pamidronate use.
Cast nephropathy is not typically a/w severe proteinuria.

Answer: 11
C - Non-dihydropyridine (rate limiting) calcium channel blockers reduce
proteinuria over and above their anti-hypertensive effect. There is good
evidence to suggest that reduction of proteinuria below 0.5g/day leads to
better renal survival. Adding spiro may exacerbate hyperkalemia. Doxazosin
has no effect on protein excretion beyond its anti-hypertensive effect. There is
currently not enough evidence to recommend low protein diet. In a recent
study it has been shown to lead to worse outcomes.

Answer: 12
C - The patient has de novo hypertension and proteinuria after 20 weeks
gestation and therefore, by definition has pre-eclampsia.
She does not have evidence of significant heamolysis or HELLP syndrome.
Acute kidney injury complicates 2% of cases of pre-eclampsia.
Pre-eclampsia is a leading cause of maternal mortality in the UK and
worldwide. In the UK, most deaths are due to pulmonary oedema as a result
of excessive fluid resuscitation and endothelial dysfunction. Invasive

177
monitoring is usually required and should be managed in a high dependency
unit aiming for a CVP of 2 to 6 cm H2O only.
Magnesium sulphate infusion reduces the progression to eclampsia and
improves outcomes.

Answer: 13
D - Asymptomatic bacteruria in pregnancy should be treated with appropriate
antibiotics and not monitored, irrespective of symptoms as it is associated
with increased risks of overt urinary tract infection, pyelonephritis and preterm
delivery. Pyelonephritis in pregnancy can be particularly aggressive.
Bacteruria is more common in pregnancy as a result of physiological dilation
of the renal pelvis and ureters.
Quinolones may be associated with abnormal musculoskeletal development
and necrotising enterocolitis. Cephalosporins and penicillins are generally
safe during pregnancy. Gentamicin can be life-saving in pyelonephritis.
Trimethoprim should be avoided when trying to conceive and in early
pregnancy (when trying to conceive – folate antagonist) and nitrofurantoin
avoided in late pregnancy (when huffing and blowing – haemolytic anaemia in
the newborn).

Answer: 14
D - Pregnancy in patients with CKD 2 in the absence of hypertension rarely
leads to complications although the risk of pregnancy-induced hypertension
and pre-eclampsia (and hence the risk of intra-uterine growth retardation and
pre-term delivery) may be increased with renal disease. Vesicoureteric reflux
may be inherited and identified in utero or in infancy. Given the structural
anomalies associated with vesicoureteric reflux, the risk of bacteruria is
further increased and screening is advisable.

Answer: 15
B - Blood pressure control and baseline renal function are the key predictors
of successful maternal and fetal outcomes. One study has shown proteinuria
to be predictive of poor maternal outcome, when associated with
eGFR<40ml/min. Methyldopa, nifedipine, labetalol and hydralazine are
considered safe in pregnancy.
ACE inhibitors and ARBs are teratogenic in the second and third trimesters.
More recently, ACE inhibitor use in the first trimester has been shown to be
associated with malformations; ARBs in the first trimester are similarly
contraindicated by association. Statins should not be prescribed in pregnancy
(congenital anomalies). They should be stopped a month before attempted
conception.
Pregnancy with eGFR >60 ml/min is associated with negligible additional risks
in the absence of hypertension.

178
Baseline creatinine <125 μmol/l 125-180 μmol/l >180 μmol/l
IUGR 25% 40% 65%
Preterm delivery <37 weeks 30% 60% 90%
Pre-eclampsia 22% 40% 60%
Perinatal death 1% 4% 10%
Loss of 25% GFR 2% 40% 70%
Permanent loss of 25% GFR <1% 20% 50%
ESRD within 1 year <1% 2% 35%
Williams D, Davison J. Chronic disease in pregnancy. BMJ 2008;336(7637):211-215

Answer: 16
D - She has nephrotic syndrome. It is too early (by definition) for pre-
eclampsia. HUS/TTP is associated with pregnancy but there are no clinical
signs of this. Thromboembolic events are increased in pregnancy but
normally in the second or third trimester or the puerperium. IVC and renal vein
thrombosis may exacerbate proteinuria in the nephrotic syndrome but this is
not likely in the absence of pain, haematuria or impaired renal function. Lupus
nephritis may present with nephrotic syndrome but disease activity is typically
reduced during pregnancy and there are no additional clinical features. Renal
biopsy is feasible throughout pregnancy, although indications should include
potential for effective therapeutic intervention. The outcome of lupus nephritis
presenting in pregnancy is significantly worse than that of lupus nephritis if
diagnosed in advance of conception and in stable remission for > 6 months.
The major maternal risk of lupus nephritis is of mortality related to sepsis, as
well as intra-partum and early post-partum acute renal failure.

Answer: 17
D - Increased filtered bicarbonate is not matched by an equal increase in
reabsorption leading to bicarbonaturia and a metabolic acidosis. Within 6
weeks’ gestation, systemic vascular resistance falls leading to an increase in
cardiac output and circulating volume. Subsequently, renal blood flow
increases by 75% and GFR increased by 50% by 30 weeks’ gestation.
The upper limit of normal urine protein excretion increases from 150mg/d to
260mg/d in pregnancy.
Smooth muscle relaxation and the effect of the gravid uterus commonly lead
to a degree of hydronephrosis, more commonly on the right than the left. In
the absence of pain, renal dysfunction or recurrent infection this does not
require further investigation during pregnancy.

Answer: 18
C - Pregnancy outcomes following transplantation (95% success in terms of a
healthy live offspring with at most mild (<30%) decline of maternal renal
function) are vastly better than if still on dialysis assuming the following
general guidelines:
1. 1 year post transplant (some say 18-24 months)
2. Stable renal function on stable immunosuppression for > 6
months
3. Minimal proteinuria (<0.3 grams/ day, PCR < 30)
4. Blood pressure well-controlled (diastolic BP < 85 mm Hg)
5. No transplant rejection

179
6. Minimal levels of appropriate immunosuppression
“Appropriate immunosuppression” is based on observation, expert opinion
and experience rather than licenses or RCTs:

“Safe” “Not safe”


Prednisolone (<7.5mg/d) Mycophenolate
Azathioprine (<2mg/kg) Sirolimus
Tacrolimus
Cyclosporin
Nifedipine (and almost certainly ACE inhibitors
amlodipine) ARBs
Methyldopa Diuretics
Labetalol Most beta-blockers
Hydralazine Spironolactone

Success is reduced to 75% when pre-conception creatinine is >125μmol/l and


a permanent loss of graft function occurs in 15% of transplant patients.
This patient should be counselled as to the small but significant risk of the
teratogenic effects of mycophenolate and lisinopril.

Answer: 19
D - In the UK, an average sized renal unit will expect to treat one pregnant
patient on dialysis every four years. Management guidelines are therefore
based on expert opinion and experience. There is consensus that increased
dialysis frequency and duration is associated with improved pregnancy
outcomes, assuming caution is taken to avoid excess removal of (for
example) potassium and phosphate, and fluid balance is carefully tailored to
the developing pregnancy.
Haemodynamic instability during and post dialysis is believed to represent an
increased risk of placental insufficiency and fetal loss. BP should be
mponitored for 1-2 hours to document post-dialysis stability, at least for a few
dialysis sessions when pregnancy is first confirmed.
Successful pregnancy with peritoneal dialysis is reported and patients on PD
need not be converted to HD routinely. Nevertheless, fluid balance becomes
particularly tricky later in pregnancy as a result of decreased effective
peritoneal membrane availability and HD is often required. Many advocate
addition of haemodialysis to supplement solute clearance.
Erythropoetin and vitamin D analogues appear safe in pregnancy.
Many patients will elect for termination of pregnancy while on dialysis
because of their health. There is also a 90% risk of prematurity and
intrauterine growth restriction, a 75% risk of pre-eclampsia and a 50% risk of
perinatal death which many patients would find unacceptable.
Women of child bearing age on dialysis should be advised to use
contraception and wait until transplantation before conceiving if possible.

Answer: 20
E - The SCE is littered with questions about complement level. This patient
has nephrotic syndrome with depressed C3 and C4. Options a, b and d do not
lead to complement consumption. Lupus nephritis is very rare in the absence

180
of anti-dsDNA. The rash and arthralgia suggest a hunt for cryoglobulins is
required.

Answer: 21
C - He is at high risk of renovascular disease and his acute presentation may
represent renal infarction. Caridac emboli (from atrial fibrillation) or decreased
cardiac output (from myocardial infarction) may precipitate this. Differential
diagnosis includes renal stone disease. Acute glomerulonephritis is unlikely to
present so acutely with loin pain.

Answer: 22
B – This is orthostatic proteinuria.

Answer: 23
A - Pre-eclampsia is an endothelial disease. The last triennial report on
maternal mortality identified pulmonary oedema as a common cause of pre-
eclampsia related death. Fluids should be administered with caution and a
low threshold for invasive monitoring employed. Patients should be left ‘dry’ in
the acute setting. Pre-eclamptic patients are intravascularly deplete and
diuretics may worsen splanchnic hypoperfusion. A renal biopsy is not
recommended in the diagnosis of pre-eclampsia and there is no evidence of
MAHA to justify plasma exchange.

Answer: 24
A - Renal biopsy can be performed up to 24 weeks gestation in the prone
position and up to 32 weeks in the sitting position. It should be reserved for
patients in whom the results are likely to provide information to facilitate
management decisions or offer prognostic indications. Corticosteroids can
cause fetal adrenal suppression and increase the risk of maternal infection,
therefore doses and duration should be minimised if possible. Nephrotic
syndrome in early pregnancy is associated with increased rates of fetal loss,
IUGR and early delivery. Thromboprophylaxis with LMWH should be initiated
in all pregnant patients with nephrotic syndrome unless strictly
contraindicated.

Answer: 25
C - The two most predictive risk factors for progression of glomerular
diseases are the quantity of protein excreted in the urine and the extent of
tubulointerstitial disease on renal biopsy; therefore, option C is correct. The
quantity of protein excreted was not one of the choices in this question. There
is no evidence that option A is a predictive factor. Although a growing body of
evidence has implicated genetic factors in the natural history of established
glomerular diseases, there is no consistent evidence that the ACE genotype
predicts progression of glomerular diseases, particularly FSGS; therefore,
option B is not correct. Option D, the level of plasma renin activity, has not
been demonstrated to be a predictor of renal disease progression. In fact, in
most chronic kidney diseases (CKDs), plasma renin activity is suppressed,
most likely related to volume expansion. Thus, the beneficial effects of ACEIs
in slowing progression of renal disease are somewhat of a paradox. Possible
explanations have included activation of the intrarenal renin-angiotensin

181
system. There is no evidence that option E, a family history of hypertension,
predicts progression of FSGS. There is some evidence that a family history of
hypertension is a risk factor for the development of diabetic nephropathy. The
link between tubulointerstitial disease and progression may relate to
reabsorption of proteins by proximal tubular cells, with activation of these cells
and subsequent release of inflammatory mediators.
Nath K: The tubulointerstitium in progressive renal disease. Kidney Int 54: 992–994, 1998
Remuzzi G, Bertani T: Pathophysiology of progressive nephropathies. N Engl J Med 339:
1448–1456, 1998
Iwano M, Neilson EG: Mechanisms of tubulointerstitial fibrosis. Curr Opin Nephrol Hypertens
13: 279–284, 2004

182
Urinary tract infection

stones

& obstruction

ANSWERS

Answer: 1
E – Characteristic “coffin-lid” calcium oxalate crystals. Treatment includes:
maintain a high urine output; citrate; pyridoxine (enzyme coF); Oxalobacter
formigenes supplementation and liver transplant (Primary hyperoxaluria).

Answer: 2
E – This is secondary hyperoxaluria, causes include: enteric (fat
malabsorption); dietary (excess spinach, rhubarb); ascorbic acid intoxication
(metabolised to oxalate); deficiency in intestinal flora (Oxalobacter
formigenes); ethylene glycol intoxication (alcohol dehydrogenae metabolises
ethylene glycol to oxalate). Treatments listed are appropriate for other forms
of stones. He needs to start reabsorbing fat again- therefore needs operation
reversing.

Answer: 3
E – He has cytinuria, therefore needs penicillamine- don’t confuse with
cystinosis! Autosomal recessive disease causing increased excretion of
cystine, lysine, arginine and ornithine. Diagnosed by spot urinary cystine
and/or microscopy of fresh urine. Treatments include: increasing urine output,
alkali therapy, dietary sodium restriction, cysteine-binding drugs (prevent
formation of cystine)- penicillamine, tiopronin, captopril, ascorbic acid.

Answer: 4
C – He has hypercalciuria and calcium containing stones are most likely.
Nephrolithiasis: incidence 1 in 1000; recurrence rate of 50-80%. Causes
include urine supersaturation and presence/absence of stone promotors &
inhibitors (pH, citrate).
CaOx & CaPhos > CaOx > Struvite > CaPhos > Uric acid > Cystine
Investigations: urine microscopy for crystals, spot urine for cystine, 24h urine
collections.
Risk factors for calcium containing stones:
 hypercalciuria (idiopathic hypercalciuria, hypercalcaemic
hypercalciuria)
 distal RTA (hypercal, hyperphos, high ur pH, hypocitraturia)
 hypocitraturia (metab acidosis, hypokal, hypomag)
 hyperoxaluria
 hyperuricosuria

183
 hypokalaemia (intracel acidosis causing hypocitraturia)
Treatment of calcium containing stones:
 sodium restriction
 increase urine output
 normal calcium diet NO supplements
 thiazide diuretic
 potassium supplement
 citrate supplement

Answer: 5
E – Vesicoureteric reflux: VUR is a radiological diagnosis, reflux nephropathy
is the disease caused by VUR. Regresses with age, cause of 15% ESRD in
teenagers/adults; occurs more frequently in first-degree relatives-
multifactorial, likely polygenic (some families may be single gene defect).
Presents as antenatal diagnosis or UTI in childhood, look for neuropathic
bladder, post urethral valves. Investigations: renal USS- size, hydronephrosis,
hydroureter, residual volume. Cystography gold standard (MCUG or MAG3
isotope cystogram). DMSA isotope scan to look for scars and split function.
Management = manage UTI. No evidence base for surgical intervention.

Answer: 6
D – Remember: you can be obstructed without a dilated urinary tract, you can
have a dilated urinary tract without being obstructed. Renal isotope studies:
[99mTc]MAG3
 largely replaced [99mTc]DTPA
 used in dynamic renography
 excreted by tubular secretion
 split function
 diagnose obstruction
 diuresis renogram
 assess renal perfusion
 captopril renogram
99m
[ Tc]DMSA
 retained and concentrated in renal cortex
 used in static studies
 provides high resolution anatomic images
 split function
 renal scarring
 ectopic renal tissue

Answer: 7
B - Patients with type 2 diabetes have a low urine pH due to insulin resistance
– impairment of urinary ammonium excretion. Insulin stimulates ammonia
synthesis as well as the activity of the Na+//H+ exchanger in the proximal
tubule. Therefore, uric acid and calcium oxalate unlikely as urine is alkaline
these stones are unlikely to precipitate at this pH. Calcium phosphate is the
most likely as the urine is alkaline and he has a high sodium load due to him
taking sodium bicarbonate. Struvite is unlikely as there is no significant history
of urinary tract infections. He is now on allopurinol which blocks conversion of
xanthine to uric acid again making a uric acid stone unlikely. Xanthine stones

184
could form but usually only cause problems when the patient is poorly
hydrated and there is no evidence for this.

Answer: 8
D – citrate is an inhibitor of stone formation. Animal protein results in
reduction in urinary pH which would encourage precipitation of urate stones in
particular. Uric acid can form a nidus for calcium stone formation. There is
emerging evidence about the benefit of bisphosphonates in reducing calcium
stone formation and preventing bone diease in this group of patients but not
enough data available to make it routine practice.

Answer: 9
C – The incidence of oxalate stones is increased when the last 100cm of
ileum is resected. Oxalobacter formigenes colonisation is low in stone formers
compared to non-stone formers (especially following fluroquinolones).
Although therapeutic application of Oxalobacter formigenes has shown to
reduce urinary oxalate excretion it is associated with only a small decline in
urinary calcium oxalate supersaturation and stone formation. Alkalinisation is
useful only if there is an accompanying uric acid stone. Pyridoxine increases
degradation of oxalate by enhancing activity of liver enzymes. Undigested fat
promotes oxalate absorption. Also stool bicarbonate losses cause low urine
pH and citrate levels resulting in increased calcium oxalate supersaturation
and uric acid supersaturation.

Answer: 10
D - This question focuses on the treatment of asymptomatic urinary tract
infections. Treatment in pregnant women is proven beneficial. There is no
evidence to support treatment of asymptomatic urinary tract infections in
transplant recipients post-three months. Pyelonephritis in the first three
months posttransplant is associated with acute rejection and poor outcome
therefore asymptomatic urinary tract infections in the first three months post
transplant are generally treated. There is no evidence for treatment of
asymptomatic urinary tract infections in patients with diabetes or in elderly
patients Prophylactic antibiotics are controversial and again there is no clear
evidence to support their use. Even evidence in children with vesico-uereteric
reflux is mixed but they are usually used as part of the medical management
of children with recurrent of urinary tract infections complicating VUR.

Answer: 11
B - Vesicoureteric reflux: The chance of spontaneous resolution of reflux is
high in children younger than 5 years with grades I-III reflux and in children
younger than 1 year (especially boys). Even higher grades of reflux (grades
IV-V) may resolve spontaneously as long as they remain infection free.
Following surgical repair, the incidence of pyelonephritis significantly
decreases (in comparison to medical management with long-term antibiotic
therapy); however, the incidence of cystitis or renal scarring is the same
following both medical and surgical management of vesicoureteral reflux. Trial
data, including a metaanalysis of 10 trials show no difference in long term
outcome with medical or surgical treatment. Poor predictors of renal survival
are: renal scars; hypertension; elevated creatinine in childhood. Renal

185
scarring is unusual after the age of five years. Patients with reflux
nephropathy who are hypertensive are four times more likely to develop renal
failure than normotensive patients. There is some evidence that removal of a
small scarred kidney in hypertensive patients with unilateral reflux
nephropathy may improve or cure hypertension. Pregnancy in patients with
reflux nephropathy can be complicated by pre-eclampsia (24%) and
deterioration of kidney function (18%).

Answer: 12
C - This is a case of interstitial cystitis or reflex sympathetic dystrophy of the
bladder which is a type of complex pain syndrome. It is difficult to manage
and patients usually end up having a lot of pelvic surgery. Cystoscopic
features are those given in the question.

Answer: 13
C – Beware the non-hydronephrotic obstructed kidney- you don’t always have
a dilated tract with obstruction- the radiologists need to know this! Polyuria
may occur in partial obstruction due to development of a form of nephrogenic
diabetes insipidus resulting from a concentrating defect that may be mediated
by direct effects of increased tubular pressure on the function of distal tubular
cells. Renal histopathological changes that occur following urinary tract
obstruction include –
1 – 7 days: flattening of renal papillae and dilatation of distal nephron
7 – 14 days: atrophy and necrosis of collecting tubules
14 – 28 days progressive dilatation of distal and collecting tubules
> 28 days 50% decrease in medulla, thinning of cortex, proximal tubule
atrophy, glomerular changes
Renal outcome is less favourable with: increased duration of obstruction;
increased severity of obstruction; superadded infection; pre-existing renal
impairment.

Answer: 14
A - Sjogren’s is associated with RTA Type I. Distal acidification defect
associated with acidaemia and hypercalciuria. acidemia, promotes stone
formation both by increased calcium phosphate release from bone during
bone buffering of retained acid and by direct reduction of the tubular
reabsorption of these ions. The degree of hypercalciuria is roughly
proportional to the severity of the acidemia.Calcium phosphate precipitates in
alkaline urine. Histopathology reveals calcium phosphate deposits plugging
inner medullary collecting and Bellini ducts. Hyperparathyroidism may be
present with normal serum calcium in severe vitamin D deficiency but is
otherwise unlikely. RTA II is not associated with stone formation.

Answer: 15
B - all the others increase the risk of stone formation but do not themselves
precipitate as stones

Answer: 16
D - diagnosis is type II primary hyperoxaluria. Type I PHO is more severe and
is caused by AGT deficiency. Glycolate excretion is increased and as GFR

186
falls systemic oxalosis occurs. Type II the enzyme defect is the cytosolic
enzyme glyoxalate reductase/D-glycerate dehydrogenase which converts
glyoxlate to glycolate. Patients with this disorder excrete increased amounts
of L-glyceric acid as well as oxalate. In type I primary hyperoxaluria if
pyridoxine therapy fails combined liver-kidney transplantation may become
necessary. Type II does not usually progress to ESRD.

187
Dialysis

(general principles, clinical management & complications)

ANSWERS

Answer: 1
B – she passes only 100ml urine per day. APD is suitable for a high
transporter type. The nifedipine does not account for her respiratory
symptoms. The EAPOS study demonstrated that TOTAL fluid removal of
<750ml per day was associated with decreased survival.

Answer: 2
B – He is a high transporter type and would benefit from APD. Using greater
volumes may improve his cramps by greater metabolite removal but would be
less effective than APD and would not improve his fluid balance. The same is
true of non-glucose based solutions. Diuresis would be ineffective in this
case.
http://patients.uptodate.com/topic.asp?file=dialysis/5739

Answer: 3
A – The patient is likely to have EPS. There is a 15-20% probability of
developing EPS after 8 years on PD. Up to 65% of new cases occur after
stopping peritoneal dialysis. The ascitic effluent is usually an exudate. There
are associations between EPS and duration of dialysis, membrane type,
recurrent peritonitis and some drugs, including beta-blockers.
http://www.njmonline.nl/njm/getpdf.php?t=a&id=10000339
From UK EPS guidelines:-
CT scanning is recommended as the diagnostic imaging modality of choice.
However, mild peritoneal membrane changes on CT without encapsulation or
gastrointestinal dysfunction do not make the diagnosis of EPS (GRADE 1B).
• a plain abdominal x-ray may be of value in confirming or excluding the
presence of bowel obstruction. If bowel obstruction is present, there
will be a high suspicion of EPS,
• in the absence of peritoneal calcification the diagnosis should not be
made on plain film alone.
• CT scanning is probably of greatest value - widely available; greatest
reproducibility.
Reporting should be done by individuals with experience of diagnosing EPS
looking for peritoneal calcification, bowel wall thickening, bowel tethering, and
bowel dilatation

Answer: 4
C - For staph aureus and pseudomonal peritonitis the presence of an
associated tunnel infection should mandate catheter removal. Fungal
peritonitis is always an indication for early catheter removal, TB is not.

188
Answer: 5
D – The others are relative contraindications only

CONTRAINDICATIONS TO PERITONEAL DIALYSIS


ABSOLUTE RELATIVE
Known peritoneal sclerosis or loss Weight(>100kg)
of membrane function Anuria
Mentally or physically incapable Intolerance of fluid in abdomen
(but note assisted PD) (e.g. back pain, respiratory
Large irreparable hernia or other disease)
abdominal wall defects Malnutrition
Abdominal stoma(?) Recurrent diverticulitis/IBD
Peritoneal leaks
Recent intra-abdominal prosthesis
Adapted from KDOQI guidelines

Answer: 6
E - He is likely to have DDS and continuing dialysis may lead to death.
Intracranial bleeds are more common in haemodialysis patients than those on
peritoneal dialysis but stopping the heparin alone is not adequate given the
risk of DDS. Prevention requires slow low reduction of serum urea by short
low efficiency dialysis repeated frequently

DIALYSIS DYSEQUILIBRIUM SYNDROME


RISK FACTORS SYMPTOMS/SIGNS

New ESRF or missed dialysis Headache


Serum urea >60mmol/L Nausea
Older age Disorientation
Paediatric patients Restlessness
Metabolic acidosis Asterixis
Pre-existing CNS disease Blurred vision
Confusion
Seizures
Coma

Answer: 7
D – All of these components are important but the process of reverse osmosis
through a membrane is the step which ensures there is minimal risk of
contamination of the final dialysate. Individual RO machines are used for
home haemodialysis patients and for individual machines which are not
provided by a central water supply system e.g. where patients are treated on
wards or ITUs.

Answer: 8
C - The PCR is determined by measuring the inter-dialytic appearance of urea
in body fluids plus any urea lost in the urine in patients with residual renal
function. The PCR is a function of protein catabolism and reflects protein
intake only if the patient is in a steady state regarding nutrition. A target of 1.0

189
to 1.2 g/kg per day or higher is recommended by both American and
European HD guidelines. A low pre-dialysis urea in a patient with no
significant residual renal function is a marker of malnutrition

Answer: 9
C – The first thing to check is the position of the needling. Ultrasound dilution
and similar techniques which depend on producing artificial recirculation by
reversing the lines and then injecting a bolus of saline can only be performed
if there is a straight length of vein without any side branches coming off
between the needles. Long standing AVF may have multiple channels and
this may lead to marked over estimation of access flow using these
techniques.

Answer: 10
E – High flux HD allows for some MMW clearance but less so than HDF.
Though all answers have some theoretical ground, there is variable
reproducibility across clinical studies. Observational studies have suggested a
significant improvement in mortality but 3 prospective randomised
studies(Dutch, French and Italian) are currently looking at this.
http://ndt.oxfordjournals.org/cgi/reprint/gfm791v1

Answer: 11
D – The clinical picture is that of Carpal Tunnel Syndrome and joint
accumulation of ß-2 microglobulin in a long term haemodialysis patient.
• Related to accumulation of beta2 microglobulin
• Historically high prevalence in long term patients
• Improved with high flux and haemodiafiltration
• Daily nocturnal HD improves beta2M clearance over HF/HDF
• Renal transplantation most likely to lead to regression of amyloid

DIALYSIS PREVALENCE
VINTAGE (yrs) DRA (%)
<2 21
4-7 50
7-13 90
>13 100

Answer: 12
D - The high venous pressure may indicate a stenosis developing in the
outflow of her graft. Biocompatible membranes do not significantly increase
small solute clearance. Changing to larger SA dialyser may improve her URR
but not of access flow is limiting factor, a small bore needle will limit arterial
flow rates, longer hours will be effective but may not be a good lifestyle choice
for a young patient. An alternative would be higher flow rate but for this she
would need a wider bore needle and ensuring her graft is functioning properly.

Answer: 13
A - The patient could have peritoneal TB. Peritoneal fluid samples are rarely
positive on direct AFB staining. Extra-pulmonary disease is significantly more

190
common in the CKD population. An effluent sample has been taken and the
patient should be further investigated with imaging and gamma interferon
testing. Anti-TB treatment may be indicated depending on the clinical course
and these results even in the absence of a positive culture. In some reports
mortality is lower if treatment is commenced earlier rather than later.
However, anti-TB therapy is not without morbidity
http://www.nature.com/ki/journal/v70/n5/full/5001610a.html

Answer: 14
A – Both infection and steal are up to 5 times more common with PTFE grafts
over AVF. Thrombosis is also more common, though declotting is successful
in 60-95% of cases and surgical intervention may be more successful than for
AVF. The maturation is significantly shorter and a synthetic graft may be
considered as first access where a line is to be avoided but dialysis is needed
quickly. This aside, they are generally reserved for cases where AVF is
unlikely to be successful or there have been multiple failed AVF attempts.
http://www.vascular-access.info/public/JVA/Article/Article.aspx?UidArticle=4D7ACE81-D470-
4789-9F85-A8360254AF3B

Answer: 15
E – Cardiovascular death is the most common cause of death in dialysis
patients and sudden cardiac death accounts for more than 60% of these
events. Though coronary artery disease is the most common post-mortem
finding in such cases (>40%), pump failure due to dilated cardiomyopathy and
associated / independent LVH. Electrolyte abnormalities may also contribute.
Though this patient would be likely to suffer hyperkalaemia, haemodialysis
patients are most at risk of arrhythmic events associated with post dialysis
hypokalaemia. The patient’s headache may indicate hypertension and though
diabetic “dead-in-bed” syndrome is recognised, it is rare and most commonly
seen in type 1.
http://www.pdiconnect.com/cgi/reprint/29/1/58

Answer: 16
E - Several studies show that sudden death occurs more frequently:
immediately after HD; during HD, towards the end of 72hr break

Bleyer AJ, Hartman J, Brannon PC, et al. Characteristics of sudden death in hemodialysis
patients. Kidney international. 2006;69(12):2268-73

191
AKI, acute renal replacement therapy

& plasmapharesis

ANSWERS

Answer: 1
C - Numerous studies have investigated the effects of ‘low dose’ dopamine on
outcome in AKI. The majority of these studies have been poorly designed and
underpowered. In the larger, adequately designed studies, no benefit in renal
outcome or survival has been demonstrated. However, no published study
has been sufficiently large to exclude a small benefit in renal outcome.
In one small study involving only 40 patients, low dose dopamine was
demonstrated to increase renal vascular resistance in patients with AKI,
whereas it reduced renal vascular resistance in patients without AKI. No renal
outcome data were given.
Studies in AKI patients have also failed to show potential benefit of dopamine
on renal medullary perfusion and oxygenation.

Lauschke A, Teichgraber UKM, Frei U, et al: ‘Low-dose’ dopamine worsens renal perfusion in
patients with acute renal failure. Kidney International 69:1669-1674, 2006
Bellomo R, et al: Low-dose dopamine in patients with early renal dysfunction: a
placebocontrolled randomized trial. Australian and New Zealand Intensive Care Society
(ANZICS) Clinical Trials Group. Lancet 356: 2139-2143, 2000

Answer: 2
D - A systematic review of furosemide to prevent or treat AKI has found that
high dose furosemide (1-3.4 grams daily) was associated with an increased
risk of temporary deafness and tinnitus (RR 3.97; 95% CI 1.00-15.78; P =
0.05). The review also found that there was no reduction in in-hospital
mortality or reduction in duration of renal replacement therapy. Part of the
previous rationale for administering furosemide was to improve urine output,
but no difference in mortality or renal recovery has been shown between
oliguric and non-oliguric AKI. Experimental evidence has shown that
furosemide can increase medullary oxygenation by decreasing oxygen
demand from the thick ascending loop of Henle but this has not been
translated into decreased severity of clinical AKI.
Ho KM, Sheridan DJ: Meta-analysis of frusemide to prevent or treat acute renal failure. BMJ
333:420-426, 2006
Cantarovich N, Rangoonwala B, Lorenz H, et al: High-dose furosemide for established ARF:
a prospective, randomized, double-blind, placebo-controlled, multicenter trial. Am J Kid Dis
44:402-409, 2004

Answer: 3
E - Tenofovir is a nucleoside analogue of adenosine-5’-monophosphate used
as an inhibitor of viral reverse transcriptase in the therapy of HIV infection. It
is well described that tenofovir can cause acute kidney injury in association

192
with severe lactic acidosis. The toxicity is thought to be mediated through
mitochondrial DNA depletion and is potentiated by concomitant therapy with
stavudine or didanosine.
Allergic interstitial nephritis is a possible cause of AKI here, but urinalysis
would be expected to more strongly positive for blood and protein and
tenofovir is statistically more likely.
Perazella MA. Acute renal failure in HIV-infected patients: A brief review of common causes.
Am J Med Sci 319:385-391; 2000.
Coca S, Perazella MA. Rapid communication: acute renal failure associated with tenofovir:
evidence of drug induced nephrotoxicity. Am J Med Sci 324:342-4; 2002.

Answer: 4
E - There is evidence that pre-hydration with isotonic crystalloid can reduce
the risk of contrast induced nephropathy (CIN). However, at present there is
no clear evidence that isotonic sodium bicarbonate offers greater
renoprotection than isotonic saline. Arguably there is weak evidence of a
benefit of bicarbonate over saline, but there have been no adequately
powered head to head studies.
Similarly, there is no clear consensus from the numerous studies which have
investigated N-acetyl cysteine as a renoprotective agent in CIN. It is often
used because it is cheap, with no evidence of detrimental effect.
Mannitol alone has been shown to increase the risk of contrast nephropathy
when compared to intravenous fluids.
Navaneethan SD, Singh S, Appasamy S, Wing RE, Sehgal AR:Sodium bicarbonate therapy
for prevention of contrast-induced nephropathy:A systematic review and meta-analysis. Am J
Kidney Dis November 21, 2008
Merten GJ, Burgess WP, Gray LV, Holleman JH, Roush TS, Kowalchuk GJ, Bersin RM, Van
Moore A,Simonton CA, 3rd, Rittase RA, Norton HJ, Kennedy TP: Prevention of contrast-
induced nephropathy with sodium bicarbonate: a randomized controlled trial. JAMA
291:2328-2334, 2004
Solomon R, Werner C, Mann D, D'Elia J, Silva P. Effects of saline, mannitol, and furosemide
to preventacute decreases in renal function induced by radiocontrast agents. N Engl J Med
331:1416-20; 1994.Trivedi HS, Moore H, Nasr S, et al. A Randomized Prospective Trial to
Assess the Role of Saline Hydrationon the Development of Contrast Nephrotoxicity. Nephron
93:C29-C34; 2003.
Mueller C, Buerkle G, Buettner HJ, et al. Prevention of contrast media-associated
nephropathy: randomized comparison of 2 hydration regimens in 1620 patients undergoing
coronary angioplasty. Arch Int Med 162:329-36; 2002.
Kelly AM, Dwamena B, Cronin P, Bernstein SJ, Carlos RC: Metaanalysis:Effectiveness of
drugs for preventing contrast-induced nephropathy.Ann Intern Med 148: 284–294, 2008
Gonzales DA, Norsworthy KJ, Kern SJ, Banks S, Sieving PC, Star RA, Natanson C, Danner
RL: A meta-analysis of N-acetylcysteine in contrast-induced nephrotoxicity: Unsupervised
clustering to resolve heterogeneity. BMC Med 5: 32, 2007

Answer: 5
B - In this patient with hepatorenal syndrome (HRS) due to chronic hepatitis C
infection, a trial of therapy with terlipressin (a vasopressin analogue) should
be initiated, as this is associated with sustained improvement in renal function
Furthermore, normalization of renal function with terlipressin prior to
transplantation has resulted in similar post-transplant outcomes as those with
normal renal function. A combined liver-kidney transplantation is not indicated
here as the majority of patients will recover renal function following successful
liver transplantation. Hyperoncotic albumin has been shown to be of benefit
in the prevention and treatment of AKI only in patients with liver disease

193
undergoing large volume paracentesis and, in conjunction with antibiotic
therapy, in patients with spontaneous bacterial peritonitis.
Gines P, Cardenas A, Arroyo V, Rodes J: Management of cirrhosis and ascites. N Engl J Med
350:1646-1654, 2004
Restuccia T, Ortega R, Guevara M, Gines P, Alessandria C, Ozdogan O, Navasa M, Rimola
A, Garcia- Valdecasas JC, Arroyo V, Rodes J: Effects of treatment of hepatorenal syndrome
before transplantation on post transplantation outcome. A case-control study. J Hepatol
40:140-146, 2004
Wong LP, Blackley MP, Andreoni KA, Chin H, Falk RJ, Klemmer PJ. Survival of liver
transplant candidates with acute renal failure receiving renal replacement therapy. Kidney Int
68:362-370; 2005.

Answer: 6
C - Early and vigorous administration of isotonic crystalloid solutions is
essential as primary intervention for the prevention of progressive acute
kidney injury in patients who have sustained crush injuries. N-acetylcysteine,
low-dose dopamine and prophylactic haemofiltration have not been shown to
be of benefit in AKI secondary to rhabodomyolysis.
Gunal AI, Celiker H, Dogukan A, et al. Early and vigorous fluid resuscitation prevents acute
renal failure in the crush victims of catastrophic earthquakes. J Am Soc Nephrol.15:1862-7;
2004
Brown CV, Rhee P, Chan L, Evans K, Demetriades D, Velmahos GC. Preventing renal failure
in patients with rhabdomyolysis: do bicarbonate and mannitol make a difference? J Trauma
56:1191-6; 2004

Answer: 7
A - In about 75% of patients who survive to hospital discharge, their health
status remains good to excellent. Overall in-hospital mortality in critically ill
patients with AKI needing renal replacement therapy is high with short-term
mortality rates of 40 to 70%. Long-term outcome data are limited, but in those
patients who survive to hospital discharge about 60% have near normal renal
function with about 10% need long term renal replacement therapy. Of
patients who survive to hospital discharge, 6-month survival is approximately
70% and 5-year survival is approximately 50%.
Gopal I, Bhonagiri S, Ronco C, Bellomo R. Out of hospital outcome and quality of life in
survivors of combined acute multiple organ and renal failure treated with continuous
venovenous hemofiltration/hemodiafiltration. Intensive care Med 23:766-772; 1997.
Morgera S, Kraft AK, Siebert G, Luft FC, Neumayer HH. Long-term outcomes in acute renal
failure patients treated with continuous renal replacement therapies. Am J Kidney Dis.40:275-
9; 2002.
Salmanullah M, Sawyer R, Hise MK. The effects of acute renal failure on long-term renal
function. Ren Fail.25:267-76; 2003.

Answer: 8
D - Multiple randomized controlled trials have been published comparing
survival in patients with AKI treated with intermittent haemodialysis and
CVVH, but none has demonstrated clear survival benefit of one modality over
the other. Confounding factors in these studies have included differences in
case-mix between the two groups and difficulties in comparing delivered dose
of therapy. No satisfactory studies have been performed comparing survival
using “hybrid” modalities (e.g. SLED) with intermittent haemodialysis or
continuous renal replacement therapy.

194
Answer: 9
B - The case described includes the ‘classic triad’ of cholesterol emboli
following a vascular procedure: acute kidney injury, livedo reticularis and
eosinophilia. Other organs can be involved (e.g. gut infarction, ischaemic
stroke, adult respiratory distress syndrome).
Interstitial nephritis can cause AKI in association with eosinophilia and might
be associated with rash, but is not characteristically associated with livedo
reticularis.

Answer: 10
E - The most likely diagnosis is tumour lysis syndrome, which can occur
usually 48-72h after initiation of treatment of malignancy. It is most likely to
occur in association with treatment of bulky and rapidly proliferating tumours
such as high grade NHL and acute leukaemias. It is characterized by
hyperkalaemia (one of the earliest manifestations), hyperphosphataemia,
hyperuricaemia and hypocalcaemia.
In this case, LDH is likely to be elevated, but this could simply reflect the
tumour load and would likely be elevated pre-treatment. Uric acid crystals
might form in the urinary tract but would not necessarily be evident on a plain
radiograph.

Answer: 11
C - See also explanation for question 7. The ATN study (Palevsky et al 2008)
did not demonstrate any benefit of ‘intensive’ renal replacement therapy
versus less intensive therapy. This study incorporated both intermittent and
continuous therapies in each arm, with the latter being used for less
haemodynamically stable patients, if considered clinically appropriate. It was
an important study in that it, on balance, disagreed with conclusions from a
previous well cited study (Ronco et al 2000) which had suggested a benefit of
higher dose CVVH over lower dose, and also a previous study which had
suggested benefit of daily over alternate day haemodialysis in critically ill
patients with AKI (Schiffl et al 2002). Mortality associated with need for renal
replacement in the critical care department is known to be high, particularly
with multiple organ failure. Figures vary widely between studies, but 50% 60
day mortality is within most published estimated ranges.
Palevsky PM, Zhang JH, O’Connor TZ, Chertow GM, Crowley ST,Choudhury D, Finkel K,
Kellum JA, Paganini E, Schein RM, Smith MW, Swanson KM, Thompson BT, Vijayan A,
Watnick S, Star RA,Peduzzi P: Intensity of renal support in critically ill patients with
acute kidney injury. N Engl J Med 359: 7–20, 2008
Ronco C, Bellomo R, Homel P, Brendolan A, Dan M, Piccinni P, La Greca G: Effects of
different doses in continuous veno-venous haemofiltration on outcomes of acute renal failure:
A prospective randomised trial. Lancet 356: 26–30, 2000
Schiffl H, Lang SM, Fischer R: Daily hemodialysis and the outcome of acute renal failure. N
Engl J Med 346: 305–310, 2002

Answer: 12
D - See also answer to question 6. This is controversial! There is evidence
that appropriate pre-hydration with crystalloid is renoprotective. The difference
between isotonic saline and isotonic bicarbonate for prehydration is
debatable. The safer answer would probably be normal saline, but on current
evidence this is an unreasonable question! In one study looking at the use of
IV saline the overall incidence of contrast-induced nephropathy was 1.4

195
percent, and was significantly lower in patients given isotonic saline (0.7
versus 2.0 percent). The benefit with isotonic saline was more pronounced in
diabetic patients (0 versus 5.5 percent) and those given more than 250 mL of
contrast (0 versus 3 percent). Mueller C, Buerkle G, Buettner HJ et al.
Prevention of contrast media-associated nephropathy: randomized
comparison of 2 hydration regimens in 1620 patients undergoing coronary
angioplasty. Arch Intern Med 2002;162:329-36.
The use of sodium bicarbonate again has conflicting evidence. However, a
meta-analysis of 2290 patients showed no benefit in larger trials but only in
smaller trials that were characterised by marked heterogeneity (Brar SS,
Hiremath S, Dangas G et al. Sodium bicarbonate for the prevention of
contrast induced-acute kidney injury: a systematic review and meta-analysis.
Clin J Am Soc Nephrol. 2009;4:1584-92).
One proposed method of protecting high-risk patients from contrast-induced
acute renal failure is removal of the contrast agent using prophylactic dialysis.
The efficacy of this approach was assessed in a 2006 meta-analysis of eight
studies (six randomized) which included 412 patients (with a mean baseline
serum creatinine ranging from 2.5 to 4.0 mg/dL [221 to 354 micromol/L]) who
were treated with haemodialysis (six studies) or continuous dialysis modalities
(two studies) compared to usual medical therapy alone (Cruz DN, Perazella
MA, Bellomo R et al. Extracorporeal blood purification therapies for prevention
of radiocontrast-induced nephropathy: a systematic review. Am J Kidney Dis.
2006;48:361-71). There was no benefit and a suggestion of possible harm
with haemodialysis.
LOW osmolar contrast agents have been shown to reduce the risk of AKI,
compared with high osmolar agents, and are now administered for the
majority of radiologic procedures using intravascular contrast media.
Acetylcysteine is a thiol compound with antioxidant and vasodilatory
properties. Although not well understood, a possible mechanism of benefit in
contrast-induced nephropathy involves minimizing both vasoconstriction and
oxygen free radical generation after radiocontrast agent administration.
Numerous prospective trials examining the prophylactic effect of
acetylcysteine in contrast nephropathy have been performed, with substantial
inconsistency in reported results.

Answer: 13
C - Loop diuretics have no impact on outcome in AKI. In the context of early
rhabdomyolysis, loop diuretics may worsen the already existing trend for
hypocalcaemia, since they induce calciuria and may increase the risk of cast
formation (Sever, MS, Vanholder, R, Lameire, N. Management of crush-
related injuries after disasters. N Engl J Med 2006; 354:1052)
Experimental studies suggested that mannitol might be protective primarily by
causing a diuresis, which minimizes intratubular haem pigment deposition and
cast formation. However, experimental studies showed no amelioration of
proximal tubular necrosis with mannitol, and mannitol may cause
hyperosmolality and other complications (Zager RA. Combined mannitol and
deferoxamine therapy for myohemoglobinuric renal injury and oxidant tubular
stress. Mechanistic and therapeutic implications. J Clin Invest 1992;90:711-9).
The use of sodium bicarbonate to achieve a forced alkaline diuresis may
prevent haem-protein precipitation with Tamm-Horsfall protein, intratubular

196
pigment cast formation, and uric acid precipitation. However, there is no clear
clinical evidence that an alkaline diuresis is more effective than a saline
diuresis in preventing acute kidney injury, as no direct comparative trial has
been performed. The best data in support of an alkaline diuresis are derived
from uncontrolled case series.
RRT is initiated for the usual indications. There is no benefit in its use
prophylactically.

Answer: 14
D - See also answer to question 11. Survival — The two principal outcomes
that have been examined with CRRT and IHD are patient survival and
recovery of renal function. A paucity of evidence exists. However, current data
suggest that survival and recovery of renal function are similar with both
CRRT and IHD. The majority of studies comparing CRRT and IHD have been
observational or retrospective case series. After adjustment for severity of
illness, there appears to be no survival benefit associated with CRRT (Guerin
C, Girard R, Selli JM, Ayzac L. Intermittent versus continuous renal
replacement therapy for acute renal failure in intensive care units: results from
a multicenter prospective epidemiological survey. Intensive Care Med.
2002;28:1411-8). Recovery of renal function — Recovery of kidney function
appears to be the same with CRRT and IHD. Although some studies report
better recovery with CRRT, these reports only evaluated renal recovery in
patients who survived, thereby failing to account for mortality differences
between groups. When the analysis combined mortality and nonrecovery of
renal function, both groups show similar recovery of function (Palevsky PM.
Dialysis modality and dosing strategy in acute renal failure. Semin Dial.
2006;19:165-70). Randomized studies have also found no such benefit with
CRRT (Uehlinger DE; Jakob SM; Ferrari P et al. Comparison of continuous
and intermittent renal replacement therapy for acute renal failure. Nephrol Dial
Transplant 2005;20:1630-7).

Answer: 15
A - Diagnosis of acute interstitial nephritis described in answer A. B is ATN; C
is hypertension; D is IgA nephropathy and E is diabetes.

Answer: 16
A - Haemodialysis is the best method to remove rapidly both toxic acid
metabolites and parent alcohols and it plays a fundamental role in treating
severely poisoned patients. Haemofiltration is less effective and should be
used only when dialysis is unavailable.
Haemodialysis can be avoided by the prompt administration of antidotal
therapy (fomepizole or ethanol - Barceloux DG, Krenzelok EP, Olson K,
Watson W. American Academy of Clinical Toxicology Practice Guidelines on
the Treatment of Ethylene Glycol Poisoning. J Toxicol Clin Toxicol
1999;37:537-60) but in this case the patient is acidotic with evidence of renal
failure and so these therapies alone would be insufficient.
There is little, if any, role for gastrointestinal decontamination with charcoal in
methanol or ethylene glycol poisoning, as these simple alcohols are rapidly
absorbed.

197
Answer: 17
E - RRT dose has been addressed in a number of trials. The largest RCT
was of 1124 patients treated with IHD, CRRT, or SLED based upon
haemodynamic status (The VA/NIH Acute Renal Failure Trial Network.
Intensity of renal support in critically ill patients with acute kidney injury. N
Engl J Med 2008; 35). Patients randomly assigned to one of two dosing arms:
• Intensive therapy: Haemodialysis and SLED were given six times per
week and a target Kt/V of 1.2 to 1.4 per treatment, while CRRT was
provided with an effluent flow rate of 35 mL/kg/h.
• Less intensive therapy: Haemodialysis and SLED were given three
times per week, while CRRT was provided with a flow rate of 20 mL/kg
per hour.
The death rate at day 60 was the same for both groups (53.6% with intensive
therapy and 51.5% with less intensive therapy). In addition, the duration of
renal replacement therapy and the rate of recovery of kidney function or
nonrenal organ failure were similar for both treatment arms. The group that
received intensive therapy had an increased number of hypotensive episodes.
Thus, more intensive renal support beyond that obtained with a standard
thrice-weekly regimen (with a target Kt/V of 1.2 to 1.4 per treatment) or
standard CRRT (with an effluent flow rate of 20 mL/kg/h) does not appear to
improve clinical outcomes. This conclusion conflicts with those of previous
studies and the subject remains controversial. It is also possible that intensive
treatment increases the risk of some complications, such as haemodynamic
instability, but mortality and renal outcomes do not appear to be worse.

Answer: 18
C - The absolute indication for acute PD is the need for dialysis and the
inability to perform any other renal replacement technique. Given the possible
advantages of PD in the acute setting, other relative indications in adults
include:
• Haemodynamically unstable patients
• The presence of a bleeding diathesis or haemorrhagic conditions
• Difficulty in obtaining vascular access
• Removal of high molecular weight toxins (>10 kD)
• Clinically significant hypothermia and hyperthermia
• Heart failure refractory to medical management
Relative contraindications to acute PD:
• Recent abdominal and/or cardiothoracic surgery
• Diaphragmatic peritoneal-pleural connections
• Severe respiratory failure
• Life-threatening hyperkalaemia
• Extremely high catabolism
• Severe volume overload in a patient not on a ventilator
• Severe gastroesophageal reflux disease
• Low peritoneal clearance
• Faecal or fungal peritonitis
• Abdominal wall cellulitis
• Acute renal failure in pregnancy
Taken from: Nolph, KD. Peritoneal dialysis for acute renal failure. ASAIO
Trans 1988;34:54.

198
Transplant Medicine

(pre-transplant, acute and chronic)

ANSWERS

Answer: 1
C - Extended criteria donors how been shown to have an excess mortality in
the perioperative period with cumulative survival not equalling that of standard
therapy until 3.5 years post tranplant (JAMA. 2005 Dec 7;294(21):2726-33.)
A live donor who was a poor match would not be superior to a well matched
cadaveric transplant.
Survival rates in the first year post transplant are comparable to those for both
haemodialysis and peritoneal dialysis (Kidney Int 2002 Oct;62(4):1423-30).
Studies comparing survival between dialysis patients and transplanted
patients are based on a standard dialysis prescription (J Am Soc Nephrol
1998 Nov;9(11):2135-41)

Answer: 2
C - HLA DR and B antigens are the most important with DR having the
greatest effect (Transplantation 1987 May;43(5):669-74). Maximal effect of DR
and B mismatching has it’s main effect at six months post transplant and two
years post transplant respectively (Dialysis and Transplantation. Owen,
Pereira, Sayegh (Eds), WB Saunders, Philadelphia 2000. p. 504)

Answer: 3
A - This is too late for a hyperacute antibody mediated rejection (which would
be confirmed by biopsy and C4d staining). The surgeon would often be the
first to know of this when the kidney becomes mottled and fails to produce
any urine.
Thrombosis of either the renal artery or vein presents with an abrupt reduction
in urine volume (Semin Dial. 2005 Nov-Dec;18(6):505-10). Venous
thrombosis (but usually not arterial) tends to have a swollen tender allograft.
There are uncontrolled studies showing that tacrolimus may be effective
rescue therapy for acute rejection unresponsive to corticosteroids and ATG
(Transplantation 1994 Mar 27;57(6):860-5).
Postischaemic acute tubular necrosis is the main cause of delayed graft
function. The risk increases with cold ischaemia time and with ciclosporin
induction therapy (Nephron Clin Pract 2005;101(2):c65-71. Epub 2005 Jun 7).
Plasmapharesis has been used in three settings – 1; treating steroid resistant
acute rejection; 2; preparing recipients for ABO incompatible organs; 3;
reducing the tire of HLA antibodies in highly sensitised patients.

Answer: 4
A - There is only limited information regarding basiliximab, with follow up
periods of only twelve months. There is a suggestion that in diabetic patients

199
it may improve graft survival at one year, a trend not seen in non-diabetic
patients (Transplantation 2000 Sep 15;70(5):784-90).
Tacrolimus and ciclosporin have similar side effect profiles. Nephrotoxicity is
dose related (Clin Transpl 13 (Suppl 1):48, 1999).
The use of mycophenolate in conjunction with sirolimus initially seemed to be
as effective as MMF and ciclosporin (Transplantation 74:1070,2002) but a
large multicentre study did not confirm this and had to be stopped due to high
incidence of acute rejection. MMF and sirolimus have an overlapping side
effect profile which further argues against their combined use.
In studies, both preparations of MMF have had similar side effect profiles.

Answer: 5
E - As above, tacrolimus and ciclosporin have very similar side effect profiles.
Safety data has not been established for the use of pamidronate in renal
impairment. In GFR of over 35ml/min aledronate can be used with no dose
modification.
Several studies have shown that bisphosphonates are superior to vitamin D
analogues in preventing loss of bone density (N Engl J Med. 2006 Aug
17;355(7):675-84). This effect has been confirmed in impaired renal function with
GFR as low as 30ml/min (Nephrol Dial Transplant. 2007 Jun;22(6):1593-600.
Epub 2006 Oct 13).
Bone density usually improves after discontinuation of glucocorticoid therapy
(J Clin Endocrinol Metab 1995 Oct;80(10):2859-65). Risk of fracture declines
rapidly in the first year off therapy (J Bone Miner Res 2000 Jun;15(6):993-
1000)

Answer: 6
B

Answer: 7
A - Calcineurin inhibitors improve graft survival so they are normally
continued, albeit with close monitoring of drug level and renal function (Lancet
354:1147,1999).
There is a consensus view that conception is safe after twelve months if there
is good graft function and there have been no episodes of rejection (Am J
Tranplant 5:1592,2005).
Combined oral contraceptives commonly cause a mild increase in blood
pressure within the normal range, but overt hypertension can occur. Relative
risk of hypertension compared to women who have never used the combined
oral contraceptive is 1.8 for current users and 1.2 for previous users
(Circulation 1996 Aug 1;94(3):483-9).
Barrier contraception remains the safest method.

Answer: 8
A - Urinary tract infection is the most common bacterial infection post
transplant and there is an increased incidence with ureteric stenting
(Cochrane Database Syst Rev 2005 Oct 19;4:CD004925). Given this, there
is no need to alter his immunosuppression.
Traditionally infections occurring more than three to six months after
successful transplant are considered less significant. This has occurred at

200
ten weeks so should definitely be taken seriously (Transplantation 1988
Jan;45(1):45-52).
Transplant patients with a UTI are not managed differently from the general
population. He clearly does not need iv therapy, but equally a single dose of
antibiotic would be inadequate.

Answer: 9
E

Answer: 10
B - Staining for C4d can be performed on frozen sections using a monoclonal
antibody or on formalin-fixed, paraffin tissue sections using a polyclonal
antibody. Both techniques will detect C4d in the peritubular capillaries, but
the polyclonal technique may fail to detect C4d in the mesangium and GBM.
As previously, plasmapharesis has been used in three settings – 1; treating
steroid resistant acute rejection; 2; preparing recipients for ABO incompatible
organs; 3; reducing the tire of HLA antibodies in highly sensitised patients.
This case does not meet those criteria.
Answers

Answer: 11
B - A large meta-analysis concluded that there as an average 5mmHg rise in
BP in donors in a 5 - 10 year period which could not be attributed to aging
alone (Boudville et al). A large retrospective analysis has estimated the
incidence of ESRD in donors as 0.5%. This is not higher than the non-donor
population.
There is a small increase in urinary protein excretion and reduction in GFR
which does not worsen.
There is no evidence to suggest higher longer term mortality in donors. In fact
there is evidence to the contrary, although there is a selection bias.

Answer: 12
B - Life expectancy is better with transplantion compared to being on dialysis.
There is no evidence to suggest that weight loss reduces the risk of graft
failure. There is increased risk of post-operative surgical complications but no
increased risk of graft loss in obese (?moderately obese) recepients.

Answer: 13
B - Early steroid withdrawal has not been shown to reduce the incidence of
NODAT when compared to low dose steroid. However, there is possibly
increased risk of rejection in steroid free/ early steroid withdrawal regimens.

Answer: 14
C - Reduction in the dose of anti-metabolite is thought to be the most
important intial step. Cidofovir is nephrotoxic and is generally reserved as a
second line. There is currently not enough evidence for
Leflunomide/ciprofloxacin to be used as first line therapy. Moreover, there is
risk of haemolysis and TMA with Leflunamide.

201
Answer: 15
D - Collapsing variant is more likely to cause de-novo disease. Non-familial
forms have a higher risk of recurrence after transplantation. Recurrent FSGS
can lead to graft failure. CAN is the commonest cause of proteinuria post-
transplantation.
Plasma exchange has been shown to induce remission of proteinuria in post
transplant recurrent FSGS (Gohh et al).

Answer: 16
B - Sirolimus associated pneumonitis is the most likely diagnosis. Treatment
is discontinuation of sirolimus and this usually results in complete resolution of
pneumonitis.

Answer: 17
E - The patient has vascular rejection. This does not usually respond to high
dose steroids.
OKT3 is associated with significantly more adverse incidents.

Answer: 18
E - Reduction in CNI dose has been shown to improve outcomes in patients
with CAN. Low dose tac and MMF has been shown to be a particularly good
regime (SYMPHONY trial). There is no evidence for increasing tac dose.

Answer: 19
C - MMF is associated with significant risk of malformations.
Aza/CnIs/steroids are generally considered to be safe. Aspirin lowers the risk
of pre-eclampsia in high risk pregnancies.

Answer: 20
B - Kaposi sarcoma post-transplantation responds very well to conversion
from CnI to mTOR inhibitors. Reduction in dose of anti-metabolites is
generally recommended.

202
Chronic Kidney Disease

(anaemia, bone disease, drug prescribing,

reno & cardiovascular disease,

hyperlipidaemias & nutrition)

ANSWERS

Answer: 1
C - The most likely outcome is improved BP control. Several studies have
suggested the responsive to antihypertensive therapy improves in treatment
resistance hypertension mainly in those individuals with bilateral renal artery
disease. Less than 30% of patients show and improvement in renal function,
about 50% show no change or stabilisation and about a fifth show an
improvement in GFR. Renal revascularization for atherosclerotic disease has
not been shown to improve survival.
Webster J, Marshall F, Abdalla M, Dominiczak A, Edwards R, Isles CG, Loose H, Main J,
Padfield P, Russell IT, Walker B, Watson M: Randomised comparison of percutaneous
angioplasty vs continued medical therapy for hypertensive patients with atheromatous renal
artery stenosis. J Hum Hypertens 12:329-335, 1998
van Jaarsveld BC, Krijnen P, Pieterman H, Derkx FHM, Deinum J, Postma CT, Dees A,
Woittiez AJJ, Bartelink AKM, Man in't Veld AJ, Schalekamp MADH, for the Dutch Renal
Artery Stenosis Intervention Cooperative Study Group: The effect of balloon angioplasty on
hypertension in atherosclerotic renal-artery stenosis. N Engl J Med 342:1007-1014, 2000
Garovic V, Textor SC: Renovascular hypertension and ischemic nephropathy. Circulation
112:1362-1374, 2005

Answer: 2
E - The rise of plasma creatinine typically begins three to five days after the
start of an ACE inhibitor and thus it is suggested to check renal function at
this time in patients thought to be at risk. A meta-analysis of randomized
controlled trials evaluating this concern found a strong association between
acute increases in serum creatinine of up to 30% that stabilize within the first
two months of ACE-inhibitor therapy and long-term preservation of renal
function. There is however general concensus that a dose reduction or
withdrawal of ACE inhibitor therapy should there be a rise in serum creatinine
>30% from baseline in the first two months of medication use.
Kidney Disease Outcomes Quality Initiative (K/DOQI). K/DOQI clinical practice guidelines on
hypertension and antihypertensive agents in chronic kidney disease. Am J Kidney Dis 43[5
Supp1]:S1; 2004
Bakris GL, Weir MR. Angiotensin-converting enzyme inhibitor-associated elevations in serum
creatinine: is this a cause for concern? Arch Intern Med 160:685-693; 2000

Answer: 3
B – With this degree of anaemia, IV iron is likely to be of greater therapeutic
benefit. EPO should start with, rather than before, iron therapy. Though GI

203
investigations are appropriate, treatment for renal anaemia should not be
delayed.
Nephrol Dial Transplant (2001) 16: 459-468

Answer: 4
D – Inflammatory markers improve on statin therapy though a direct
therapeutic benefit has not yet been proven. Statins improve survival by
reducing cardiovascular events in the general population but this is not proven
in advanced renal disease. Dyslipidaemia is higher in the CKD population
than the general population, outwith those patients with nephritic syndrome. It
is particularly common inpatients on PD compared to haemodialysis and is
also associated with uraemia.
Nephrol Dial Transplant (2001) 16: 459-468

Answer: 5
C – Hypotension is an indicator of poor cardiac function in the renal
population and carries a grave prognosis. A nocturnal dip of >10% is
associated with better outcome than in “non-dippers”.
Nephrol Dial Transplant (2001) 16: 459-468

Answer: 6
C – Vascular calcification is not universal in CKD 5, though is very common,
particularly in diabetic patients. It is likely to progress regardless of treatment.
Calciuim intake should be kept to a minimum in this setting though in early
CKD, control of hyperphosphataemia is most valuable.
Nephrol Dial Transplant (2001) 16: 459-468

Answer: 7
C - This patient has developed skin necrosis associated with calcific
vasculopathy. This syndrome is called calciphylaxis. Hyperphosphatemia
plays a role but the elevation of serum phosphate is typically persistent.
Hyperparathyroidism is frequently found associated with calciphylaxis
Vasculitis and atheroembolic disease are important entities in the differential
diagnosis of calciphylaxis but are less likely in this patient.
Nephrol Dial Transplant (2001) 16: 459-468

Answer: 8
B

Answer: 9
A - This patient has responded to treatment with an ACEI with a decrease in
his BP and urinary protein excretion; however, his serum creatinine has
increased from 244 to 286µmol/L. This is an approximately 17% increase in
serum creatinine. ACEIs are commonly associated with an increase in
creatinine after initiation of therapy, as a result of a preferential effect on
dilating the efferent arteriole. If the increase in serum creatinine is ≤30%, then
the ACEI can be continued with a recheck of serum creatinine; therefore,
option C is the correct answer. Option A is incorrect for the reasons just
mentioned. In patients with bilateral renal artery disease, serum creatinine
can certainly go up on an ACEI, but it is usually more than a 30% increase,

204
and it tends to be persistent. Option B is incorrect because it is unlikely that
the loop diuretic is causing the increase in serum creatinine in the absence of
evidence of volume depletion. Option D is incorrect because an angiotensin
receptor blocker and an ACEI both are likely to have a similar effect on an
increase in creatinine. Option E is incorrect because there is no evidence that
steroids and cyclophosphamide are indicated in the treatment of this disease.
Bakris GL, Weir MR: Angiotensin-converting enzyme inhibitor-associated elevations in serum
creatinine:Is this a cause for concern? Arch Intern Med 160: 685–693, 2000
Taal MW, Brenner BM: Evolving strategies for renoprotection: Non-diabetic chronic renal
disease. CurrOpin Nephrol Hypertens 10: 523–531, 2001
Meyrier A: An update on the treatment options for focal segmental glomerulosclerosis. Expert
Opin Pharmacother 10: 615–628, 2009
Ryan MJ, Tuttle KR: Elevations in serum creatinine with RAAS blockade: Why isn’t it a sign of
kidney injury? Curr Opin Nephrol Hypertens 17: 443–449, 2008

Answer: 10
D - In this patient with CKD and coronary artery disease, aggressive
management with coronary angiography and either percutaneous intervention
or surgery is indicated. Patients with CKD have higher rates of restenosis
after percutaneous coronary artery interventions and have a greater risk for
death after coronary bypass grafting. Option A is correct. Patients who are
treated with percutaneous coronary artery intervention have a survival
advantage over those who are treated with medical therapy. It remains
controversial and unproved whether coronary artery bypass grafting ensures
better long-term survival over percutaneous intervention; therefore, option B is
incorrect. Some but not all studies have shown that bleeding complications
are the same with the use of platelet-like protein IIb/IIIa inhibitor therapy, and
most would recommend the use of these agents after coronary artery stent
placement.
Herzog CA: How to manage the renal patient with coronary heart disease: The agony and the
ecstasy of opinion-based medicine. J Am Soc Nephrol 14: 2556–2572, 2003
Reddan DN, Szczech LA, Tuttle RH, Shaw LK, Jones RH, Schwab SJ, Smith MS, Califf RM,
Mark DB, Owen WF: Chronic kidney disease, mortality, and treatment strategies among
patients with clinically significant coronary artery disease. J Am Soc Nephrol 14: 2373–2380,
2003
Keeley EC, Kadakia R, Soman S, Borzak S, McCullough PA: Analysis of long-term survival
after revascularization in patients with chronic kidney disease presenting with acute coronary
syndromes. Am J Cardiol 92: 509–514, 2003

Answer: 11
B - There are limited data to suggest that correction of anaemia in patients
with CKD is associated with regression of left ventricular hypertrophy and
could improve cardiovascular outcomes and mortality. Improvement of left
ventricular hypertrophy is associated with increase of haemoglobin levels
from 10 to levels of 11 to 12 g/dl. However tested in clinical trials, the
hypothesis that correction of anaemia ameliorates cardiovascular disease
outcome has not been confirmed. As an example, the Normal haematocrit
Study gave negative results.
Besarab A, Bolton WK, Browne JK: The effects of normal as compared with low hematocrit
values in patients with cardiac disease who are receiving haemodialysis and epoetin. N Engl
J Med 339: 584–590, 1998
Cianciaruso B, Ravani P, Barrett BJ, Levin A, ITA-EPO-7 Investigators: Italian randomized
trial of hemoglobin maintenance to prevent or delay left ventricular hypertrophy in chronic
kidney disease. J Nephrol 21: 861–870, 2008

205
Answer: 12
E - This patient has severe hyperparathyroidism with hypercalcaemia and
hyperphosphatemia. One could consider the addition of cinacalcet which
would decrease PTH and achieve small reductions in calcium and
phosphorus levels. However, even if PTH was reduced by 50%, the levels are
still unacceptably elevated. Thus, a parathyroidectomy would achieve a rapid
correction of the hyperparathyroidism.
Foley RN, Li S, Liu J, Gilbertson DT, Chen SC, Collins AJ: The fall and rise of
parathyroidectomy in U.S. haemodialysis patients, 1992 to 2002. J Am Soc Nephrol 16:210-
218, 2005

Answer: 13
B - The use of calcium based phosphorus binders has been associated with
progression of cardiac valvular and vascular calcification compared to the use
of sevelamer as a phosphate binder. Neither sevelamer nor lanthanum use
has been associated with increased incidence of adynamic bone and could be
used in this case to control hyperphosphatemia.
Asmus HG, Braun J, Krause R, Brunkhorst R, Holzer H, Schulz W, Neumayer HH:
Two year comparison of sevelamer and calcium carbonate effects on cardiovascular
calcification and bone density. Nephrol Dial Transplant 20:1653-1661, 2005

Answer: 14
D - Failure of nocturnal blood pressure to decline by at least 10%, one
commonly used definition of “non dipping” has been associated with more
rapid decline in GFR, as well as higher risk of mortality and ESRD.
Agarwal R, Andersen MJ: Prognostic importance of ambulatory blood pressure recordings in
patients with chronic kidney disease. Kidney Int 69(7):1175-80, 2006
Davidson MB, Hix JK, Vidt DG, Brotman DJ: Association of impaired diurnal blood pressure
variation with a subsequent decline in glomerular filtration rate. Arch Intern Med 166(8):846-
52, 2006

Answer: 15
D - Inflammation is common in CKD, and is predictive of cardiovascular risk in
this population. C-reactive protein (CRP) has emerged as a valuable indicator
of inflammation and predictor of cardiovascular risk. Although elevated CRP
concentrations probably reflect inflammation that occurs in association with
atherosclerotic disease, it has been hypothesized that CRP may actually play
more of a causative role, mediating processes involved in plaque progression.
Statin therapy is highly effective for reducing cardiovascular events among
patients without kidney disease, but there is less evidence in CKD. In
particular, the recent 4D study did not find benefit for stating in patients with
ESRD. Although there has been interest in lipid lowering therapy as a method
for preserving renal function, there is at yet little evidence to support its
efficacy. Hyperlipidaemia does occur commonly in later stages of CKD,
although the phenotype may differ from other more common forms of
hyperlipidemia. Treatment with atorvastatin has not been found to reduce the
need for ACEIs or ARBs in CKD.
Panichi V, Paoletti S, Mantuano E, Manca-Rizza G, Filippi C, Santi S, Taccola D,
Barsotti G: In vivo and in vitro effects of simvastatin on inflammatory markers in pre-dialysis
patients. Nephrol Dial Transplant 21(2):337-44, 2006

206
Wanner C, Krane V, Marz W, Olschewski M, Mann JF, Ruf G, Ritz E: German Diabetes and
Dialysis Study Investigators. Atorvastatin in patients with type 2 diabetes mellitus undergoing
haemodialysis. N Engl J Med 353(3):238-48, 2005
Menon V, Greene T, Wang X, Pereira AA, Marcovina SM, Beck GJ, Kusek JW
C-reactive protein and albumin as predictors of all-cause and cardiovascular
mortality in chronic kidney disease. Kidney Int 68(2):766-72, 2005

207
FURTHER NOTES

208
FURTHER NOTES

209
FURTHER NOTES

210

You might also like